ENT EXAM MASTER

Ace your homework & exams now with Quizwiz!

Case A 28-year-old man presents with diplopia and the inability to move the right eye outwards. He was hit by a ball on the right side of his face while playing volleyball 2 hours ago. His symptoms are non-progressive. On examination, his visual acuity is normal in both eyes. Right eye is medially deviated and cannot be moved laterally; otherwise, there is no abnormality detected. Question What nerve is most likely injured?

Correct answer: Abducens Explanation The abducens, or cranial nerve VI, is the most common nerve traumatized in head injuries. It is the only supply to the lateral rectus muscle of the eye, which is responsible for external or lateral deviation of the eye. Injury to this nerve results in esotropia (internal deviation) of the eye and failure to move the eye outwards. The oculomotor, or cranial nerve III, supplies most extraocular muscles except the SO (superior oblique) and lateral rectus. Injury results in external deviation, ptosis, and pupil dilatation. The trochlear nerve supplies the SO. An injury to the nerve alone is rare, resulting in defective downward and inward gaze. The facial, or cranial nerve VII, supplies the facial muscles; an injury to the nerve results in facial paralysis (i.e., inability to close the eye, inability to frown with the forehead, and loss of mouth movement with smiling). The trigeminal nerve supplies the muscles of mastication and sensation of the face. An injury to this nerve results in defective facial sensation and mastication.

Case A 5-year-old boy presents with sore throat, malaise, fever, and chills. His voice is muffled, speaking as if he holds a hot potato in his mouth, and he is drooling. On physical examination, he appears toxic. There is a visible bulge in his pharynx, pharyngeal swelling, uvula deviation, erythema, edema, purulent exudate, unilateral cervical lymphadenopathy, and neck rigidity. Question Ico-delete Highlights What is the most likely diagnosis?

Correct answer: Abscess formation Explanation An abscess is a collection of pus accumulated in a cavity formed by tissue and surrounded by a membrane, This patient probably has a pharyngeal abscess. Pharyngeal abscess is a disease of infants and young children; their retropharyngeal lymph nodes are not atrophied. Pharyngeal abscesses, which may be either retropharyngeal or parapharyngeal, often derive from a metastatic spread via the lymphatic system after an upper respiratory tract infection. Although this patient has symptoms of acute inflammation, granulomatous disease (with a course over months to years) is a chronic condition. Zenker's diverticulum is a pulsion diverticula of the hypopharynx that occur primarily in the elderly. Symptoms include dysphagia, fetor ex ore, regurgitation of undigested foods, and borborygmi in the neck. This patient is young and has symptoms of an acute systemic infection. Anaphylaxis is a severe multisystem reaction that develops over minutes to hours. It is due to the systemic effects of histamine released in type I hypersensitivity. Chronic inflammation generally has a course of weeks; symptoms are milder. Purulent exudate is not typical in chronic inflammation because the predominant cell types are mononuclears (i.e., lymphocytes, monocytes, macrophages, and plasma cells).

Case A 35-year-old woman presents with slowly progressive right-sided hearing loss, tinnitus, and continuous vertigo. Patient's Weber test reveals lateralization to the left ear. Question Ico-delete Highlights What is the most likely cause of this patient's symptoms?

Correct answer: Acoustic neuroma Explanation Acoustic neuroma is a benign tumor of the VIII cranial nerve. While these tumors are benign, their growth can lead to sensorineural hearing loss, vertigo, and tinnitus. Because the tumor is slow growing, symptoms typically have a gradual onset. Diagnosis is made by MRI. In vestibular neuronitis, patients present with paroxysmal episodes of vertigo without hearing loss. The patient may also have nystagmus. Ménière's disease also presents with vertigo, sensorineural hearing loss, and tinnitus, but these symptoms are typically episodic in nature, whereas the symptoms of acoustic neuroma are continuous. Labyrinthitis presents with an acute onset of continuous vertigo, hearing loss, and tinnitus. Symptoms gradually improve over several weeks, though hearing may remain impaired following recovery. Acoustic neuroma has a gradual onset, and symptoms will not improve until the tumor is treated. Patients with benign paroxysmal positional vertigo have recurrent episodes of brief vertigo without hearing loss; the vertigo is related to changes in head position. There is no associated hearing loss.

Case A 64-year-old Asian man presents with a 1-hour history of severe right eye pain that started while he was watching a movie at the theater. He notes blurred vision and seeing halos around lights when using his right eye. He denies loss of vision, trauma, discharge, and any symptoms in the left eye. His last eye exam was 6 months ago, which resulted in new glasses. Past medical history is negative, and the patient denies any allergies. On physical exam, visual acuity is OS 20/25, OD 20/70, and OU 20/40. Pupil on right eye is 7 mm and left eye is 3 mm. Right pupil is non-reactive to light; left pupil is reactive to light. Right cornea is steamy in appearance and left cornea is clear. Question Ico-delete Highlights What is the most likely diagnosis?

Correct answer: Acute angle-closure glaucoma Explanation Acute angle-closure glaucoma is frequent among the older age group and in the Asian population. Essential for diagnosis is rapid onset of severe pain and profound vision loss/blurring with halos around lights. On physical exam, a fixed and dilated pupil and a red eye with a steamy cornea are the hallmarks. Chronic glaucoma is usually an insidious onset of bilateral loss of peripheral vision. Cataracts usually present as gradually progressive and non-painful; they present with clear corneas, normal pupil size, and normal light reaction. Acute uveitis presents with blurred vision; the cornea is clear and the affected pupil is small with poor papillary light response. Acute conjunctivitis presents with copious discharge; it does not impact vision. There is a clear cornea, normal pupil size, and normal papillary light reaction.

Case A 40-year-old man presents with severe pain in his left eye, decreased vision, nausea, and abdominal pain. On examination, the patient's left pupil is moderately dilated and nonreactive. The cornea is 'steamy' in appearance and generally the eye is red. Question What is the diagnosis?

Correct answer: Acute angle-closure glaucoma Explanation This patient has acute angle-closure glaucoma. The typical characteristics of this condition are all exhibited by this patient (i.e., steamy cornea, severe pain, blurred vision, dilated and nonreactive pupil). A physical exam finding in a patient with acute uveitis would be a small pupil. In addition to these symptoms, patients may also present with abdominal pain. A corneal infection and corneal ulcer would cause circumcorneal injection and watery, or purulent, discharge.

Case A 42-year-old man presents with a 3-day history of headache, stuffy nose, greenish nasal discharge, and low-grade fever. He also has body aches, facial pain, and a dry cough. He denies shortness of breath, abdominal pain, nausea, and vomiting. He has no past medical history; he is on OTC meds, including acetaminophen, for his condition. On exam, he has a temperature of 99.9°F, a pulse of 86/min, BP is 120/76 mm Hg, and SPO2 is 92%. Lungs are clear, and the abdomen is normal. Nasal mucosa appears boggy, and there is tenderness over the facial bones (maxillary area). Pharynx is without exudates. Question What is the most likely diagnosis?

Correct answer: Acute sinusitis Explanation Acute sinusitis is a common condition characterized by nasal congestion, purulent nasal discharge, headache, facial pain, cough, teeth pain, myalgias, and low-grade fever. Other symptoms include halitosis, anosmia, and metallic taste in the mouth. Out of these 11 features, presence of 4 or more is diagnostic for bacterial sinusitis. History regarding previous medications (rhinitis medicamentosa), occupational rhinitis, polyps in the nose, or vasomotor rhinitis needs to be taken. Maxillary sinuses are tender, as in this case, along with congested nasal mucosa. Diagnosis is usually clinical. Radiological confirmation may aid in definitive diagnosis and treatment. Transillumination of maxillary sinuses with a torchlight may reveal loss of normal illumination, but doing so is generally not recommended in practice. Plain radiographs in various sinus views may show opacification or air-fluid level. The imaging of choice in recent times has been a limited CT scan of sinuses, which is fairly reasonable price-wise and has proven to be quite helpful. It can show opacification, mucosal thickening, abscess formation, bone destruction, tumors, and blockage of the osteomeatal complex. Treatment includes a 2-week course of amoxicillin, amoxicillin-clavulanate, cefuroxime, or quinolones, along with an oral decongestant, nasal decongestant, and anti-inflammatory medications. Acute rhinitis is usually viral and characterized by serous nasal discharge, rather than purulent, fever, fatigue, sneezing, scratchy throat, and moderate cough. Examination is not very remarkable. It is self-limiting, and it resolves after 4-7 days. There is no response to antibiotics. Symptomatic treatment with decongestants and antitussives is indicated. Migraine headaches are not associated with nasal congestion, fever, cough, or sore throat. Recurrent headaches are common, and patients may get an aura prior to an attack. Acute pharyngitis is associated with severe sore throat, pain on swallowing, erythematous pharynx with or without exudates, and enlarged tonsils. There may be headache and body ache with fever, especially in streptococcal infections. Throat swab can be sent for culture. Office-based rapid streptococcus tests are also available. Sinus pain and tenderness are absent, and nasal congestion is absent or mild. Influenza is a viral illness with high fever and chills, severe myalgias, headache, malaise, and fatigue. It can be debilitating and life-threatening in the elderly population. Dry hacking cough with sore throat may be present. Examination reveals few positive findings. Mild hyperemia of the pharynx and flushed face may be the only findings other than fever. Diagnosis is clinical. Treatment is supportive.

Case A 28-year-old man presents with "burning mouth." The patient states he has had an unusual burning sensation on the inside of his right cheek for the last 3 days; he feels that the burning is worsened with hot, spicy, or acidic foods. After performing a comprehensive oral exam on this patient you note 3 lesions on the buccal mucosa on the right side of the oral cavity. You further document the description of these lesions as the following: 3 round lesions, each measuring approximately 3 mm in diameter with presence of a white-yellow center surrounded by a red halo. Question Based on the given history and physical examination, what is the most likely diagnosis that this patient is experiencing?

Correct answer: Aphthous ulcers Explanation The patient above is experiencing an episode of aphthous ulcers, also sometimes referred to as a canker sores or aphthous stomatitis. These painful, open sores are found in the oral cavity and are the most common form of mouth ulcer. They are benign, noncancerous, noninfectious, and in most cases the cause is unknown. Many times the lesions are described as having a white or yellow center and are surrounded by a bright red area. Fordyce spots, also known as Fordyce granules, is also not the correct answer. These are sebaceous glands that appear as small yellowish spots in the buccal mucosa or even on the lips. Fordyce spots are a benign finding. Koplik's spots are a sign indicating measles. These will be small white specks that resemble grains of salt on a red background. Many times they will appear on the buccal mucosa near the first and second molars. Leukoplakia is described as a thickened white patch anywhere in the oral mucosa that results from chronic local irritants, such as chewing tobacco. Typically this is a benign reactive process but can eventually lead to a malignant process and should be biopsied. Oral candidiasis, more commonly known as thrush,is an incorrect choice in this scenario. Oral candidiasis lesions usually consist of creamy-white patches overlying erythematous mucosa that are notably painful to the patient. The white patches usually can be rubbed off. Although thrush may be encountered in infants and babies, the following adult patient populations commonly encounter this pathology: those who wear dentures, prolong history of poor oral hygiene, diabetic patients, anemic patients, those patient using antibiotic therapy, those undergoing chemotherapy or local radiation, and those using daily corticosteroids (both oral and inhaled versions). The patient above possesses at least 2 of these significant risk factors for thrush.

Case A 13-year-old girl presents with a 2-week history of nasal congestion; it is associated with purulent nasal discharge, pain in the right side of her face just under her orbit, intermittent mild headache, and cough. Pertinent physical examination findings include oral temperature of 100°F, edematous nasal mucosa, and tenderness over the right maxillary sinus. Question What is the most likely diagnosis?

Correct answer: Acute sinusitis Explanation She most likely has acute sinusitis. The purulent nature of her nasal discharge is likely to be associated with an infection, and patients with acute sinusitis typically experience nasal congestion with discharge. Headache, along with otalgia, halitosis, dental pain, and ear pressure or pain are other symptoms that may be associated with acute sinusitis. The diagnosis of acute sinusitis is usually given when the symptoms have been persistent for at least 10 days, but less than 4 weeks. In addition, acute sinusitis can be associated with a fever. The sinuses are not fully developed until about age 20, so secondary infection following a viral upper respiratory infection (URI), allergic rhinitis, or asthma flare is common. She would not be diagnosed with chronic sinusitis, as symptoms need to be present for more than 12 weeks in order to be considered chronic. In addition, fever is not typical in chronic sinusitis. The patient may have had an URI initially, which can lead to acute sinusitis. However, her diagnosis at the time of her appointment would not be URI for several reasons. A URI is a viral disorder, and the nasal drainage is not typically purulent, but rather clear. In addition, the patient would not have tenderness over her maxillary sinus if her only diagnosis was an URI. Allergic rhinitis is a potential diagnosis for the patient due to the nasal congestion and cough. However, her nasal drainage would not likely be purulent in allergic rhinitis. Fever and maxillary sinus tenderness would not be present on physical examination of a patient with allergic rhinitis. If she had allergic rhinitis, she would also likely be experiencing pruritus of her nose, eyes or ears, sneezing, and watering of her eyes. The patient had a headache as one of her symptoms, which is common for patient with acute sinusitis due to the congestion. However, she would not be diagnosed with cluster headaches. While cluster headaches can be associated with some nasal drainage, the drainage is usually clear and not purulent. In addition, cluster headaches are defined as episodes of severe, unilateral, sharp pain usually in the periorbital area and temple. These headaches can occur daily at similar times each day, and may cause patients to be restless. They are usually present for 4-16 weeks, and the general nature of the headaches is episodic and severe.

Question A previously healthy 5-year-old boy presents with complaints of bilateral eye pain and redness. The child's mother reports that several children in his daycare facility have had similar symptoms. Results of physical exam indicate bilateral nonsuppurative conjunctival inflammation, photophobia, and preauricular lymphadenopathy. What is the most likely organism responsible for this outbreak of conjunctivitis? 1 Haemophilus ducreyi 2 Adenovirus 3 Neisseria gonorrhoeae 4 Chlamydia trachomatis 5 Human herpesvirus 6

Correct answer: Adenovirus Explanation A common source outbreak of epidemic keratoconjunctivitis, 'pink eye', is usually a result of contact with other infected individuals or improperly cleaned optical instruments. In this case, the acute onset, bilateral presentation, and epidemic nature point to adenovirus (types 8, 19, and 37) as the most likely pathogen. Ophthalmic infection with this highly communicable virus, for which humans are the only known reservoir, is about the only time the preauricular lymph node is evident. Haemophilus ducreyi is the causative agent of chancroid, a sexually transmitted disease causing a hard, localized chancre in the genital area. Neisseria gonorrhoeae may cause conjunctivitis in adults and can be transmitted to neonates. Chlamydia trachomatis is the leading cause of blindness worldwide. Infection with this obligate, intracellular pathogen results in trachoma (a chronic keratoconjunctivitis) in endemic areas such as Africa, Asia, and the Mediterranean. Chlamydia trachomatis also causes inclusion conjunctivitis, the most commonly seen infection in sexually active young adults. Infection occurs from inoculation of the eye with infected genital secretions. Human herpesvirus 6 (HHV-6) is a common herpesvirus about which little is known. HHV-6 causes a maculopapular rash in infants known as roseola infantum, and may also be a common cause of high fevers in children.

Case A 3-day-old female newborn presents with rapidly progressing bilateral conjunctivitis with white discharge. She was born full term via precipitous vaginal delivery; her mother had no prenatal care. On exam, she is alert and active. The eye discharge is purulent, and she has bilateral eyelid edema. Question Ico-delete Highlights What should be the next step in the management of this newborn?

Correct answer: Admit to the hospital for IV antibiotics and evaluation. Explanation The correct response is to admit the infant to the hospital for IV antibiotics and evaluation. Ophthalmia neonatorum is a form of conjunctivitis occurring in infants younger than 4 weeks. The usual incubation period is 2-5 days for N. gonorrhoeae, and 5-14 days for C. trachomatis. Conjunctivitis due to silver nitrate drops usually occurs 6-12 hours after birth. The condition may also be caused by a virus, most commonly herpes simplex. Due to the timing and severity of conjunctivitis, in conjunction with the fact that the mother had no prenatal care, it is possible that this newborn has gonococcal conjunctivitis. Giving an IM dose of ceftriaxone is not enough. Due to the possibility of blindness, gonococcal meningitis, or sepsis, the patient should be monitored closely and admitted to the hospital for eye, blood, and (possibly) CSF cultures. Tests for other infections (e.g., C. trachomatis, syphilis, hepatitis B, and HIV) should be performed.

Question A patient presents with epistaxis. What in the history and physical is most important to obtain?

Correct answer: Airway patency Explanation Many aspects of the history and physical are important to evaluation of epistaxis (or nose bleeds). However, the most urgent and important factors initially are assuring airway patency and hemodynamic stability. In cases of severe trauma or posterior epistaxis, bleeding can be brisk and compromise the airway. History of easy bruising can help establish the possibility of a bleeding disorder as the pathology behind the patient's epistaxis. Signs of facial trauma (as well as history of trauma) help the clinician to assess the cause of the epistaxis and address other potential injuries or conditions. The age of the patient is not particularly helpful in evaluation of epistaxis. In some cases, pediatric patients may be more likely to put foreign bodies in their noses, and elderly patients may be slightly higher risk for posterior epistaxis. The color of the blood is not helpful in evaluation of epistaxis.

Case A 25-year-old male medical resident presents with 'acute red eyes' with copious watery discharge. He also notes some aversion to bright light. While rubbing his left eye, he describes a sensation of a 'gritty' foreign body. Other than the aversion to bright light, he denies any visual disturbance or pain. On physical exam, the conjunctiva of both eyes are injected and mildly edematous. The remainder of the exam is within normal limits. Question What is the most likely diagnosis?

Correct answer: Allergic conjunctivitis Explanation Allergic conjunctivitis is almost always secondary to environmental allergens; therefore, it usually presents with bilateral symptoms. The other hallmark symptom would be pruritus, which might be suggested by the rubbing of his eye. Infections caused by bacteria, including chlamydial organisms, are transmissible by eye-hand contact. Often, these infections initially present in 1 eye, with the 2nd eye becoming involved a few days later. Bacterial infections present with a purulent discharge, as opposed to a clear one. Contact lens abuse would often lead to a purulent discharge; it is often unilateral. Hyperacute conjunctivitis has an abrupt onset, and it is characterized by a copious yellow-green purulent discharge that reaccumulates after being wiped away. The symptoms of hyperacute conjunctivitis, which are typically rapidly progressive, also include redness, irritation, and tenderness to palpation. Patients demonstrate marked conjunctival injection, conjunctival chemosis (excessive edema), lid swelling, and tender preauricular adenopathy.

Question Ico-delete Highlights A 4-year-old boy has had intermittent rhinorrhea, nasal congestion, and cough for about 2 - 3 weeks. His mother says he had felt warm at night when he first became ill and sneezes occasionally. He goes to a large day care 4 days a week. Otherwise he has been healthy except for an occasional dry, itchy rash that he has had on and off for "a long time". Mom has been treating this with OTC moisturizers. On exam, his temperature is 98.4F, respirations 24, pulse 86 beats/min. He appears somewhat tired, with dark circles under his eyes. There is slightly cloudy nasal drainage, turbinates seem a little boggy, lungs are clear, ears and throat are normal and his neck is supple without any lymphadenopathy. His skin exam reveals a fine roughened, slightly hyperpigmented maculopapular rash to the dorsal surfaces of both forearms and with a few healing excoriations covered with a greasy film. What is the most likely diagnosis?

Correct answer: Allergic rhinitis Explanation Allergic rhinitis can frequently be mistaken for a viral or bacterial infection of the upper respiratory tract especially in patients too young to communicate their symptoms well. Allergic rhinitis is a hypersensitivity reaction to specific allergens that is mediated by immunoglobulin (Ig)E antibodies in sensitized patients resulting in inflammation. It can be mild or severe and may be seasonal or perennial. Allergic rhinitis also may be intermittent or persistent. Children who have 1 component of atopy such as asthma or as in this child eczema, have a 3-fold greater risk of developing a 2nd component. A variety of signs and symptoms may be present such as mouth breathing, snoring, nasal sounding voice, paroxysmal sneezing, nasal itching, snorting, nose blowing, nasal congestion and drainage, and occasional coughing. As a result patients can also experience headaches, fatigue, impaired concentration, decreased sleep, and social functioning. Classic findings of pale, bluish nasal mucosa, enlarged or boggy turbinates, clear nasal secretions, and pharyngeal cobblestoning may or may not always be seen. Certain characteristic signs include allergic shiners due to suborbital edema and allergic crease or a transverse skin line across the bridge of the nose due to constant rubbing upward. Some children may display chronic mouth breathing resulting in orthodontic disturbances, such as palatal arching, increased facial length and flattened mid-face. Infectious rhinitis will typically present in the early years of childhood before allergic rhinitis occurs. Nasal secretions are commonly mucopurulent; there may be a posterior pharyngeal discharge and fever. Chronic infectious rhinosinusitis also involves a mucopurulent nasal discharge, postnasal drip with cough, and an olfactory disturbance for at least >28 days. Vasomotor rhinitis consists of profuse rhinorrhea and nasal obstruction occurring when experiencing a change in environmental conditions, such as going from a warm house to frigid outdoor temperatures. Though a foreign body could be likely in this age group, especially with a history of day care attendance, it usually will present with persistence of unilateral nasal obstruction and a purulent, foul smelling nasal discharge.

Case An 8-year-old girl presents with her mother to the pediatrician's office with persistent clear nasal drainage and nighttime cough for the past month. Her physical examination reveals clear rhinorrhea, dark circles under her eyes, and a transverse nasal crease. Question What is the most likely diagnosis?

Correct answer: Allergic rhinitis Explanation Allergic rhinitis is a disorder of the nasal mucosa caused by IgE-mediated inflammation. It can present at almost any age, but incidence decreases with advanced age. In children, allergic rhinitis can occur along with atopic dermatitis and/or food allergies. The symptoms typically include persistent clear nasal drainage, stuffiness, and/or congestion. Pruritus of the eyes, nose, palate, or ears can occur. Patients often sneeze and have postnasal drainage that can lead to a bothersome cough, particularly at night. The physical examination can reveal allergic shiners, which are dark circles under the eyes. Presence of a clear nasal discharge is usually seen as well. A transverse nasal crease can be seen in children from them rubbing their noses upwards rather than using a tissue. A nasal foreign body is usually unilateral, so the drainage is also usually unilateral. The drainage would not usually be clear either, but purulent with an odor. Sinusitis would present more acutely than with a month of symptoms, and any drainage is likely to be thick and purulent. Physical examination would not likely reveal allergic shiners or a crease on the nose, but it may reveal fever, edema of the nasal mucosa, and tenderness over the sinuses. The symptoms of allergic rhinitis and upper respiratory infection (URI) may be very similar, but patients with a URI would likely not be symptomatic for a month and might have various systemic symptoms. A nasal crease and allergic shiners are also not likely to be present. Patients with influenza would not have symptoms for a month and would usually have systemic symptoms, such as malaise, fever, and arthralgias. The cough would usually be more significant than a bothersome nighttime cough secondary to postnasal drip. A nasal crease and allergic shiners are also not likely to be present.

Case A 72-year-old man presents with a neck mass. Examination reveals a mass within a multinodular goiter. The patient undergoes a fine needle aspiration, which reveals cancerous cells. Question What is the most aggressive type of thyroid cancer?

Correct answer: Anaplastic carcinoma Explanation Anaplastic carcinoma is the correct response. Anaplastic carcinoma is the most aggressive form of thyroid cancer. It is associated with gene mutations and early metastasis. Papillary carcinoma, follicular carcinoma, medullary carcinoma, and squamous cell carcinoma are all incorrect responses. Anaplastic carcinoma is the most aggressive type of thyroid cancer.

Case A 21-year-old Caucasian man reports symptoms of sneezing, runny nose, itchy nose and eyes, and occasional cough occurring intermittently. His symptoms typically occur following exposure to pollen or pet dander. He describes his symptoms as mild and intermittent; they do not negatively impair his quality of life. Past medical history includes a fractured collarbone in childhood; there is no other significant past medical history. He denies any history of high blood pressure or enlarged prostate. Previous surgeries: None. Medication allergies: None known. Usual home medications: Occasional Tylenol for headache. Social History: The patient is a full-time engineering student. He does not use alcoholic beverages or illicit drugs. Family history: Both parents are alive and in good health. He has one brother who has asthma and eczema. He has no children. Review of systems: occasional headaches, otherwise non-contributory. Physical Examination: General - Alert white male with dark circles under both eyes. Vital Signs: Temperature 98.4°F, pulse 72 and regular, respirations 12, blood pressure 124/76. HEENT: Normocephalic. EOMs intact. PERRLA. Erythematous injection of the conjunctiva is noted. Pale, boggy nasal mucosa is present. Oral mucosa is pink with a small amount of post-nasal drainage present. Heart: Normal S1 and S2 without rub, murmur, or gallop. Lungs are clear to auscultation and percussion. No rales, rhonchi, or wheezes. The remainder of the physical examination is unremarkable. Question Which treatment is the best initial choice for this patient?

Correct answer: Antihistamine Explanation The correct answer is antihistamine. Patients who only have intermittent symptoms are often adequately initially treated with oral antihistamines. This patient has symptoms of episodic allergic rhinitis that are mild and intermittent without prominent symptoms of nasal congestion. The mild cough is likely related to post-nasal drip. For patients with allergic rhinitis whose primary symptoms are sneezing and itching, the treatment of choice is second-generation non-sedating oral antihistamine for use on an as-needed (prn) basis. Patients with prominent symptoms of nasal congestion may not obtain adequate relief of nasal congestion with antihistamines alone; if adequate treatment is not achieved, a glucocorticoid nasal sprays may be prescribed. Decongestants may be useful in reducing nasal congestion in allergic rhinitis, but they are not first-line therapy for the condition. This patient did not report significant symptoms of nasal congestion. Intranasal corticosteroids are the treatment of choice for patients with symptoms of allergic rhinitis that are persistent or affect quality of life. This patient has only mild, episodic symptoms that do not negatively impact his quality of life. Saline nasal wash may be used as a sole or complementary treatment for allergic rhinitis, but it is not the first-line treatment of choice. While topical nasal decongestant spray may be effective in reducing nasal congestion on a temporary basis, rebound nasal congestion may result; recommended usage is generally limited to 3-5 days. Prolonged use of topical nasal decongestant sprays may result in a condition known as "rhinitis medicamentosa" associated with persistent nasal congestion.

Question Ico-delete Highlights A 28-year-old woman presents with an itchy throat, prolonged sneezing episodes, watery red eyes, and inflamed nasal membranes. Her temperature is normal and a throat culture is negative. She is most likely suffering from allergic rhinitis. Her physician will most likely prescribe diphenhydramine. To what class of compounds does the drug of choice belong?

Correct answer: Antihistamine (H1) Explanation Antihistamines are drugs that have major applications in treating the symptoms of allergic rhinitis and urticaria. They may also be used in treating motion sickness and nausea. Some antihistamines, because of their strong sedative properties, are used in the treatment of insomnia; therefore, they also find their way into many over-the-counter sleep aids. The most common adverse effect observed with H1 receptor blockers is sedation. Other effects seen are tremors, blurred vision, lassitude, dizziness, fatigue, drying of the nasal passages, and dry mouth. Antihistamines can interact with other drugs, leading to serious consequences such as the potentiation of the effects of CNS depressants (alcohol, etc.). Monoamine oxidase (MAO) inhibitors can potentiate the anticholinergic effects of antihistamines. In spite of this, H1 receptor blockers are relatively safe. Chronic toxicity is rare; however, acute poisoning is common, especially among children, and leads to dangerous effects such as hallucinations, ataxia, convulsions, and (if untreated) coma and cardiorespiratory collapse.

Question During a routine physical exam, a 16-year-old boy notes a painful lesion on the inside of his cheek. A round whitish-gray ulcer with an erythematous surrounding halo is noted on exam. The lesion is tender to the touch. The patient denies any medical, surgical, sexual, or social history. What is the primary diagnosis?

Correct answer: Aphthous ulcer Explanation This lesion is likely an aphthous ulcer, also known as a canker sore. An aphthous ulcer is a painful round whitish-gray ulcer that has a classic red surrounding halo. It is self-limiting and will heal within 1-1.5 weeks. Second-stage syphilis presents as a mucous patch that is gray in color, raised, oval-shaped, and may be distributed in multiple areas in the oral cavity. Syphilis needs to be treated with penicillin. Caviar lesions are varicosities found on the dorsal side of the tongue only. This type of lesion does not have any significance clinically; no intervention is needed. Tori mandibulares are benign, painless bony projections found on the inner side of the lower teeth. The mucosa covering these tori are of normal color and appearance. Leukoplakia is characterized by non-tender white patches found on the inside of the mouth. Biopsy is necessary to determine malignancy.

Case A 13-year-old girl presents with a painful reddened lesion on the inside of her cheek. She has noticed she had this lesion a few days ago and it will not go away. Upon physical exam, the inside of her right cheek had a yellow-grey appearance with an erythematous halo surrounding the lesion. Patient overall feels well besides her specific complaint, and no other rashes or lesions are noted. All laboratory findings are within normal limits. Additionally, HSV-1 and HSV-2 are both negative. Question What is the most likely diagnosis?

Correct answer: Aphthous ulcers Explanation The clinical picture is suggestive of aphthous ulcers,due to the description of the lesion described in the stem of the problem. The yellow-grey appearance and erythematous halo surrounding the lesion is descriptive of aphthous ulcers. Folate deficiency is not the correct answer, as even though the patient will present with mouth sores, patients also present with fatigue, which is not described in this patient. Herpangina is not correct because the patient usually presents with lesions or sores not only in the mouth, but also on the hands and feet. Additionally, the lesions appear with a white base and a red border that is not descriptive of aphthous ulcers. Herpes Labialis, or cold sore,is not correct because the patient would also be diagnosed with herpes simplex, which is not found with the patient in the stem of the problem. Additionally, HSV-1, and HSV-2 are both negative. Lichen planus is not correct because patient would present with a lacy white patch, which is not present in the patient.

Case A 25-year-old man presents to his primary care provider with a 2-day history of acute pain in his mouth; he has no significant past medical history. He states that the pain is in a localized part of his gums; it is aggravated by eating food. It is also aggravated when he speaks and when he smiles. He denies any history of smoking, risky sexual behavior, drug use, transfusions, trauma, fever, chills, otalgia, otorrhea, rhinitis, eye pain/discharge, halitosis, regurgitation, skin changes, swollen glands, sore throat, or headache. His physical exam is noteworthy for the following finding (see image). Question What would be the most appropriate intervention at this time?

Correct answer: Application of topical corticosteroids Explanation The correct response is application of topical corticosteroids. Topical corticosteroids, such as triamcinolone acetonide, 0.1%, or fluocinonide ointment, 0.05%) in an adhesive base (Orabase Plain) appear to provide symptomatic relief in many patients. Aphthous ulcers are very common and easy to recognize. Their cause remains uncertain, although an association with human herpesvirus 6 has been suggested. Found on freely moving, non-keratinized mucosa (such as buccal and labial mucosa and not attached gingiva or palate), they may be single or multiple. They are usually recurrent, and they appear as painful small round ulcerations with yellow-gray fibrinoid centers surrounded by red halos. Minor aphthous ulcers are < 1 cm, and they generally heal in 10 - 14 days. Major aphthous ulcers are > 1 cm; they can be disabling due to the degree of associated oral pain. A referral to a head and neck surgeon is not necessary at this time. Treatment or prevention of apthous ulcers with vitamins has not been proven beneficial. Treatment with antibiotics is not warranted at this time; this is not a bacterial infection. Other topical therapies shown to be effective in controlled studies include diclofenac 3% in hyaluronan 2.5%, doxymycine-cyanoacrylate, mouthwashes containing the enzymes amyloglucosidase and glucose oxidase, and amlexanox 5% oral paste. A 1-week tapering course of prednisone (40 - 60 mg/d) has also been used successfully. Cimetidine maintenance therapy may be useful in patients with recurrent aphthous ulcers. Thalidomide has been used selectively in recurrent aphthous ulcerations in HIV-positive patients.

Case A 22-year-old woman presents with nosebleed. Her nosebleed occurred spontaneously with no known injury. Her past medical history is significant for seasonal allergies and migraine. Examination reveals bleeding from the nasal septum. Question What is the appropriate initial treatment for this patient?

Correct answer: Apply direct pressure with patient leaning forward Explanation Apply direct pressure with patient leaning forward is correct. The patient has epistaxis. The majority of cases of epistaxis originate from Kiesselbach's plexus in the anterior nose. Patient should first have direct pressure applied to the nose while the patient is leaning forward. Pressure should be continued for a minimum of 5 minutes. Have patient lean back to drain blood from nose is incorrect. Leaning back will cause the blood to collect in the posterior pharynx. Blood in the posterior pharynx may cause nausea or airway obstruction. Posterior nasal packing is incorrect. This patient has anterior epistaxis, so posterior packing should not be done. Cauterization of the nasal cavity is incorrect. Cauterization may be required for severe bleeding, but it is not a first line treatment. Surgical ligation of the nasal artery supply is incorrect. Surgery may be required for severe bleeding, but it is not a first line treatment.

Case A 19-year-old woman with asthma presents with a decreased sense of smell and a feeling of obstruction in her nasal cavity. Her asthma has been well controlled, and she denies any wheezing or shortness of breath. On exam, her lung sounds are normal, but there is the presence of multiple pale swollen masses in her nares. Question In addition to treatment of her condition, which medication should be avoided?

Correct answer: Aspirin Explanation Aspirin should be avoided in this patient. Given the description of pale swollen masses, it is likely that the patient has nasal polyps. Patients with a history of asthma and nasal polyps may also have a sensitivity to aspirin that causes bronchospasm. The combination of asthma, nasal polyps, and aspirin sensitivity is known as the Samter triad. Due to this potential reaction, aspirin should be avoided in this individual. Acetaminophen, ibuprofen, and oxycodone are not contraindicated in patients with asthma and nasal polyps. Prednisone is not contraindicated in patients with nasal polyps. In fact, a short course of nasal or oral steroids may actually treat nasal polyps. For those refractory to medical management, surgical removal of nasal polyps may be necessary.

Case A 22-year-old woman was scuba diving in Florida last week and presents due to clogging of the right ear, hearing loss, dizziness, ringing of the ear, and ear pain. Upon physical exam, a thorough ear exam was conducted. The eardrum appears slightly pushed outward from where it normally sits. Laboratory results showed no abnormal findings. Question What is the most likely explanation of the findings?

Correct answer: Barotrauma Explanation The clinical picture is suggestive of barotrauma. The patient has a history of a pressure change from diving and has symptoms of ringing of her ear, dizziness, and headaches. Furthermore, her physical exam helps confirm it. Acoustic neuroma is not the correct answer because a patient with a tumor would have gradual symptoms that presents with hearing loss, tinnitus, taste disturbances, and dizziness, which are not present in this patient. Acute otitis media is not the correct answer, because the patient usually presents with a headache, fever, otalgia, and otorrhea. It usually occurs in children. Otitis externa is not the correct answer because otitis externa is an infection of the outer ear, whereas a barotrauma is inside the ear such as this presentation. Otitis media with effusion is not the correct answer because the patient does not present with fluid in her ear. Additionally, otitis media with effusion usually occurs in patients between 6 months and 3 years.

Case A 77-year-old male farmer has been your patient for many years. He presents with a sore that will not heal. When you see this patient, he describes an area on his right cheek that has been present for the last 6 months. At first, it was just a bump, and it had a pearly color to it. There is now is an open sore on the area, which has been present for the last 2 weeks and has not healed; it is what brings the patient in to see you today. Question Based on the history and physical examination findings of this patient, what is the most likely diagnosis is this case?

Correct answer: Basal cell carcinoma Explanation The lesion being described in the above patient scenario is most likely to be basal cell carcinoma. Key characteristics of lesions of basal cell include having a pearly, papular, erythematous patch that is usually larger than 6 mm or possibly appearing as a non-healing ulcer. The areas in question will commonly be found on sun-exposed areas of the body, specifically the face, trunk, or lower legs. Squamous cell carcinoma appears as a non-healing ulcer or a wart-like nodule. This also is due to long time sun exposure and is especially common in patients who are fair-skinned or those that are organ transplant recipients. Malignant melanoma is another type of skin cancer that patients may present with but the characteristics are distinctly different from the 2 previous skin cancers described. Malignant melanoma lesions may be flat or raised, and they have irregular borders; initially, they are usually a pigmented area on the skin that has recently noticeably changed in appearance. Clinicians and patients will use the mnemonic "ABCD" rule when evaluating pigmented areas: asymmetry, border irregularity, color variegation, and diameter ? 6 mm. Actinic keratosis is skin lesions that are not a form of cancer. These are small (0.2 - 0.6 cm) macules or papules that typically flesh-colored, pink, or even slightly hyper-pigmented. They possess a sandpaper consistency and may be tender when palpated. These also occur in sun-exposed area of the skin in fair skinned patients. Actinic lentigines are also known as liver spots. They commonly present in older patients on areas that have significant exposure to the elements of weather, in such areas as the dorsum of the hand, wrist, and forearm. Lesions of actinic lentigines are also a benign condition.

Case A 58-year-old Caucasian man presents with a bleeding mole on his face. The mole is located on his left cheek has been present for the past several years but has started to spontaneously bleed over the last 3 months. The patient denies any other moles with the same characteristics and just wants it taken care of so it is not as bothersome. The patient denies weight loss, night sweats, or fevers; he has no recent changes in his appetite or sleeping issues. He is a farmer and owns over 100 acres that he plants and harvests yearly and has done so for the past 40 years. On physical examination, you find a 3 cm pink papule that is pearly in appearance and possesses a telangiectatic characteristic to it. Central ulceration is present. Question Given the history and physical exam findings, what is the most likely diagnosis for this patient?

Correct answer: Basal cell carcinoma Explanation The patient in this clinical case scenario most likely has basal cell carcinoma (BCC). BCC is a common skin cancer that arises from the basal layer of the epidermis. BCC is particularly common in Caucasians and has a 30% higher incidence in men than in women. BCC incidence also increases with age, with persons aged 55-75 having a 100-fold higher incidence than those under 20. BCC clinically presents in four or so different clinical types, the nodular form being the most common (making up 60% of the BCC cases). Nodular BCC is described as a pin or flesh-colored papule that is pearly or translucent and evidence of a telangiectatic vessel within the papule. Ulceration is common, sometimes referred to as a "rodent ulcer." The other types of BCC include superficial, morpheaform, other subtypes, and even several BCC syndromes. Squamous cell carcinoma of the skin typically appears as small, red, conical, hard nodules that occasionally will ulcerate. The presence of a pearly appearance helps distinguish BCC from squamous cell, although the two malignancies present in the same patterns. Malignant melanoma of the skin is described as being a flat or raised pigmented lesion. The mnemonic of the "ABCD" rule is what is used to help screen suspicious lesions: Asymmetry, Border irregularity, Color variegation, and Diameter > 6 cm. Benign nevi are common, but any skin lesion that has an ulcer and tendency to bleed should be urgently evaluated to rule out the worst-case scenario. Actinic keratosis is also an incorrect choice. These are generally small (0.2 cm-0.6 cm) macules or papules that could be flesh color, pink, or even slightly hyper pigmented; however, they will feel like sandpaper and are generally tender when palpated. The clinical scenario does not match this description.

Case A 72-year-old woman presents with a severe unilateral headache, jaw pain, and scalp tenderness. The patient states the headache is of a piercing quality; her jaw hurts only when she chews, and feels better a few minutes after she stops chewing. A pulsation of the temporal artery on the same side as her headache cannot be appreciated, and prednisone is prescribed until the patient can see a specialist. Question What is the reason prednisone is prescribed in this case?

Correct answer: Because the patient is at risk for blindness Explanation The clinical picture is suggestive of giant cell (temporal) arteritis; this is a systemic arteritis affecting the medium and large blood vessels (arteries) in patients over 50 years of age. It is often referred to as temporal arteritis because it frequently involves the extracranial branches of the carotid artery. Signs and symptoms include headache, scalp tenderness, jaw claudication, visual problems, and throat pain. If not treated appropriately, blindness can occur due to the involvement of the posterior ciliary branch of the ophthalmic artery. Prednisone should be initiated immediately. This patient has a vasculitis, not a stenosis, of the artery on the same side as the headache; contralateral is the opposite side. Pseudotumor cerebri is intracranial hypertension with similar symptoms, but treatment is with diuretics and weight loss. A berry aneurysm is an out-pouching of a cerebral blood vessel wall. A typical symptom is described as "the worst headache of my life" by patients. Some symptoms are similar to a temporal arteritis, but treatment is surgical. Angioedema of the oropharynx is usually due to an allergic reaction to numerous agents. Common symptoms are difficulty breathing and throat fullness as a result of airway occlusion. Treatment is the removal of the offending agent and securing the airway (if indicated).

A 50-year-old man with a past medical history of hypertension, asthma, hyperthyroidism, tachyarrhythmia, and aspirin allergy presents with chronic "nasal congestion," nasopharyngeal itching, sneezing, and diminished sense of smell. His physical examination reveals normal vital signs, no lymphadenopathy, and normal eye, mouth, throat, and pulmonary exams. His nasal exam was noteworthy for reduced patency, diminished sense of smell, and for the findings on internal speculum exam in the image below. Question Ico-delete Highlights What is the most effective pharmacological treatment of this patient's condition?

Correct answer: Beclomethasone (Beconase) nasal spray Explanation This patient is demonstrating signs and symptoms consistent with a nasal polyp and contributory allergic rhinitis. They present as yellowish boggy masses of hypertrophic mucosa and are associated with long-standing allergic rhinitis. Intranasal corticosteroids are the mainstay of treatment of allergic rhinitis. Intranasal corticosteroid sprays have revolutionized the treatment of allergic rhinitis. Evidence-based literature reviews show that these are more effective (and frequently less expensive) than nonsedating antihistamines. Patients should be reminded that there may be a delay in onset of relief of 2 or more weeks. Corticosteroid sprays may also shrink hypertrophic nasal mucosa and nasal polyps, thereby providing an improved nasal airway and osteomeatal complex drainage. Because of this effect, intranasal corticosteroids are critical in treating allergy in patients prone to recurrent acute bacterial rhinosinusitis or chronic rhinosinusitis. Intranasal anticholinergic agents, such as ipratropium bromide 0.03% or 0.06% sprays (42-84 mcg per nostril 3 times daily), may be helpful adjuncts when rhinorrhea is a major symptom. Ipratropium nasal sprays are not as effective as intranasal corticosteroids for treating allergic rhinitis but are useful for treating vasomotor rhinitis. In general, first- and second-generation antihistamines have been shown to be effective at relieving the histamine-mediated symptoms associated with allergic rhinitis (e.g., sneezing, pruritus, rhinorrhea, ocular symptoms), but are less effective than intranasal corticosteroids at treating nasal congestion. Antihistamines offer temporary, but immediate, control of many of the most troubling symptoms of allergic rhinitis. Over-the-counter antihistamines include nonsedating loratadine (10 mg orally once daily), fexofenadine (60 mg twice daily or 120 mg once daily), and minimally sedating cetirizine (10 mg orally once daily). Clindamycin is an antibiotic and not appropriate for non-infections conditions. Oral and topical decongestants (such as Sudafed) improve the nasal congestion associated with allergic rhinitis by acting on adrenergic receptors, which causes vasoconstriction in the nasal mucosa, resulting in decreased inflammation. The abuse potential for pseudoephedrine should be weighed against its benefits. Common adverse effects that occur with the use of intranasal decongestants are sneezing and nasal dryness. Duration of use for more than 3 to 5 days is not usually recommended, because patients may develop rhinitis medicamentosa or have rebound or recurring congestion. Because oral decongestants may cause headache, elevated blood pressure, tremor, urinary retention, dizziness, tachycardia, and insomnia, patients with underlying cardiovascular conditions, glaucoma, or hyperthyroidism should only use these medications with close monitoring.

Case A 61-year-old woman presents a 1-week history of intermittent episodes of feeling like she was spinning. She states the episodes are brief, but they occur 2-3 times per day. It is worse when she turns to her right side while lying in bed. Even when she is not dizzy, she feels off balance. She denies tinnitus, decreased hearing, fever, syncope, nausea, vomiting, diplopia, or any other related symptoms. During the Dix-Hallpike maneuver, the patient exhibits nystagmus, with her eyes beating upward and torsionally when the right ear is turned downward. The nystagmus diminished with each time the maneuver was performed. Question Based on the above description, what is the most likely diagnosis?

Correct answer: Benign paroxysmal positional vertigo Explanation Benign paroxysmal positional vertigo (BPPV) episodes are brief in nature and occur with changes in position. People often feel off balance even when an episode is not occurring. BPPV does not typically cause hearing loss. The Dix-Hallpike maneuver elicits an episode of vertigo, with nystagmus being noted during the exam. The nystagmus diminishes with each maneuver due to fatiguability. Labyrinthitis is usually caused by an infection, which the patient did not have. Labyrinthitis often includes hearing loss and or tinnitus, with episodes often lasting days or weeks. Vertigo with Meniere's disease usually lasts from 20 minutes to 24 hours. Sensorineural hearing loss and tinnitus are also common features. Typical onset is usually age 20-40. Most people with vestibular schwannoma this have hearing loss and or tinnitus. True spinning vertigo is uncommon with this disorder. People do have unsteadiness with walking. Some other symptoms can include paresthesia, hypesthesia, facial paresis, and taste disturbances. Symptoms are slow onset. Brainstem infarction patients present with sudden onset of symptoms, and the symptoms persist for days to weeks. Nystagmus has central characteristics, as opposed to the above patient, who showed peripheral characteristics. This would also have associated neurological signs and symptoms.

A 65-year-old Caucasian man presents with a 3-day history of severe dizziness. The symptoms are exacerbated by turning his head and relieved by lying still. He reports nausea and vomiting for the first 2 days of his illness but successfully eats breakfast on the day he is seen in the clinic. He denies hearing loss and tinnitus. His past medical and surgical histories are unremarkable. He has no previous exposure to ototoxic drugs and denies further neurologic symptoms. The otologic examination is without abnormality. Weber testing with a 512 Hz tuning fork is to midline. Romberg and Fukuda testing indicate right-sided pathology. Other than a crisp left-beating nystagmus, cranial nerve examination is normal. Vertigo is experienced after the Dix-Hallpike maneuver. Nystagmus is observed after a few seconds of lying down during the maneuver. Question Ico-delete Highlights What is the most likely diagnosis?

Correct answer: Benign positional vertigo Explanation Benign positional vertigo (BPV) is the most likely cause of this patient's vertigo. BPV is not typically associated with hearing loss. Patients have brief episodes of vertigo with positional changes, typically when turning over in bed. This patient had a positive response to the Dix-Hallpike maneuver, which confirms the diagnosis. BPV is due to deposition of calcium debris in the semicircular canals. Medications such as diazepam or meclizine as well as Canalith Repositioning Procedure (Epley maneuver) are used to treat the condition. The latter is a series of head rotational positions intended to relocate free-floating particles in the semicircular canals. An acoustic neuroma (sometimes termed a neurilemmoma or schwannoma) is a benign (non-cancerous) tissue growth that arises on the eighth cranial nerve leading from the brain to the inner ear. Acoustic neuromas usually grow slowly over a period of years. They expand in size at their site of origin and, when large, can displace normal brain tissue. The brain is not invaded by the tumor, but the tumor pushes the brain as it enlarges. The slowly enlarging tumor protrudes from the internal auditory canal into an area behind the temporal bone called the cerebellopontine angle. Since the balance portion of the eighth nerve is where the tumor arises, unsteadiness and balance problems may occur during the growth of the neuroma. The most common presentation is unilateral hearing loss. The presence of episodes of vertigo along with fluctuating hearing loss is consistent with the diagnosis of Meniere's disease, which is also characterized by tinnitus or a ringing sound in the ear. Some patients also experience a pressure sensation in the ear. Episodes occur at regular intervals for years and may also be marked by periods of remission. The cause is an increase in the volume of endolymph. Vestibular neuronitis presents as acute onset of vertigo, nausea, and vomiting lasting for several days. As this condition is not clearly inflammatory in nature, neurologists often refer to it as vestibular neuropathy. The etiology is unknown, but it appears to be a sudden disruption of afferent neuronal input from the left and right inner ears. This imbalance in vestibular neurologic input to the central nervous system causes symptoms of vertigo. Besides the unilateral caloric weakness, electronystagmography reveals a directional preponderance and beating nystagmus away from the affected side. Workup of patients with vestibular neuronitis begins with a thorough history and physical examination. Audiometry and vestibular testing are the cornerstones of diagnosis, with imaging and laboratory studies being guided by findings on examination. Bedside examination includes a doll's eye test, head-shaking nystagmus, dynamic visual acuity, caloric testing, rotational testing, past pointing, Romberg and Fukuda tests, and tandem walking. Patients with this condition do not generally have auditory symptoms. Dix-Hallpike maneuver is negative. Labyrinthitis is an inflammation or dysfunction of the vestibular labyrinth, which is a system of intercommunicating cavities and canals in the inner ear. The syndrome is defined by the acute onset of vertigo that is commonly associated with head or body movement. Nausea, vomiting, and malaise often accompany the vertigo. The pathophysiology of this syndrome is not completely understood, but a dysfunction of the vestibular apparatus is clearly present when labyrinthitis occurs. Many cases of labyrinthitis are associated with systemic or viral-like illnesses. Suppurative or bacterial labyrinthitis is rare, but it should be considered in patients with acute or chronic otitis media. Cases are reported in association with meningitis, but the presentation of meningitis often overwhelms the vestibular symptoms. Dix-Hallpike manuever is negative.

Case Ico-delete Highlights A 78-year-old woman presents with behavioral changes over the past few months. Her daughter feels that her mother is depressed, as she talks less frequently to her family members. She was a regular at various social gatherings, but the patient has been avoiding them lately due to some difficulty in communication. She has been spending more time painting portraits and has been watching television at a higher volume than usual. She has no suicidal thoughts. She is well oriented to time, place, and person; her mood today seems good. Ear examination and tests indicate sensorineural hearing loss. The rest of her physical examination is within normal limits. Question Ico-delete Highlights What is the best initial step in treating her condition?

Correct answer: Bilateral hearing aids Explanation The correct answer is bilateral hearing aids. The patient is suffering from presbycusis. She avoids social gathering, as patients with her condition have difficulty hearing, particularly in noisy environments2. It does affect self-esteem; therefore, it can be confused with depression. This patient avoids social gatherings, but she has not lost interest in other activities (like painting portraits), which helps rule out depression. Presbycusis is a multifactorial sensorineural loss initially affecting high frequencies and becoming progressively worse over decades1. Presbycusis is the most common form of hearing loss affecting older adults (older than 50 years)1. Ear examination, including pinnae, canals, and tympanic membranes, appear normal. Audiometery reveals bilateral symmetric moderate to severe sensorineural hearing loss, more pronounced in the higher frequencies1. Hearing aids are used to amplify sounds, though they cannot fully restore the quality of hearing2. Cochlear implants are a choice when hearing aids prove inadequate2. Psychotherapy, supportive counseling, and antidepressant medications are used for patients diagnosed with depression.

Case A 74-year-old man presents with a 1 ½-hour history of severe pain and blurred vision in his left eye. Upon examination, his left eye is erythematous with a steamy cornea and a nonreactive, dilated pupil. An ophthalmologic consult is ordered, and tonometry is completed, revealing an elevated intraocular pressure and a confirmed diagnosis of acute angle-closure glaucoma. Question Ico-delete Highlights What will be the definitive treatment for this patient?

Correct answer: Bilateral laser peripheral iridotomy Explanation The correct answer is bilateral laser peripheral iridotomy. This is a procedure during which a puncture-like opening is made near the base of the iris in order to decrease intraocular pressure in patients with angle-closure glaucoma. While there are various medications used to treat acute episodes, this procedure will correct the disorder definitively, whereas the medications are temporary treatment. Patients with narrow anterior chambers are at risk for angle-closure glaucoma. If this occurs unilaterally, they are even more at risk for acute episodes in the other eye. For this reason, the procedure is typically performed bilaterally. Left laser peripheral iridotomy is not the correct answer. While there are various medications used to treat acute episodes of angle-closure glaucoma, this procedure will correct the disorder definitively, whereas the medications are temporary treatment. Patients with narrow anterior chambers are at risk for angle-closure glaucoma. Narrow anterior chambers always occur bilaterally. If acute angle-closure glaucoma occurs unilaterally, they are even more at risk for acute episodes in the other eye. For this reason, the procedure is typically performed bilaterally as opposed to being done in JUST the affected eye. IV acetazolamide is not the correct answer. This medication is given in episodes of acute angle-closure glaucoma in order to decrease the intraocular pressure. It is typically given in a single 500mg IV dose followed by 250mg orally 4 times daily. This is effective to control the acute episode, but will not treat the disorder definitively, as the patient's underlying issue is narrow anterior chambers. Oral glycerol is not the correct answer. This medication is an osmotic diuretic that can be given 1-2 g/kg in order to decrease a patient's intraocular pressure during an acute episode of angle-closure glaucoma. This is effective to control the acute episode, but will not treat the disorder definitively, as the patient's underlying issue is narrow anterior chambers. Topical timolol 0.25% is not the correct answer. This medication is a topical β-adrenergic blocking agent used twice daily chronically in patients who have chronic glaucoma. The disorder does not require the acute lowering of intraocular pressure such as angle-closure glaucoma. Topical timolol would not be effective in lowering intraocular pressure in patients with angle-closure glaucoma.

Case A 52-year-old man presents with a burning sensation of both eyes. He denies recent trauma and no contact with individuals who have similar symptoms. On examination, his eyelid margins are red and inflamed. His eyelashes are greasy and adherent with a surrounding dandruff-like scale. Conjunctiva are clear. Question What is the most likely diagnosis?

Correct answer: Blepharitis Explanation Blepharitis is a chronic inflammation of the lid margins. Causes include infection, seborrhea, and meibomian gland dysfunction. Blepharitis characteristically causes red eyelid margins with adherent eyelashes and scale-like deposits. It is treated with eyelid scrubs with diluted baby shampoo. Antibiotics may be used if infection is suspected. Hordeolum presents as a small inflammatory nodule in a gland on the upper or lower eyelid margin. Chalazion presents as a painless lesion of the palpebral margin. Conjunctivitis presents in several different ways depending on the etiology. However, the conjunctiva would be erythematous; this patient's conjunctiva are clear Pinguecula presents as an elevated yellow fleshy mass on the sclera adjacent to the conjunctiva.

Case A client presents with a 1-year history of flaking and scaling around her lashes, along with itching and a burning sensation. She has also noted her lid margins are red, and some of her lashes are missing. Her history is significant for seborrheic dermatitis of the scalp, eyebrows, and external ears, and diabetes. Question What is the most likely diagnosis?

Correct answer: Blepharitis Explanation Blepharitis may be seborrheic or ulcerative. Seborrheic (non-ulcerative) blepharitis is commonly associated with seborrhea of the face, eyebrows, external ears, and scalp. Inflammation of the eyelid margins occurs, with redness, thickening, and often the formation of scales and crusts or shallow marginal ulcers. Ulcerative blepharitis is caused by bacterial infection (usually staphylococcal) of the lash follicles and the meibomian glands. Removal of crusts, topical antibiotics, and /or oral antibiotics remains the mainstay of treatment. Chalazion is a mildly painful swelling of the eyelid margin due to granulomatous inflammation, and usually resolves spontaneously with warm soaks and time. Multiple chalazion are seen in diabetes, alcoholism, and malnourishment. In extreme cases curettage may be required. Conjunctivitis can be caused by bacterial, viral, allergic, and irritant etiologies. Clients usually complain of red eyes and a sticky or watery discharge. The type of discharge helps to determine the etiology; it is watery in viral, and sticky green or yellow in bacterial conjunctivitis. Irritation is common, but severe pain and photophobia are not. Bacterial or viral conjunctivitis is usually self-limited, but it may be treated with a topical antibiotic, without steroids, such as sulfacetamide (10% 3-4 times/day). Topical aminoglycoside should be reserved for more refractory disease. Allergic conjunctivitis may be effectively treated with a new class of non-steroidal, topical, anti-inflammatory agents. Irritant conjunctivitis, including dry eyes, may be treated with topical, non-preserved lubricants. Ectropion is the outward turning of, usually, the lower lid occurring in older people. Surgery is indicated if ectropion causes excessive tearing, exposure, keratitis, or a cosmetic problem. Foreign body sensation is most commonly due to corneal or conjunctival foreign bodies. Other causes are disturbances of the corneal epithelium and rubbing of eyelashes against the cornea (trichiasis).

Question A 3-year-old boy presents after waking up with his eyelids glued together. His mother states that he has been rubbing his eyes constantly. The medical history is unremarkable, and the patient had not been sick before. Upon examination, redness of the lid margin, edema, conjunctival irritation, and loss of lashes can be seen. There are also some scales on the lid margin that can be removed easily. What is the most likely diagnosis?

Correct answer: Blepharitis Explanation The symptoms described above are typical for blepharitis, an inflammation of the lid margins. There are 2 types of blepharitis: Ulcerative blepharitis: it is caused by bacterial infection (mostly staphylococci) of follicles and meibomian glands. Adherent crusts develop that result in bleeding if removed. Pustules form in the lash follicles and turn into small ulcers. Repeated episodes can lead to permanent loss of lashes, scarring of the lids, and sometimes even corneal ulceration. Treatment consists of application of antibiotic ointment for 7 to 10 days. Seborrheic blepharitis: the cause is not known. Sometimes it is associated with seborrhea of scalp and face. The scales are easily removable. Treatment is usually to just keep the eyelids clean. If it does not clear, antibiotic ointment is needed. Hordeolum is a localized infection of the Zeis, Moll, or meibomian glands; it is usually caused by staphylococci. It can be associated with blepharitis or follow it. The internal hordeolum (affecting the meibomian glands) involve pain, redness, and localized edema with an elevated yellow area near the affected gland that turns into an abscess, which rarely ruptures spontaneously. The external hordeolum (affecting the glands of Zeis or Moll) starts with pain, redness, and tenderness, which turns into an induration with a yellowish spot in the center. The abscess soon ruptures and emits pus. Acute conjunctivitis is an inflammation of the conjunctiva that can be bacterial, viral, or allergic. The most common pathogens for bacterial infection in adults are Staphylococci, Streptococcus pneumoniae, and Haemophilus influenzae. In children, bacterial conjunctivitis is more common than viral conjunctivitis and is caused by Streptococcus pneumoniae, Haemophilus influenzae, and Moraxella catarrhalis. Symptoms are itching, irritation, foreign body sensation, tearing, and in bacterial infection, mucopurulent discharge. Small bumps with fibrovascular cores on the palpebral conjunctiva that look like a velvety surface are a typical papillary reaction. Bacterial conjunctivitis is usually self-limiting but can also cause corneal or systemic complications (like meningitis in primary meningococcal meningitis) and should therefore be treated with topical antibiotics (e.g., polymyxin-bacitracin, ciprofloxacin, or ofloxacin). Viral conjunctivitis is more common than bacterial conjunctivitis in adults; it is caused by herpes virus or during the course of systemic or cutaneous infections, such as rubella, measles, cytomegaly, syphilis, or systemic adenovirus infections. Occasionally permanent vision loss can occur, as well as cataract, microphthalmos, retinal involvement, interstitial keratitis, and optic neuritis. Trachoma is a chronic conjunctivitis caused by chlamydia trachomatis. It is transmitted via contact (hands, towels, etc.), and it is a major cause of blindness in developing countries due to poor hygiene and economic conditions. In the United States, only mild forms are typically seen, usually in immigrants from endemic areas. The disease presents as conjunctivitis with small lymphoid follicles. Cultures are often false negative; therefore, PCR to prove the presence of chlamydial DNA in ocular secretions should be performed. Treatment consists of topical or systemic tetracycline or erythromycin.

Case A 73-year-old man presents with a nosebleed that will not stop. The bleeding has been present for over 2 hours. The patient's nose began dripping blood at breakfast; there was no known trauma. He denies pain. He has tried applying nasal pressure and laying down to rest. Until the bleeding began, the patient had not been experiencing any nasal symptoms, such as congestion, impaired nasal patency, or rhinitis. Because he felt too dizzy to drive, his wife drove him to the emergency department. The patient has no diagnosed medical conditions and takes no medications. He denies prior episodes of severe nosebleeds, easy bruising, and any known bleeding disorder. The patient's vitals are shown in the table. Weight 148 lbs. Height 69" Pulse 120 Respiratory rate 18 Blood pressure 90/66 mm Hg Temperature 97.4 °F/36.3°C On physical exam, the man is holding a large blood-soaked towel to his nose, with continued brisk bleeding. He is otherwise in no apparent distress, but seems somewhat confused. On rhinoscopy, bleeding is observed from bilateral nares. No foreign body, mass, lesions or abrasions are visualized. The bleeding site cannot be identified. Question What test should be ordered next for this patient?

Correct answer: Blood type and Rh Explanation Based upon this patient's history, and physical, he most likely has a posterior epistaxis. Posterior bleeds are less common than anterior bleeds, but should be suspected when the bleeding is large in volume and unresponsive to nasal packing/tamponade. The cause of a posterior bleed is not readily evident in this patient and many are idiopathic. However, with this patient's confusion, tachycardia, tachypnea, hypotension and blood loss, the provider must recognize that this patient is presenting with shock (hypovolemic), and this needs to be addressed prior to the epistaxis. Of the choices listed, if this patient needs a transfusion, a blood type and Rh would be appropriate. If a patient presents with shock and obvious blood loss, control of bleeding, crossmatching of blood, and infusion of fluids and blood products are first-line actions. Coagulation studies are rarely helpful in in evaluation of epistaxis, unless the episodes are recurrent or there are other reasons to suspect a coagulopathy. Because the patient is 73 years of age, it would be unusual for him to present with a coagulation disorder for the first time in his life. Computed tomography (CT) of the sinuses and/or plain nasal X-ray could be useful in evaluating patients with epistaxis and a history of trauma. Imaging may show a fracture, or in rare cases, a tumor. However, this patient denies any trauma, and he needs vascular stabilization prior to imaging. Nasal endoscopy is accomplished using a flexible or rigid fiberoptic endoscope to view the nasal and sinus passages. It can be useful in locating the origin of the bleed in epistaxis. Once this patient is vascularly stable, endoscopy may be useful.

Case A 23-year-old woman presents to the emergency department due to double vision; pain upon moving the right eye; and numbness of the cheek, nose, and right side of the eye. She was playing baseball with her team and the ball hit her in the right eye. Upon physical exam, swelling, tenderness, numbness of the nose, and epistaxis are present Question What is the most likely diagnosis?

Correct answer: Blowout fracture Explanation The clinical picture is suggestive of blowout fracture, commonly known as an orbital floor fracture. The patient presents with a history of being hit in the right eye and presents with double vision, and numbness of the cheek and right side of eye. Additionally, X-ray would confirm a fracture. Blepharitis is not the correct answer. It is when oil glands become clogged or irritated. Additionally, a patient would present with itchy eyelids, watery eyes, a feeling that something is in their eye, or crust on the eyelashes. These symptoms are not present in this patient. Conjunctivitis is not the correct answer because conjunctivitis presents with burning, itching, irritation, and discharge, all of which is not present in this patient. Corneal abrasion is not the correct answer because it does not present with swelling, double vision, and numbness of the cheek. Instead, it is diagnosed with fluorescein stain and described as very painful, due to the scratch on the surface of the cornea. Stye is not the correct answer, as it is when bacteria gets into the oil gland in the eye lids. Additionally, it creates a red bump that is located closer to the eyelashes. It can make your eye feel watery, or as though something is in your eye. All of these symptoms are not present in the case.

Case A 66-year-old man presents with sudden onset of brief episodes of blindness in his right eye, with complete recovery of vision within 24 hours. The event is described as a shade coming down across his field of vision, and it is not painful. Question What is the most common underlying condition leading to the event described above?

Correct answer: Carotid stenosis Explanation The diagnosis for this case is amaurosis fugax. The most common cause of amaurosis fugax is ipsilateral carotid stenosis, leading to carotid emboli that lodge in the retinal arteries, causing temporary blindness until the emboli dissolves. Screening is performed through carotid Doppler ultrasonography, CT, or MR angiography. Treatment includes low-dose aspirin or other anti-platelet therapy, along with treatment of underlying cause. Migraine headaches, sickle cell disease, and acute angle-closure glaucoma are all less common causes of amaurosis fugax. Lower extremity deep venous thrombosis is not a likely source for emboli causing amaurosis fugax.

Case A 30-year-old African-American man is admitted to the hospital to undergo stapedectomy for the treatment of otosclerosis. He had been experiencing increased hearing loss in the right ear over the past few years. His mother had suffered from the same condition when she was in her 40's and had been successfully operated upon. You do an assessment using the Weber and Rinne tests. Question What do you expect to find?

Correct answer: Bone conduction of the affected side is greater than air conduction Explanation The correct response is bone conduction of the affected side is greater than air conduction. Otosclerosis is a pathological condition of the middle-ear in which there is a formation of spongy bone near the footplate of the stapes. As it advances, it causes progressive fixation of the stapes footplate. Therefore, the sound transmission by air conduction from the stapes via the oval window to the perilymph of the inner ear is reduced. Weber tuning fork test is performed by placing the stem of a vibrating tuning fork on the midline of the head and having the patient indicate in which ear the tone is heard. The fork stimulates both inner ears equally. A patient with a unilateral conductive hearing loss hears the tone louder in the affected ear and a patient with a sensorineural loss hears the tone louder in the unaffected ear. In otosclerosis the sound lateralizes to the affected ear. The Rinne tuning fork test compares air to bone conduction. The stem of the vibrating tuning fork is placed on the mastoid process first and then the tines of the tuning fork are held in front of the ear and the patient is asked which stimulus is perceived better. In a healthy individual, the tone is heard longer and louder by air conduction, whereas with a conductive hearing loss, it is the other way around. Therefore, in otosclerosis the bone conduction is better than air conduction (BC>AC). With a sensorineural hearing loss, both receptions are reduced, but in the same ratio.

Case A 36-year-old man presents with nasal stuffiness, headache, fatigue, facial pain, and chronic post-nasal drip. He has had similar episodes in the past, occurring 2-3 times a year for the last several years. He has been diagnosed with acute sinusitis and antibiotics have been prescribed, providing him with relief for a brief period. This time, however, his symptoms have bothered him on and off for the last 3 months. He was given a 14-day course of antibiotics, but he experienced only partial relief. He is tired of the recurrent episodes, and he wants a cure. On exam, he is afebrile, nasal mucosa is inflamed, and there is mucopurulent secretion in the nasal cavity. The right maxillary sinus is tender on palpation. Lungs are clear. Question What is the best next step in the management of this patient?

Correct answer: CT scan of the sinuses Explanation This patient is suffering from chronic sinusitis. Repeated regimens of different antibiotics have not provided him relief, and now he fits the criteria for chronic sinusitis, including 12 weeks of symptoms. Chronic sinusitis is most commonly caused by Streptococcus pneumoniae, Haemophilus influenzae, and Moraxella catarrhalis. Together, these 3 bacteria account for 70% cases. A limited CT scan of the sinuses defines the location and extent of disease and helps in deciding further management. It is quick, low cost, and sensitive. CT scanning also helps in delineating anastomotic blockage of the osteomeatal complex, so there is a role for it in cases of endoscopic surgery. A 3-week course of antibiotics may relieve symptoms briefly, but it is unlikely to cure him, especially since he already has had several courses. Amoxicillin-clavulanate or cefuroxime are traditionally used for 3 weeks. In intractable cases, a 6-week course may also be given. Clarithromycin and clindamycin are used for patients who are allergic to penicillin. Quinolones are only used if cultures show gram-negative bacteria. Plain X-rays are no longer recommended; they are not sensitive enough in the visualization of the sinuses, and they often miss findings. Nasal cultures can be contaminated with colonized organisms in the nose, such as Staphylococcus aureus, and do not correlate well with culture obtained from the sinuses. Endoscopically-guided cultures of secretions in the middle meatus or within a sinus are usually not done in clinical practice, even though occasionally it may provide the exact causative pathogen. MRI of the sinuses tells us more about the soft tissue pathology, but bony structures cannot be studied in detail. MRI is done if malignancy is suspected or if there are signs and symptoms of possible intracranial extension.

Case A 30-year-old man presents with a recurrent history of mouth and jaw pain. He states that he gets swelling on the right side of his jaw that begins while eating and subsides within 2 hours after eating. On physical exam, there is no edema noted, but a palpable mass is noted on the right side of the jaw, near the TMJ. No other significant findings are noted. Question What is the most likely cause?

Correct answer: Calculus formation in the Stensen's duct Explanation The clinical picture is suggestive of sialolithiasis, or a calculus formation within the Stensen's duct of the parotid gland. A patient may note swelling and pain while eating or after eating. A palpable lump or visible swelling in the area of the gland is often noted. Sialadenitis is an acute bacterial infection of the parotid or submandibular gland. A patient typically presents with acute swelling, pain and swelling with meals, erythema, and tenderness of the duct of the affected gland. Pus can often be massaged from the duct, which is not seen in this patient. Most parotid gland cancers present with an asymptomatic mass within the gland. Paramyxoviral infection (or mumps) is an acute infection of the parotid gland; most patients are children. Parotid tenderness and persistent facial swelling are the most common presenting symptoms, but they are not present in this patient. In cases of a calculus formation of the Wharton's duct, a mass would be palpable on the anterior floor of the mouth.

Case A 40-year-old man presents with a sore throat. The patient is a known IV drug user and is HIV positive. He claims that there is no fever or headache present. Lungs are clear. The throat is examined, and it is remarkable for the presence of a heavy patchy white exudate spread throughout the throat and tongue. A scraping of the exudate is examined directly under the microscope and a diagnosis is made. Question What pathogen is the cause of this patient's infection?

Correct answer: Candida albicans Explanation Streptococcus pyogenes is a gram-positive coccus; it is catalase negative, beta hemolytic on blood agar, and appears as chains on Gram stain. Definitive identification to distinguish it from other beta-hemolytic streptococci is the detection of its specific "A" antigen by latex agglutination techniques. It is associated with streptococcal pharyngitis, scarlet fever, streptococcal pyoderma, necrotizing fasciitis, and streptococcal toxic shock syndrome. Bacteremia is uncommon. They are universally sensitive to penicillin. Streptococcus pneumoniae is a gram-positive lancet-shaped coccus that is catalase negative and occurs in pairs. It is a common cause of otitis media in children. It is also a major cause of meningitis in elderly people and especially those that have underlying conditions, are malnourished, or have alcoholism. The organism is alpha hemolytic on blood agar. Staphylococcus aureus is a gram-positive coccus, catalase positive, and coagulase positive, predominantly beta hemolytic on blood agar, appearing in characteristic grape clusters on Gram stain. Staphylococcus aureus can cause a variety of infections. In children with reactive tonsils, this can be a source of infection, leading to severe tonsillitis. A semisynthetic penicillin is the treatment of choice. Haemophilus influenza is a gram-negative coccobacillus. It is also a major cause of meningitis. It occurs mostly in young infants and children where it can also cause a severe epiglottitis that can necessitate intubation. When it occurs in adults, it is usually due to an underlying condition, such as paranasal sinusitis, remote head trauma, or otitis. The organism will not grow on blood agar and requires the presence of growth factors (hemin and NAD) for growth. Candida albicans is yeast. Yeast appears on Gram stain as large gram-positive organisms that are approximately 3-5 times larger than gram-positive cocci. They are aerobic and generally grow well on most nonselective agar media. The organism is a major cause of throat infections in the immunocompromised, such as patients with HIV. It is called "thrush" when causing an infection in the throat.

Question Ico-delete Highlights A patient presents due to her tongue having "a thick white coating"; she adds that it is "also red and irritated." She is able to "scrape the white stuff off" some areas of her tongue, which she reports have become raw and more erythematous. What is the most likely diagnosis?

Correct answer: Candidiasis Explanation Candidiasis is an infection that may cause the tongue to have a white coating, which can be scraped, and a sample can be analyzed for the presence of Candida. Hairy leukoplakia may be seen in people infected with HIV and AIDS. It is characterized by raised areas that are whitish-tan in color and have a feathery appearance. It is different from candidiasis of the tongue in that hairy leukoplakia cannot be scraped off. A "hairy tongue" is not actually due to hair growth on the tongue; it consists of elongated papillae that have the appearance of grayish-black hair to the naked eye. This condition may be caused by antibiotic use, or there may not be any reason. Atrophic glossitis (or smooth tongue) presents as having a smooth surface due to papillae loss. The loss may indicate deficiency in riboflavin, niacin, folic acid, vitamin B12, pyridoxine, or iron. A geographic tongue is a benign condition with unknown cause; it is characterized by a map-like pattern of smooth, red areas that do not have papillae as well as rough areas that still have papillae.

Case An 84-year-old man presents to the emergency room with 2.5-hour history of painless, progressive vision loss in his right eye, which began while he was reading. He denies any other symptoms. Past medical history is positive for hypertension and a cardiac dysrhythmia. Physical exam findings include a BP of 180/110 mm Hg; other vital signs are normal. The right eye non-reactive to light. Funduscopic exam reveals a pale retina with a red spot. The rest of the physical exam is normal. There is a "box-car" like segmentation of blood in the retinal veins. Question What is the most likely cause of vision loss in this patient?

Correct answer: Central retinal artery occlusion Explanation Central retinal artery occlusion (CRAO) interferes with the major supply of blood to the retina, causing vision loss. In 25% of individuals, the macula is supplied by the cilioretinal arteries, sparing some central vision in the event of CRAO. The occlusion is principally caused by thrombus, thromboemboli, cholesterol plaques, calcium, or vasospasm. The patient notes a sudden, painless, monocular vision loss; the physical examination reveals a problem in the visual afferent way, and the funduscopic examination shows the red spot, which is the pigment of the choroid showing through the macula. There is also a characteristic "box-car" like segmentation of blood in the veins. Treatment consists of decreasing intraocular pressure in order to increase the pressure gradient in the artery and force the embolus to dislodge, restoring vision. Application of digital pressure, carbonic anhydrase inhibitors, beta-blockers, and paracentesis of the anterior chamber are some of the methods used for this purpose. Retinal vein occlusion should be considered in the differential; it is characterized by preservation of some vision, and it rarely shows a red spot in the retina at the funduscopic examination. Retinal detachments often cause prodromal symptoms, such as flashing lights, floating 'spider webs', and the sensation of 'having a curtain drawn up or down' over the visual field. The funduscopic exam reveals an undulating, pale, detached retina. Acute angle-closure glaucoma is characterized by severe ocular pain and blurred vision, rather than loss of vision.

Case Ico-delete Highlights A 78-year-old Caucasian man presents with unilateral painless loss of vision in the right eye of 3 hours duration. Examination reveals an elderly man who is anxious but in no acute distress. Visual acuity is light perception only in the right eye and 20/30 in the left eye. Pupillary examination is significant for an afferent pupillary defect on the right side. Penlight examination of the eyes is otherwise unremarkable. Retinal examination of the right eye reveals a cherry-red spot. Retinal examination of the left eye is unremarkable. Question Ico-delete Highlights What disease process most likely accounts for the patient's presentation?

Correct answer: Central retinal artery occlusion Explanation This case represents the classic presentation of a central retinal artery occlusion. Namely, it is acute, unilateral, and painless; there is loss of vision as well as a cherry-red spot on fundus examination. Tay-Sachs disease is a lysosomal storage disease found predominantly in Ashkenazi Jews. Infants with this fatal neurodegenerative disease do have a cherry red spot on retinal examination. An adult-onset form of Tay-Sach's disease is rare but does exist. Onset of symptoms is in the 3rd or 4th decade of life; however, it is characterized by neurologic deterioration and cherry-red spots would be bilateral. Although this patient does have a cherry-red spot, there is nothing in the presentation to suggest Tay-Sachs disease. Open angle glaucoma is a chronic, slowly progressive condition that would not be expected to cause acute visual loss. The patient gives no history of trauma, and no evidence of trauma is seen on examination. A cataract, or opacification of the lens, would not be expected to cause acute visual loss.

Case Ico-delete Highlights An 18-year-old girl has had a small, slightly tender swelling in her left upper eyelid for 10 days. For the first day or two, it was red and a little painful. Now it is painless, although it has grown in size. There has not been any drainage, visual changes, or itching noted. She has been well otherwise. On exam, vitals are normal, extraocular muscles are intact, and pupils are equal and reactive to light. Her left upper eyelid has a 1.5 cm round, nontender, swollen mass that is mildly erythematous without any drainage. The underside of the lid is grayish-red. Question Ico-delete Highlights Based on these findings, what is the most likely diagnosis?

Correct answer: Chalazion Explanation A chalazion is an enlargement of a deep oil gland in the eyelid that results from an obstruction of the gland opening at the edge of the eyelid. Associated factors include immunodeficiency, leishmaniasis, seborrhea, high blood lipid concentrations, acne rosacea, tuberculosis, chronic blepharitis, carcinoma, and viral infections. Most will disappear without treatment after 1 to 3 months. If hot compresses are applied several times a day, it may disappear sooner. Persistent lesions that cause changes in vision should be examined by an ophthalmologist for possible drainage by incision and curettage or intralesional corticosteroid injection. Hordeolum, or stye, is an acute infection of one or more of the glands along the edge of the eyelid or under it. This is usually caused by a staphylococcal infection. An abscess may form and rupture. A stye will usually last 2 - 4 days and may form simultaneously with or as a result of blepharitis. It begins with redness, tenderness, and pain at the edge of the eyelid. A small round, tender swollen area often forms with a tiny, yellowish central spot. Blepharitis is an inflammation along the edges of the eyelids with possible thickening scales, crusts, shallow ulcers, or inflamed oil glands. This may be caused by staphylococcal infection of the eyelids or the ducts of deeper glands, seborrheic dermatitis of the face and scalp, or acne rosacea. The eyes and lids may itch and burn, or they may become watery and photosensitive. The eyelid may swell and become red; lashes may fall out. Sometimes small abscesses containing pus can develop in the sacs at the base of the eyelashes and form shallow ulcers known as ulcerative blepharitis. Crusts may form on the edges of the eyelid, and when removed, they may leave a bleeding surface. This tends to recur and resist treatment, but the crusts rarely damage the cornea or causes vision loss. Dacryostenosis results in a blockage of the flow of tears from the eye to the nose due to narrowing of the nasolacrimal ducts. This can result from inadequate development of any part of the ducts, chronic nasal infection, severe or recurring eye infections, or fractures of the nasal or facial bones. If dacryostenosis does not resolve, infection or dacryocystitis will result. Dacryocystitis is an infection of the lacrimal sac that results from a blockage of the nasolacrimal duct, which leads from the lacrimal sac into the nose. This may occur acutely or chronically. In acute infections, the area around the lacrimal sac becomes painful, red, and swollen; this causes the eye to become red and watery as well as ooze a purulent discharge. Pressure to the lacrimal sac may push pus through the opening near the nose. Fever is common. Infection can sometimes cause fluid to be retained in the lacrimal sac, which becomes a mucocele that can become an abscess.

Case A 24-year-old man presents with a painless, localized swelling of his left lower eyelid; it has developed over a period of weeks. He is seeking medical attention because it is now producing a foreign body sensation in his left eye; it is also hindering his path of vision. On physical examination, his visual acuity is normal; there is no evidence of injection or discharge. You palpate, and you observe a nontender, localized nodule on the lower eyelid. Question What is the most likely diagnosis?

Correct answer: Chalazion Explanation Chalazion is the correct response. Chalazion is a granulomatous inflammation of the meibomian gland, and it has the typical presentation of being a hard, nontender swelling of either the upper or lower lid. It is sometimes accompanied by redness and swelling of the adjacent conjunctiva. Blepharitis is the incorrect choice; typically, blepharitis is a bilateral condition of general inflammation of the eyelid skin, eyelashes, and associated glands. Characteristics of blepharitis include red-rimmed eyes as well as scales on the eyelashes. Tears may even have a greasy distinction to them. Ectropion is also an incorrect choice. Ectropion is the outward turning of the lower lid. This is not what was described in this case. Keratitis is also incorrect; it produces a painful eye, a hazy-appearing cornea as well as evidence of an ulcer or even an abscess in this area; hypopyon is also possible in patients who have keratitis. Malignant melanoma can have a presentation in the ocular area that is somewhat like the one described in this case; however, it is not a common presentation.

Case A 42-year-old man presents with a firm, painless bump on his left upper eyelid. On examination, you note a 5 mm mass within the tarsus of the left eye. The skin is freely movable over the mass. The remainder of the eye exam is unremarkable. Question What is most likely diagnosis?

Correct answer: Chalazion Explanation The clinical picture is an example of a chalazion. A chalazion is a painless chronic mass in the eyelid. Chalazions differ from hordeolums in that they (chalazions) are usually painless, apart from the tenderness caused when they swell up, and are generally larger in size than styes. Ectropion is when the eyelid sags outwardly and the lid does not close well. Hordeolum are acute, red, and painful. Pterygium involves the sclera.

A 5-year-old girl was brought to the emergency room in acute respiratory distress. The patient had complained of a sore throat the previous day; it was accompanied by fever and dyspnea. Physically, the patient was irritable and appeared to have great difficulty trying to breathe. She was sitting with her neck extended and chin protruding as if she had something blocking her throat. She was uncontrollably drooling saliva from the mouth. Her body temperature was 38.7°C; heart rate was 130/min. An examination of her throat revealed a bright red epiglottis that was extremely swollen and obstructing the pharynx. A diagnosis was made based on the clinical presentation. The eventual isolation of a gram-negative coccobacillus from the throat confirmed the diagnosis. Question What infection did this patient have?

Correct answer: Haemophilus influenzae Explanation Haemophilus influenzae can cause an acute respiratory obstruction of the upper airways by producing a cellulitis of the supraglottic tissues. Swelling of the epiglottis and aryepiglottic folds with complete obliteration of the vallecular and pyriform sinuses is typical. Children ages 2-7 are prone to this epiglottitis. They will develop acute symptoms quickly, with the initial symptoms being a sore throat, fever, and dyspnea progressing rapidly to dysphagia, pooling of oral secretions, and drooling of saliva from the mouth. The child will many times present as restless and anxious while adopting a sitting position where the neck is extended and chin is protruding to reduce airway obstruction. Characteristically, the epiglottis is red and swollen, and it bears a striking resemblance to a bright red cherry obstructing the pharynx at the base of the tongue. Care should be taken when examining the larynx so as not to induce fatal respiratory obstruction. Candida albicans (a yeast) is the causative agent of thrush, an infection of the throat and tongue. Thrush is characterized by creamy white curd-like patches on the tongue and on other oral mucosal surfaces. These patches are actually a pseudomembrane consisting of Candida, necrotic tissue, keratin, bacteria, leukocytes, epithelial cells, and food debris. A diagnosis can be made by simply scraping the surface of the lesion and examining for the presence of masses of hyphae, pseudohyphae, and yeast forms. Thrush has become a more common occurrence in children due to the use of inhaled steroids for the treatment of asthma. Bordetella pertussis (a gram-negative bacillus) produces an infection that is symptomatic after an incubation period of less than 1 week to more than 3 weeks. This initial symptomatic period is called the catarrhal phase; clinically, patients will exhibit mild conjunctival infection, malaise, lacrimation, rhinorrhea, and a low-grade fever. These symptoms are similar to many upper respiratory or systemic diseases. Within a few days to a week, this phase eventually leads to a dry, nonproductive cough. This cough evolves into a paroxysm that consists of a series of short expiration bursts followed by an inspiratory gasp, which can result in the typical "whoop" of whooping cough. The disease will typically produce a leuko/lymphocytosis with a total white blood cell count that can exceed 50,000 cells/mm3. Pulmonary consolidation can be seen in more than 20% of chest radiographs. Coughs, though still paroxysmal, do not always produce the characteristic "whoop" in all patients. Streptococcus pyogenes is the causative agent of Group A streptococcal pharyngitis, one of the most common bacterial infections of childhood. Streptococcal sore throat primarily occurs in children ages 5-15, and peak incidence occurs in the first few years of school. Incubation periods are between 2-4 days followed by a sudden onset of a sore throat, malaise, headache, and fever. Some patients may complain of nausea, abdominal pain, and vomiting. There is redness, lymphoid hyperplasia, and edema of the posterior portion of the pharynx. Tonsils may be enlarged and have grayish white exudate patches on the surface. Tender lymph nodes can be noted at the angles of the mandibles. Temperatures of 101°F or greater are seen and white blood cells counts can exceed 12,000/mm3 with a pronounced left shift. Rheumatic fever can be a complication. Corynebacterium diphtheriae (a gram-positive bacillus) is the causative agent of diphtheria, a painful tonsillitis and/or pharyngitis with an associated pseudomembrane production. A presumptive diagnosis can be made by the clinical presentation of a patient with tonsillitis and/or pharyngitis with associated membrane, adenopathy and cervical swelling (especially if associated with membranous pharyngitis and signs of systemic toxicity), hoarseness and stridor, palatal paralysis, serosanguineous nasal discharge with associated mucosal membrane, and a temperature elevation rarely in excess of 103°F. Because of immunization, the disease is not commonly encountered.

Question A client comes into the clinic with a complaint of a hard, non-tender swelling on the upper lid of her left eye. The conjunctiva in the region of the lesion is red and elevated. What is the most likely diagnosis?

Correct answer: Chalazion Explanation Chalazion is a mildly painful swelling of the eyelid margin, due to granulomatous inflammation, and usually resolves spontaneously with warm soaks and time. Multiple chalazion are seen in diabetes, alcoholism, and malnourishment. In extreme cases curettage may be required. Blepharitis may be seborrheic or ulcerative. Seborrheic (non-ulcerative) blepharitis is commonly associated with seborrhea of the face, eyebrows, external ears, and scalp. Inflammation of the eyelid margins occurs, with redness, thickening, and often the formation of scales and crusts, or shallow marginal ulcers. Ulcerative blepharitis is caused by bacterial infection (usually staphylococcal) of the lash follicles, and the meibomian glands. Conjunctivitis can be caused by bacterial, viral, allergic, and irritant etiologies. The clients usually complain of red eyes, and a sticky or watery discharge. The type of discharge determines the etiology-watery in viral, and sticky green or yellow in bacterial conjunctivitis. Irritation is common, but severe pain and photophobia are not. Bacterial or viral conjunctivitis is usually self-limiting, but it may be treated with a topical antibiotic, without steroids, such as sulfacetamide (10% 3-4 times/day). Topical aminoglycoside should be reserved for more refractory disease. Allergic conjunctivitis may be effectively treated with a new class of non-steroidal, topical, anti-inflammatory agents. Irritant conjunctivitis, including dry eyes, may be treated with topical, non-preserved lubricants. Ectropion is the outward turning of, usually, the lower lid occurring in older people. Surgery is indicated if ectropion causes excessive tearing, exposure, keratitis, or a cosmetic problem. Foreign Body sensation is most commonly due to corneal or conjunctival foreign bodies. Other causes are disturbances of the corneal epithelium, and rubbing of eyelashes against the cornea (trichiasis)

Case A 10-day-old male infant presents with bilateral conjunctivitis with moderate white discharge. He is acting normally, has no fever, and is feeding well. He was born full term without any complications. His mother had minimal prenatal care. He has been gaining weight well. On exam, he is alert and active. Culture with immunofluorescence reveals inclusion bodies. Question What is the most likely cause of this infant's conjunctivitis?

Correct answer: Chlamydia trachomatis Explanation Chlamydia and gonorrhea are the most common causes of conjunctivitis in the neonate. Given the timing and presentation, Chlamydia is the most likely cause of this infant's conjunctivitis. A characteristic finding of chlamydial infection is the presence of inclusion bodies in the epithelial cells of a conjunctival smear. The usual incubation period for C. trachomatis is 5-14 days and 2-5 days for N. gonorrhoeae. Gonococcal conjunctivitis tends to produce a more purulent discharge compared to C. trachomatis. Herpes conjunctivitis is sometimes contracted when infants are born to mothers with herpes lesions in the genital region, which is not the scenario in this case. Conjunctivitis due to silver nitrate drops usually occurs 6-12 hours after birth. This is not the presentation of this infant. Coxsackievirus is not a common cause of conjunctivitis in neonates.

Case A 23-year-old woman presents with a 2-week history of left eye irritation. She insists that there is a foreign object lodged under her eyelid. There are no other symptoms. Physical examination is remarkable for unilateral conjunctivitis, which appears acute and follicular in presentation, and mucoid discharge. There is no detectable foreign object present under the eyelid. 4 weeks ago, the patient presented with abdominal discomfort and vaginal bleeding. A cervical culture was obtained, and a blood specimen was obtained for a CBC. No other remarkable findings were noted on her last visit. Cervical cultures came back negative. CBC and differential were within normal limits. She had previously denied any sexual activity. When questioned again, she reveals that she has been sexually active with 2 male sexual partners in the past 4 months. The laboratory on a conjunctival specimen performs a DFA test, and the result is as follows (see image). A cervical specimen is also collected for DFA, with similar results. All routine cultures for both sites are negative for any significant organisms or pathogens, and Gram stain results are "NOS". Question The most likely diagnosis is inclusion conjunctivitis concurrent with cervicitis due to what disease?

Correct answer: Chlamydia trachomatis Explanation Chlamydia trachomatis is an obligate intracellular parasite with a unique biphasic life cycle. It does not Gram stain; laboratory procedures used for diagnosis include isolation in tissue culture, EIA detection of antigen, immunofluorescent staining, cytologic examination for intracytoplasmic inclusions, and by the demonstration of nucleic acid by direct hybridization or by amplification techniques. It can cause inclusion conjunctivitis and ocular trachoma (as well as urethritis, lymphogranuloma venereum, urogenital infections, infertility, salpingitis and endometritis, reactive arthritis, etc.). The inclusion conjunctivitis presents as an acute follicular conjunctivitis and is usually self-inoculated from an infected genitourinary site. The patient frequently notes a foreign body presence in the eye. These symptoms are usually unilateral, and in the first 2 weeks, there is a mucoid discharge that becomes purulent. Usually the inclusion conjunctivitis resolves without complications, but some untreated or improperly treated cases can result in a prolonged infection that can last for months, and it can produce conjunctival and corneal scarring that is similar to mild ocular trachoma. Antibiotics, such as the tetracyclines, macrolides, rifampin, and some of the fluoroquinolones, have activity against chlamydia. Pseudomonas aeruginosa is a Gram-negative rod; it is a non-lactose fermenting, oxidase-positive motile bacteria. Growth on MacConkey agar is usually characterized by the production of a "grape-like" smell. A blue-green color, due to the production of the diffusible fluorescent pigments pyoverdin and pyocyanin, is characteristic of the colonies growing on MacConkey. Pseudomonas aeruginosa is a very common opportunistic source of human infections, especially in the hospital setting. Pathogenesis is due to its minimal nutritional requirements, relative resistance to antibiotics, and a host of other invasive and toxicogenic substances that it produces. It can cause a keratitis that is rapid in its development. The infection is usually the result of a previous injury to the eye, which causes an interruption in the epithelial surface and allows bacterial invasion of the underlying stroma. It can also be caused by contact lenses. Fever is usually absent, and leukocytosis is absent or minimal. The infection can lead to corneal ulceration, resulting in the rapid loss of ocular function; therefore, these infections need to be approached as a medical emergency. Scrapings from the floor of the ulcer exhibiting Gram-negative rods are strongly indicative of Pseudomonas aeruginosa and should necessitate treatment. Immediate initiation of combined topical and subconjunctival therapy with an aminoglycoside antibiotic such as gentamicin or tobramycin is advised. Topical steroids are sometimes used to reduce ocular inflammation. Haemophilus aegyptius is a Gram-negative coccobacillus; it is non-motile, fastidious bacteria requiring the presence of special factors for its growth on agar media. These factors are hemin and nicotinamide adenine dinucleotide, which are present in chocolate agar. The organism is indigenous to humans. It is an important cause of a purulent conjunctivitis called pink eye, and it can occur in outbreaks because of its contagious nature. The diffuse pink color of the sclera and the presence of a serous or purulent discharge are virtually diagnostic of Haemophilus aegyptius infection. Leukocytosis is absent. The infection is not acute in presentation. The treatment of Haemophilus aegyptius is with topical antibiotics. Because of the infectious nature of the infection, instructions should be provided to the patient to help prevent the spread of the infection to others. Bacillus cereus is a Gram-positive (or Gram-variable) rod that is aerobic and spore forming; it is ubiquitous in nature. Bacillus cereus is an important cause of food poisoning. It has also been recognized as an ocular pathogen. The ocular infection is acute in presentation and requires aggressive intervention to save the eye. It is many times associated with metal-on-metal projectile injuries, soil and dust contamination as seen in rural farm areas, and drug abuse. The presence of progressive corneal deterioration and ring abscess formation is a complication of panophthalmitis caused by Bacillus cereus. Except for infections with Pseudomonas aeruginosa, this finding is almost pathognomonic of Bacillus cereus. Because of the seriousness of the infection, early diagnosis is important. Patients presenting with ocular infections after trauma or in the setting of drug abuse should arouse suspicion. As with Pseudomonas aeruginosa, it is important for the prompt initiation of therapy before permanent structural changes occur, leading to loss of vision. Clindamycin and gentamicin in combination, administered intravitreally, is the course of therapy favored by ophthalmologists. The prognosis is poor and usually results in the loss of the eye unless an aggressive approach is undertaken; even then, there is assuredly some loss of vision. Topical steroids are sometimes used to reduce ocular inflammation. Acanthamoeba is a free-living amebae that can cause granulomatous amebic encephalitis and keratitis. Detection is usually made by observing the free-living motile organisms in a wet prep preparation. Acanthamoeba keratitis is a corneal infection that occurs in healthy people and is usually associated with contact lens wearers. To prevent Acanthamoeba keratitis, it is recommended that contact lenses be cleaned and stored with Benzalkonium chloride-preserved saline and solutions containing thimerosal with edetate. Swimming in fresh water (where the organism is naturally found) with contact lenses can predispose the wearer to Acanthamoeba keratitis. The keratitis is slow in developing and is frequently mistaken for herpes, bacterial, or fungal keratitis. Frequently, the average delay to definitive treatment can range from days to months. Symptoms include blurred vision, conjunctivitis, tearing, severe pain to the eye, and photophobia. The keratitis achieves an advanced stage in several days to several months, and it can exhibit patchy stromal infiltrates and dendriform epithelial involvement without frank corneal ulceration in its early stages. A ring corneal infiltrate is characteristic of this keratitis in its late stages. Early diagnosis, aggressive surgical debridement, and medical management can prevent eye damage. High concentrations of topical antimicrobial drugs (1% miconazole, 0.1% propamidine isethionate, and Neosporin) for a minimum of 3 - 4 weeks is part of the antibiotic therapeutic regimen employed in the treatment of Acanthamoeba keratitis. Staphylococcus aureus is a Gram-positive staining cocci that is catalase positive and coagulase positive. It is probably the 2nd most common bacterial isolate of human infections behind Escherichia coli and the most common cause of bacterial endophthalmitis. Staphylococcus aureus has a host of invasive and toxigenic characteristics that enhance the pathogenesis of the organism in the human host. The organism has been described as an etiologic agent of many infections including, but not limited to, conjunctivitis, endocarditis, septicemia, abscesses, and urinary tract infections. The conjunctivitis caused by Staphylococcus aureus is usually characterized as non-severe where there is little to no lid edema, scant purulent discharge, and normal cornea; however, in some cases the presentation can be severe. Topical agents are usually used to treat this infection such as cephalosporins or semisynthetic penicillins. In suspected cases of resistance, topical vancomycin should be considered. Candida albicans is a yeast. Yeast appear on Gram stain as large Gram-positive organisms that are approximately 3 - 5 times larger than Gram-positive cocci. They are aerobic and generally grow well on most non-selective agar media. Endophthalmitis due to yeast is generally a common and serious complication of intravenous drug use. Candida albicans is the most common fungal cause. It is usually of hematogenous origin, where the patient has infective endocarditis or some other infective process occurring. The symptoms are blurred vision, decreased vision, white cotton appearing exudative lesions in the choroid and retina with vitreous haziness, and eye pain. A definitive diagnosis is made by obtaining vitreous fluid for Gram stain and culture. The treatment consists of parenteral amphotericin B together with flucytosine. Intraocular amphotericin B administration as therapy is controversial. The incidence of permanent intraocular damage is high.

Case A 37-year-old man presents with a feeling of constant left ear fullness. He states that his hearing is not as sharp out of his left ear and has been this way for the past 3 months. The patient had multiple middle ear infections as a child and had myringotomy tube placements on 4 separate occasions throughout his lifetime. He has a history of severe perennial and seasonal airborne allergies, but at this time is not seeking treatment. Physical exam reveals a Weber test that lateralizes to the left ear; during the otoscopic inspection, a 'sac' filled with a cheesy white material is located on a retracted tympanic membrane. A small perforation of the tympanic membrane is suspected; there is the presence of the same cheesy white debris behind the membrane. Question Ico-delete Highlights Based on the history and physical examination, what is the mostly likely diagnosis?

Correct answer: Cholesteatoma Explanation The patient's most likely diagnosis is a cholesteatoma; many components of his history increase the chances of development of cholesteatomas. Chronic, severe airborne allergens have most likely led to chronic eustachian tube dysfunction, which is noted to be the most likely cause of cholesteatomas. The patient has had numerous middle ear infections throughout his life; this is a very specific variation of otitis media. Cholesteatomas are squamous, epithelium-lined sacs that are usually filled with desquamated keratin which (eventually) becomes chronically infected. The abnormal findings of the Weber test (the left lateralization) indicate that this patient most likely has some invasion of the contents into at least the middle ear and ossicles, leading to conductive hearing loss. These sacs can begin to cause erosion to surrounding structures, including the tympanic membrane (rupture), mastoid, and ossicles. This can eventually disrupt the functions of the inner ear and the facial nerve. Treatment is typically surgical marsupialization of the sac. Barotrauma is a pathology that also results from eustachian tube dysfunction, but it is caused mainly by extreme barometric changes in pressure (as experienced in air travel, underwater diving, or any rapid altitudinal change). Both exostoses (bony overgrowths of the ear canal that are typically considered benign) and otitis externa (also known as swimmer's ear or an outer ear infection) are incorrect choices. Gradenigo syndrome is characterized by the triad of retro orbital pain, acute otitis media, and ipsilateral abducens nerve paralysis. This is an extremely rare complication and is not likely in the above paatient.

Case A 34-year-old man presents due to something being "wrong" with his left ear. He reports his hearing has been gradually declining, but he recently noticed some discomfort and malodorous discharge draining from this ear. He denies any trauma to the ear and any symptoms in his right ear. Upon further questioning, he admits to some tinnitus and mild vertigo. He otherwise feels well. He denies nasal symptoms, headache, sore throat, and fevers. His past medical history is unremarkable; he has no known medical conditions or history of surgery; he takes no medications and has no allergies. He lives with his wife and 2 children; he works as an office manager, and he denies the use of alcohol, tobacco, and drugs. On physical exam, his vitals are normal. Examination of the left ear reveals mucopurulent drainage within the external auditory canal. The tympanic membrane is disrupted by a retraction pocket within the upper portion, with some thick yellow debris and a polyp protruding from the pocket. Hearing tests are not performed. The right ear reveals mild tympanosclerosis on the tympanic membrane, but it is otherwise normal. The remainder of the patient's exam is normal. Question What is the most likely diagnosis?

Correct answer: Cholesteatoma Explanation This patient is presenting with a cholesteatoma, which a benign neoplasm of the tympanic membrane. It is considered a complication of chronic otitis media, so this patient could be expected to report a history of frequent otitis media. (The tympanosclerosis on the right is another clue.) The cholesteatoma is an epidermal inclusion cyst. Complications can include infection and (more significantly) erosion into bone and nerve damage. Contact dermatitis of the ear canal can produce local irritation with pain and/or pruritus, and if the inflammation is severe enough, discharge. The hearing loss and the physical exam findings of the pocket and polyp are not associated with contact dermatitis. Labyrinthitis is a condition that can cause hearing loss, tinnitus, and nausea. The cause is not well understood, but it involves the inner ear, not the tympanic membrane. Ear discharge is not associated with labyrinthitis. Otitis externa is an inflammatory condition of the ear canal, and can be most commonly caused by fungal or bacterial organisms. It can lead to ear pain and discharge. Typically, it should not causing hearing loss and tinnitus; furthermore, it would not cause the findings on this patient's tympanic membrane. Psoriasis is a dermatologic condition that affects skin all over the body. It is typically described as silvery scale over bright red plaques. It can affect the ear canals, causing pain and pruritus, but it does not typically the affect the tympanic membrane.

Case A 73-year-old African-American man presents for a routine follow-up since being diagnosed with glaucoma. His ophthalmologist has started him on a topical carbonic anhydrase inhibitor (dorzolamide). Question What is the site of action of this drug?

Correct answer: Ciliary body epithelium Explanation The correct response is the ciliary body epithelium. Carbonic anhydrase reduces intraocular pressure by reducing the synthesis of aqueous humor. Aqueous humor is produced in the epithelium of the ciliary body; it travels through the pupil into the anterior chamber and drains through the trabecular meshwork into the canal of Schlemm. A topical drug that decreases aqueous humor production would be expected to act at the ciliary body epithelium. The trabecular meshwork and canal of Schlemm are part of the outflow pathway of aqueous humor; they are not involved in aqueous humor production. The corneal endothelium and retinal pigment epithelium are not involved in aqueous humor production.

Case A 22-year-old woman presents with a 1-day history of foreign body sensation in her right eye. She woke up with pain in the right eye, and she immediately had trouble opening her eye. She wears soft contact lenses and does not remember how long the last pair was in for. She removed her contact lenses the night before the pain started. There was no trauma. Visual acuity was 20/40 O.U. without corrective lenses, and extraocular movements were within normal limits. With fluorescein stain, a defect is noted; it is round and found at the center of the cornea. No foreign bodies are noted. Question What intervention is indicated?

Correct answer: Ciprofloxacin 0.3% solution Explanation The correct answer is ciprofloxacin 0.3%; the patient has a corneal abrasion due to contact lenses. Ciprofloxacin covers pseudomonas, and pseudomonas should always be covered when someone gets a corneal abrasion from contact lenses. A pressure patch is never used in someone who gets a corneal abrasion from contact lenses because of the risk of developing infectious keratitis. Trifluridine drops are used as an antiviral for Herpes Simplex keratitis; they are not used for corneal abrasions. Ketorolac solution is used to help pain in a corneal abrasion, but it is not the mainstay of treatment. Sulfacetamide solution is not used for corneal abrasions caused by contact lenses because it does not cover pseudomonas adequately.

A 25-year-old Hispanic man underwent an orchiectomy and retroperitoneal lymph node dissection for testicular carcinoma 1 month ago. Staging revealed stage IIc disease, and he is currently undergoing chemotherapy with cisplatin, bleomycin, and vinblastine. While using these drugs, he has developed nausea, occasional vomiting, and tinnitus. More recently, he also noticed more difficulty hearing and is very worried about his auditory symptoms. He had no other relevant medical history. His vital signs are within normal range, and the physical examination is normal. Question What is the most likely cause of his hearing symptoms?

Correct answer: Cisplatin Explanation Cisplatin is the chemotherapeutic drug most associated with sensorineural hearing loss. It is a platinum-containing agent which exerts its effect by binding to DNA with preferential binding at N-7 position at guanine and adenosine. The mechanism is multifactorial but is thought to involve free-radical mediated damage to the inner ear, involving the stria vascularis and cochlear hair cell death. The hearing loss is bilateral, permanent, and progressive, with more pronounced involvement of the high-frequency hearing component. Avoidance of loud noises might slow the symptoms' progression. Monitoring with repeated audiograms before each cisplatin cycle is also warranted. Other important side effects of cisplatin include nephrotoxicity (which requires diuretics and hydration during each application), electrolyte disturbances such as hypomagnesemia, hypocalcemia, hypokalemia and hypophosphatemia, moderate myelosuppression, nausea and vomiting (requires antiemetic prophylaxis), and peripheral neuropathy. It is used mainly in the following conditions: 1.Testicular carcinomas. 2.Ovarian germ cell carcinoma. 3.Bladder carcinoma. Other uses are in certain cancers of head and neck, thyroid carcinoma, breast cancer, etc. The most severe toxicity associated with bleomycin is pulmonary . Diffuse fibrosis, cavitation, atelectasia, and consolidation may ensue. A fatal outcome ensues in 1% of these patients. Radiation to the lung fields can potentiate bleomycin toxicity. Other side effects of bleomycin are skin toxicity, with hyperpigmentation, hyperkeratosis, erythema and ulcers, myelosupression, nausea, and vomiting. A potentially fatal severe idiosyncratic reaction has been described in 1% of lymphoma patients using bleomycin, consisting of hyperthermia, hypotension, and circulatory collapse. The mechanism is unknown. A trial dose of bleomycin should be given to lymphoma patients before continuing with this drug. Vinblastine is associated with myelosupression peaking 7 to 10 days after each dose, nausea and vomiting, gastrointestinal disturbances, and peripheral neuropathy. The most common neuropathy symptoms are distal paresthesias and numbness. Inadvertent intrathecal injection of vinblastine, or other vinca alkaloids, produces devastating neurological damage and is highly lethal. However, systemic vinblastine is not associated with CNS side effects. Symptoms due to central nervous system metastases are rare in testicular carcinoma. Paraneoplastic encephalitis has been described in association with testicular cancer. This syndrome is characterized by seizures, irritability, depression, and dementia and therefore would not explain the patient's symptoms.

Question A white female comes into the evening clinic with complaint of red eyes with a sticky discharge. Her eye feels irritated, the pain is mild. She tells you she shared towels with her boyfriend who had a red eye. She asks you if you think her boyfriend gave her an infection. What is your working diagnosis?

Correct answer: Conjunctivitis Explanation Conjunctivitis can be caused by bacterial, viral, allergic, and irritant etiologies. Clients usually complain of red eyes and a sticky or watery discharge. The type of discharge helps to determine the etiology-watery in viral, and sticky green or yellow, in bacterial conjunctivitis. Irritation is common, but severe pain and photophobia are not. Bacterial or viral conjunctivitis is usually self-limited, but it may be treated with a topical antibiotic, without steroids, such as sulfacetamide (10% 3-4 times/day). Topical aminoglycoside should be reserved for more refractory disease. Allergic conjunctivitis may be effectively treated with a new class of non-steroidal topical anti-inflammatory agents. Irritant conjunctivitis, including dry eyes, may be treated with topical, non-preserved lubricants. Blepharitis may be seborrheic or ulcerative. Seborrheic (non-ulcerative) blepharitis is commonly associated with seborrhea of the face, eyebrows, external ears, and scalp. Inflammation of the eyelid margins occurs, with redness, thickening, and often the formation of scales and crusts, or shallow marginal ulcers. Ulcerative blepharitis is caused by bacterial infection (usually staphylococcal) of the lash follicles and the meibomian glands. Chalazion is a mildly painful swelling of the eyelid margin, due to granulomatous inflammation, and usually resolves spontaneously with warm soaks and time. Multiple chalazion may be seen in diabetes, alcoholism, and malnourishment. Rarely, curettage may be required. Ectropion is the outward turning of, usually, the lower lid occurring in older people. Surgery is indicated if ectropion causes excessive tearing, exposure, keratitis, or a cosmetic problem. Foreign Body sensation is most commonly due to corneal or conjunctival foreign bodies. Other causes are disturbances of the corneal epithelium and rubbing of eyelashes against the cornea (trichiasis).

Case A 22-year-old man presents with a 3-month history of worsening diarrhea that comes and goes. While performing a comprehensive oral exam, you note 2 lesions on the buccal mucosa on the right side of the oral cavity. You document these lesions as 2 round lesions that measure approximately 2 mm in diameter and have a white-yellow center, which is surrounded by a red halo. Pertinent positives also include a reduced appetite, abdominal pain, and cramping. Question Ico-delete Highlights Based on these current history, physical exam findings, and possible disease pathologies, what diagnosis is very high on your differential?

Correct answer: Crohn's disease Explanation The patient above has an aphthous ulcer, also sometimes referred to as a canker sore or aphthous stomatitis. These painful, open sores are found in the oral cavity and are the most common form of mouth ulcer. Many times these lesions are described as having a white or yellow center and are surrounded by a bright red area. They are benign, noncancerous, noninfectious, and many times, the cause is unknown; however, aphthous lesions are common extra intestinal manifestations found in patients suffering from Crohn's disease. Patients who have the other form of inflammatory bowel disease, ulcerative colitis, will not typically develop such signs as aphthous ulcers. More common signs of cholecystitis include right-sided abdominal pain, vomiting, fever, a positive Murphy's sign, or even palpable gallbladder. GERD symptoms generally surround the complaint of heartburn, although it can cause issues such as asthma, chronic cough, chronic laryngitis, sore throat, or even non-cardiac chest pain. Colon cancer can present with anemic qualities on serology tests, abdominal pain, changes in bowel habits, as well as hemoccult-positive stool.

Case A 32-year-old woman presents with a 3-day history of irritation, burning, itching, and redness of both eyelids. She denies fever, visual changes, and photophobia. On physical examination, you note the presence of scales clinging to the eyelids bilaterally. Question What is the proper management in this case?

Correct answer: Daily cleaning with a damp cotton applicator and baby shampoo Explanation The scenario presented above depicts a patient with anterior blepharitis, which is a common disorder seen in primary care; it typically consists of a recurrent bilateral inflammation of the lid margins that involves the eyelid skin, eyelashes, and associated glands. Commonly, the underlying cause is seborrhea, which usually originates in the scalp, eyebrows, or ears. Sometimes, anterior blepharitis can be ulcerative, and the origin in the presented case is staphylococci. Anterior blepharitis can typically be resolved and controlled by cleaning the affected areas daily using a damp cotton applicator, warm water, and a baby shampoo mixture. The object of the daily cleaning is to remove the visible scales as efficiently as possible. None of the other listed options are an appropriate treatment plan for anterior blepharitis. Patients can also be diagnosed with what is known as posterior blepharitis, which is an inflammation of the meibomian glands of the eyes. It is usually staphylococcal in origin, and it typically presents with significantly worse signs and symptoms, such as hyperemic lids, the presence of telangiectasias, inflammation of the gland or their orifices, or even abnormal secretions; tears may be described as being frothy or greasy. More significant cases of posterior blepharitis can lead to conjunctivitis, hordeola, chalazions, eyelash trichiasis, or even corneal vascularization and thinning. Treatments for posterior blepharitis may consist of long-term oral antibiotic therapy, short-term topical steroids, or short-term topical antibiotics eye drops; if significant complications are evident, an ophthalmologist referral is indicated.

Case Ico-delete Highlights A 27-year-old woman presents with a 3-day history of left eye pain. The patient notes sensitivity to light, and she comments that her eye throbs in pain at night. On physical examination, there is redness and a loss of visual acuity. There is no drainage from the eye or visible lesions. Slit-lamp exam demonstrates inflammatory cells in the aqueous humor. Question What would be an appropriate treatment for this patient?

Correct answer: Dexamethasone and homatropine ophthalmic drops Explanation The correct response is dexamethasone and homatropine ophthalmic drops. The clinical picture is suggestive of uveitis. Patients with uveitis usually note redness, pain, photophobia, and visual loss. Treatment of anterior uveitis is with topical steroids; posterior uveitis may require oral corticosteroids. There are multiple causes of uveitis, but it is primarily immunogenic and seen in the context of another systemic condition. Cool compresses and artificial tears are not an effective treatment for uveitis. Cortisporin is effective against bacterial conjunctivitis. Patients typically present with a copious discharge in the affected eye with mild discomfort. There is no loss of visual acuity. Oral acyclovir is used in the treatment of herpes simplex keratitis. Dendritic ulcer is seen on staining with fluorescein, which is not seen in this patient. IV acetazolamide is used in the treatment of acute angle-closure glaucoma. Patients typically present with rapid onset of severe eye pain, profound visual loss, and halos around lights. The symptoms of severe eye pain, profound vision loss, and halos around lights are not seen in this patient.

Case A 52-year-old man presents with gradual bilateral deteriorating vision; he was treated at a visit 6 months prior, and was instructed by his ophthalmologist to follow-up in 6 months. The patient has a 15-year history of NIDDM and is on an oral hypoglycemic (sulfonylurea drug); his blood sugar level is usually well-maintained. He has some skin trophic changes in the lower limbs without impairment of sensation. Vision is 6/36 bilaterally. The lens is clear, and the vitreous shows no abnormalities. The image shown is the fundus of the patient's right eye. Question What is the most likely diagnosis?

Correct answer: Diabetic retinopathy Explanation Diabetic retinopathy is 1 of the most common complications of diabetes. Diabetes is well-known to cause microvasculopathy and neuropathy. Vasculopathy affects the kidneys and the retina of the eye. Retinopathy is characterized by neovascularization and retinal hemorrhages, and may also lead to fibrosis, adhesions, and subsequent retinal detachment. Early detection, prevention, and treatment help to retain vision. The most widely used treatment is panretinal photocoagulation (PRP) with a laser; it is always applied to the periphery of the retina, outside the arcades. As shown in the image, the white dots are the scars of the laser; the center shows the minute retinal hemorrhages and the neovascularization. Diabetic macular edema is characterized by marked, rapid, defective vision, usually in 1 eye, which may be followed by the other eye. The image shows central yellow dots, or hemorrhages, and is treated with central laser or intravitreal injection with steroids. Central retinal vein occlusion may occur in diabetics; it is usually unilateral, and the retina shows congested veins and marked hemorrhages. It may be treated with PRP. Central retinal artery occlusion causes unilateral sudden blindness; it may be due to an embolus or thrombus, and the fundic picture shows a pale white retina with a central cherry red spot. Age-related macular degeneration occurs in older people, usually after age 70. It may be exudative with central edema and exudates, or dry with macular scarring. It causes gradual diminution of vision.

Case A 42-year-old man presents with a 24-hour history of severe vertigo, tinnitus, hearing loss, nausea, and vomiting intermittently; episodes last at least 30 minutes. After a thorough history and examination, the patient is diagnosed with Ménière's disease. Question What medication is he most likely to be given to treat the acute symptoms?

Correct answer: Diazepam (Valium) Explanation Diazepam (Valium) is a benzodiazepine medication that can be given 5-10 mg IV in order to treat an acute attack during a severe episode of Ménière's disease. Other medications that are fast-acting and can effectively treat an acute attack are atropine and transdermal scopolamine. A few second-line choices for acute treatment are droperidol, promethazine (Phenergan), and diphenhydramine (Benadryl). With the exception of the transdermal scopolamine, all of the listed medications are given by IV or IV push in a controlled setting. Hydrochlorothiazide/triamterene (Dyazide/Maxzide) is a combination medication not used for an acute attack, but it is the first-line treatment for maintenance in patients who have had recurrent attacks. The goal of treating patients with Ménière's disease is to prevent the number of disabling spells of vertigo that they experience. Along with a low sodium diet, diuretics are the mainstay of long-term treatment. Patients should be instructed about a low sodium diet and are to restrict their sodium intake to 1500 mg per day. Other lifestyle changes, such as smoking cessation, caffeine restriction, and alcohol restriction, should also be followed when applicable. If the sodium restriction and diuretic are not effective, then patients should be counseled on an even more restrictive diet of 1000 mg sodium per day and have their diuretic dose increased (if not contraindicated) before considering another treatment option. Meclizine (Antivert) is an antihistamine medication that can be used for maintenance and long-term prevention of recurrent attacks. Acetazolamide (Diamox) is another diuretic medication choice that can be used for maintenance and long-term prevention of recurrent attacks. Dimenhydrinate (Dramamine) is another antihistamine medication that can be used for maintenance and long-term prevention of recurrent attacks.

Case An 11-year-old girl presents with an increase in the frequency of her nosebleeds. She has experienced 3 episodes in the last week. Her mother was able to control and stop the bleeding by applying direct pressure to her external nasal area. Past medical history shows no significant issues with bleeding discrepancies or coagulopathies; she had a tonsillectomy when she was 8 years old; there were not any postoperative complications. Physical examination reveals a raw and irritated anterior right nare; the presence of dried crusted blood is noted. Question Ico-delete Highlights What is most likely cause of this patient's anterior epistaxis?

Correct answer: Digital trauma Explanation Anterior epistaxis is an extremely common presenting complaint in pediatric patients in a primary care office. Common factors are usually due to local trauma that predisposes patients to developing anterior epistaxis; they include (but are not limited to) nose digital trauma (nose picking), forceful nose-blowing/rubbing, administration of supplemental nasal oxygen, or low humidity. A deviated nasal septum can also predispose a patient to epistaxis, but it was not present on this patient's physical examination. The most likely source of this patient's epistaxis is Kiesselbach's plexus, which is a superficial venous plexus in the anterior septum. This is further supported by the physical exam findings of dried blood being seen in only 1 of the nares of the nose. Treatment of anterior epistaxis is usually direct pressure by compression at the site of the nares for at least 15 minutes; the patient should be in the sitting position and leaning slightly forward in order to decrease the swallowing of blood. A clotting disorder (or coagulopathy) can predispose a patient to developing epistaxis, but it is only found to be the underlying cause 5% of the time. Hypertension is rarely the cause of epistaxis in the pediatric population.

Case A 19-year-old woman presents for management of seasonal allergic rhinitis. Her symptoms begin in the spring peak in the late summer/early fall, and have worsened over the last 2-3 years. She has taken over-the-counter (OTC) diphenhydramine (Benadryl) 25 mg in the past, which controlled her symptoms well, but her use of the medication was impaired by the drowsiness she experienced. Currently, she takes loratadine (Claritin) 10 mg daily, but she has not experienced any improvement in her symptoms. Question What medication is the best choice to initiate next in this patient?

Correct answer: Mometasone furoate (Nasonex) 100 mcg (2 sprays) once daily per nostril Explanation The treatment options for patients with allergic rhinitis are numerous, and choosing which medication to use in a particular patient is based on their clinical symptomatology as well as its timing, efficacy, cost, and side effects. Intranasal corticosteroid sprays (e.g., mometasone furoate, fluticasone propionate [Flonase]) have revolutionized the treatment of allergic rhinitis, and evidence-based literature shows that these medications are more effective than non-sedating antihistamines. In some cases, they are also less expensive. Because intranasal corticosteroid sprays also can shrink hypertrophic nasal mucosa seen in allergic rhinitis, increasing airway patency and enhancing drainage, these medications are also crucial in treating allergy patients prone to recurrent sinusitis. Side effects are limited, with epistaxis being the most bothersome. Several over-the-counter (OTC) and prescription antihistamines are available. First-generation antihistamines (diphenhydramine, brompheniramine, chlorpheniramine) offer immediate but temporary control of many of the most troubling symptoms of allergic rhinitis. While these medications are inexpensive, they are usually associated with higher rates of drowsiness. Second-generation antihistamines (cetirizine, fexofenadine [Allegra], loratadine, and desloratadine [Clarinex]) boast less sedation, but they can still cause some levels of drowsiness and are usually more expensive—even those that are now over the counter (loratadine and cetirizine). Other potential side effects of oral antihistamines include xerostomia (dry mouth), dizziness, decreased urination, and blurred vision. Intranasal anticholinergic agents (ipratropium bromide) can be helpful as adjunct therapy when rhinorrhea is a major symptom, but these nasal sprays are not as effective as intranasal corticosteroids for treating allergic rhinitis. They are more useful for treating vasomotor rhinitis. Oxymetazoline is an intranasal alpha agonist that causes vasoconstriction. It may be used for relief of nasal congestion caused by allergic rhinitis. It is not recommended to be used long-term, however, due to the risk of rebound vasodilation and development of dependence. As stated above, allergy patients can be prone to both acute and recurrent/chronic bacterial rhinosinusitis as a result of allergy-induced hypertrophy of the nasal mucosa and impaired ostiomeatal complex drainage. While antibiotics (e.g., amoxicillin) may be necessary in these patients to treat the associated infection, they are not indicated in the current care of this patient.

Question What description of "red eye" is caused by acute glaucoma?

Correct answer: Dilated central and conjunctival vessels, dilated pupil, and decreased vision Explanation Acute glaucoma is the sudden increase in intraocular pressure secondary to blocked drainage from the anterior chamber. It manifests as dilatation of both the central and conjunctival vessels. Pupil size is often dilated, and vision is decreased. It is considered an ocular emergency. Conjunctival injection is the dilatation of the conjunctival vessels. Pupil size and vision are unaffected. It is a result of superficial processes, such as infection, allergies, irritation, and vasodilators. Ciliary injection is the dilatation of the branches of the anterior ciliary artery, and it manifests as a diffuse flush and dilatation of the central vessels around the iris. The pupil may be normal, but it is usually small and anisocoric. Vision is decreased. It is a result of disorders of the cornea or inner eye, and requires immediate attention. A subconjunctival hemorrhage is a patch of blood that appears outside the vessels between he conjunctiva and sclera. Pupil size and vision are unaffected. It may be the result of trauma, bleeding disorders, or sudden increases in venous pressure, such as from coughing. It is usually not of clinical significance. Blepharitis is inflammation of the eyelid margins. Vessels, pupil size, and vision are all normal. Seborrhea and staphylococcal infections are common causes.

Case A 4-year-old boy accompanied by his mother presents with fever, sore throat, muffled voice, and breathing and swallowing difficulty. The child is leaning forward with his head and nose tilted upward and forward. He is irritable, with moderate respiratory distress and inspiratory stridor. Pulse is 94/min; BP is 110/70 mm Hg, and temperature is 101 F. Question What is the next best step to confirm the diagnosis?

Correct answer: Direct fiberoptic laryngoscopy in the operating room Explanation Direct fiberoptic laryngoscopy is the correct answer. The patient has symptoms of acute epiglottitis, a diagnosis that can be made on clinical grounds. The next step is direct fiberoptic laryngoscopy performed in a controlled environment - usually the operation theater - in order to visualize and culture the edematous larynx, as well as to secure the airway through placement of an endotracheal tube1. Direct visualization in the examination room with tongue depressor or indirect laryngoscopy is not recommended because of the high risk of immediate laryngospasm and complete airway obstruction1. Lateral neck radiograph usually reveals an enlarged edematous epiglottis (thumbprint sign). Laboratory investigations like complete blood count (CBC) typically reveal elevated leukocytes with neutrophil predominance, and blood cultures are usually positive1. These investigations assist the diagnosis but may delay the critical step of placing the endotracheal tube1.

Case A 4-year-old Caucasian boy is seen due to a 2-week history of purulent nasal discharge. He has been afebrile and has had no respiratory symptoms. Past history is unremarkable except for his mother's assessment that "he gets into everything." Examination revealed only a right-sided, purulent nasal discharge, which was greenish-brown in color and extraordinarily foul smelling. Question Ico-delete Highlights What is the most direct method of diagnosing and treating this child's likely condition?

Correct answer: Direct visualization of the right nasal vestibule Explanation The history of an extremely foul smelling, unilateral purulent nasal discharge without other symptoms in an active young child strongly suggests a retained nasal foreign body, most readily diagnosed via direct visualization. Culture and sensitivity may be useful in guiding antibiotic therapy but will not diagnose the underlying cause. Similarly, Gram stain and KOH prep may be useful in guiding antibiotic therapy but will not diagnose the underlying cause. Sinus radiographs will not visualize the nasal foreign body. Radiography may not prove helpful, as such foreign bodies are frequently vegetable in nature and are radiolucent.

Case You are evaluating a 24-year-old woman for bilateral eye pain. She describes red, itching, irritated eyelids for several weeks. She states she has had "several bouts" of similar symptoms over the last few years. Exam is consistent with blepharitis. She does not wear contacts, and she occasionally wears eye makeup. She denies any other infectious complaints. Question What is the most appropriate treatment?

Correct answer: Discuss good eye hygiene and eyelid scrubbing Explanation The correct answer is that you should discuss good eye hygiene and eyelid scrubbing with the patient. Blepharitis is chronic inflammation of the eyelids. Patients often present with irritation of the lid margins that may produce minor crusting, a gritty sensation when blinking, eye itching and redness but a full preservation of vision. The majority of patients have recurrent bouts of inflammation, which can be resolved by eyelid washing and avoidance of eye makeup.1 The treatment of blepharitis rarely requires antibiotics. While it is possible to have a secondary infection, typically staph aureus, there is no evidence of a secondary infection in our patient.2 Initiation of ophthalmic steroids should be done under the guidance of an ophthalmologist. Additionally, there is no benefit of an ophthalmic steroid in the treatment of blepharitis. The majority of cases of uncomplicated blepharitis do not need to be referred to an ophthalmologist unless the diagnosis is uncertain or there are other complicating factors. Additionally, there is no surgical correction for uncomplicated blepharitis. Our patient does not have signs and symptoms of increased intraocular pressure.

Case A 24-year-old woman presents with a 13-day history of headache, fever, and green nasal discharge. Past medical history is positive for allergic rhinitis for 3 weeks, but she is out of her medications. Allergies include dust, cats, sulfa drugs, and penicillin. On physical exam BP is 120/78 mm Hg, pulse is 80 bpm, respirations are 12/min, and temperature is 101° F. HEENT exam is positive for right maxillary tenderness, green nasal discharge, and increased sinus pressure with leaning forward. Neurological exam is normal. Question What would be the best first-line therapy prescription for this patient? Answer Choices 1 Trimethoprim-sulfamethoxazole 2 Doxycycline 3 Amoxicillin 4 Amoxicillin-clavulanate 5 Moxifloxacin

Correct answer: Doxycycline Explanation In cases of rhinosinusitis of <10 days duration, supportive care is best. In this case, however, the patient has a 13-day history of symptoms, so antibiotic therapy is appropriate. Doxycycline is the best alternative to first-line therapies such as amoxicillin or amoxicillin-clavulanate because it can be used in patients with a penicillin allergy (as in the patient described in this case). In addition, there is a high rate of S. aureus colonization of the sinus mucosa in chronic rhinitis sinusitis patients, and doxycycline therapy has been shown to decrease polyp size due to its anti-inflammatory and antistaphylococcal effects. Trimethoprim-sulfamethoxazole is often included as part of first-line therapy for sinusitis; this patient is allergic to sulfa-drugs, making its use inappropriate. In addition, trimethoprim-sulfamethoxazole is not recommended for empiric therapy because of its high rates of resistance of S. pneumoniae. Similarly, amoxicillin is not appropriate because of the patient's allergy to penicillin. Amoxicillin-clavulanate is best for non-penicillin allergic patients, unlike in this case. High doses of amoxicillin-clavulanate are often used in patients over 65, the immunocompromised, or those recently hospitalized. Although a respiratory fluoroquinolone such as moxifloxacin is another option for penicillin-allergic patients, it is a second-line therapy drug. There is a need to limit overuse of fluoroquinolones in an effort to slow development of resistance to this antibiotic class, so moxifloxacin is not the best choice in this case.

Case A 42-year-old man presents with a 4-day history of worsening headache, stuffy nose, and excessive yellow-colored nasal discharge. He admits to facial pain, as well as a dry cough. He denies shortness of breath, abdominal pain, nausea, or vomiting. He is a non-smoker, has no significant past medical history, and is only taking acetaminophen. On exam, he has a slight fever of 99.2° F orally, pulse of 86/min, BP of 120/76 mm Hg in the left arm while sitting, and SPO2 of 94% on room air. Lungs are clear and abdomen is normal. Nasal mucosa appears boggy, and there is tenderness with palpation over the facial bones (maxillary area). Pharynx is without exudates. Question What component of the history prompts you to only recommend symptomatic therapy at this time?

Correct answer: Duration of symptoms Explanation The correct response is duration of symptoms. The patient's condition is highly indicative of acute viral rhinosinusitis (acute sinusitis). Acute sinusitis clinically includes such symptoms and signs as green/yellow, purulent discharge, facial pain or pressure over the affected sinus, nasal obstruction, and congestion; it may also include cough, malaise, fever, or even headache. Acute sinusitis has an acute onset of symptoms, ranging from 1 - 4 weeks in length of duration by the time the patient presents clinically. More commonly, the origin of sinusitis is viral; however, symptoms relating to this will resolve as time passes (not intensify or worsen). Given the history of the present illness given by this patient and the physical examination findings, the cause of his sinusitis is highly likely to be viral.

Question Which of the following drugs used in glaucoma therapy has a prolonged longest duration of action and can be administered relatively infrequently?

Correct answer: Echothiophate Explanation Elevated intraocular pressure, if untreated, can lead to retinal and optic nerve damage, restriction of the visual fields, and blindness. The open-angle form of glaucoma is a chronic condition involving elevated intraocular pressure that often responds to drug therapy. A variety of topical drops are available for direct instillation into the conjunctival sac for treatment of this condition. Intraocular pressure is a function of the volume of aqueous humor present in the anterior cavity of the eye. The production of aqueous humor by the ciliary epithelium is normally balanced by the drainage of aqueous humor by the trabecular system at the anterior chamber angle. Ciliary epithelium possesses β-adrenergic receptors that are coupled with the cyclic adenosine-3',5'-monophosphate (cAMP) secondary messenger system. Activation of this cyclic nucleotide system by beta-adrenergic stimulants causes the ciliary epithelium to increase the output of aqueous humor. Therefore, beta agonists such as albuterol can worsen the elevated intraocular pressure of glaucoma. Muscarinic antagonists such as atropine, ipratropium, and tropicamide produce mydriasis and cycloplegia. The blockade of ciliary muscle function by antimuscarinics decreases the anterior chamber angle and impedes the drainage of aqueous humor, thus exacerbating a condition of glaucoma. Drug treatment of glaucoma is directed at reduction of aqueous humor secretion, improvement of aqueous drainage, or both. Beta-adrenergic blockers, diuretics, cholinomimetics, and alpha-adrenergic agonists are used in glaucoma therapy: Reduction of aqueous humor secretion Beta blockers are front-line drugs employed in glaucoma therapy. These derivatives of propranolol are effective in open-angle glaucoma and offer dosing convenience while producing relatively few adverse effects. The ophthalmologic drugs are available as topical drops and include timolol, betaxolol, carteolol, metipranolol, and levobunolol. Diuretics such as the carbonic anhydrase inhibitor, acetazolamide, reduce the availability of bicarbonate ion (HCO31-), a critical component of aqueous humor. As a result the secretion of aqueous humor by the ciliary epithelium is diminished. Enhancement of aqueous humor drainage Cholinomimetics are front-line drugs in the treatment of open-angle glaucoma. The cholinomimetics (or cholinergic agonists) include direct-acting choline esters with M3-muscarinic activity such as pilocarpine and carbachol, and indirect-acting cholinesterase inhibitors such as physostigmine and echothiophate (an ultra-long-acting organophosphate). Cholinesterase inhibitors are nonspecific cholinergic agonists that bind available cholinesterase enzymes such as acetylcholinesterase and plasma cholinesterase. Compared to acetylcholine, these drugs are more slowly hydrolyzed from the enzyme. More importantly, cholinesterase is prevented from inactivating acetylcholine while cholinesterase inhibitor molecules are attached. Physostigmine exhibits reversible cholinesterase inhibition whereas anticholinesterases such as the organophosphates, echothiophate and isoflurophate, produce extended action in glaucoma therapy. The prolonged duration of effect of these cholinergic drugs aids in patient compliance in the treatment of glaucoma. The extended effect is due to the irreversible nature of the cholinesterase enzyme inhibition and the time required to synthesize new enzyme to replace that which has been hydrolyzed. (It should be noted that the risk of cataract development is higher with the organophosphate derivatives than with other drugs used in glaucoma therapy.) The cholinomimetics produce miosis and contraction of the ciliary muscle which increases tension on the trabecular meshwork, opens pores, and improves the drainage of aqueous humor into the canal of Schlemm. Alpha agonists such as epinephrine cause an increased outflow of aqueous humor through the trabecular meshwork at the anterior chamber angle.

Case A 25-year-old woman presents with constant sneezing episodes that have progressively worsened over the past few months. She also suffers from severe bilateral eye irritation, excessive tearing, and pruritus. On physical examination, you note hypertrophic nasal mucosa that is boggy in appearance, with the turbinates appearing pale with a presence of clear discharge. The patient states that symptoms are especially bothersome during spring months, but they do occur to some degree during the fall season as well. Question A nasal swab is most likely to reveal a prominent amount of what type of cells?

Correct answer: Eosinophils Explanation The correct response is eosinophils. The scenario above is describing a common presentation of allergic rhinitis, most likely leaning more towards seasonal rhinitis. This is a very common condition in the United States, with the prevalence in adults ranging from 14% to up to 40%. Seasonal allergic rhinitis is most commonly caused by pollen and spores, flowering shrubs and tree pollens common in the spring, flowering plants and grasses in the summer, and ragweed and molds being the main culprits in the fall. Perennial rhinitis is another classification of allergic rhinitis; it refers to indoor allergens, such as dust mites, pet dander, and cockroach. Patients, when exposed to the items listed above, as well as those who are predisposed to sensitivity will have a prominent activation of the humoral (B-cell) and cytotoxic (T-cell) immune responses along with allergen-specific IgE responses, leading to a release of inflammatory mediators. Interleukin and cytokine release is also a part of this response; this subsequently leads to a specific release of mast cells, eosinophils, plasma cells, and basophils. These circulating cells will migrate to the ocular and nasal epithelium, and they contribute directly to the symptoms seen in patients with allergic rhinitis. A nasal swab completed in a patient with allergic rhinitis will reveal such cytology as eosinophils, mast cells, neutrophils, and even lymphocytes. Cytokines, monocytes, red blood cells, and antigens are not components of the human body reaction to an allergen-specific IgE response.

Case A 7-year-old boy presents to his pediatricians office with a 3-week history of clear nasal discharge, itchy eyes, and excessive sneezing. The family recently took in a stray cat, which sleeps with the boy. On examination of the nasal passage, you note swelling of the turbinates with clear drainage from the nares bilateral. On examination of the oral cavity, you note slight pharyngeal erythema, post-nasal drip, and no tonsillar edema. Question Based on your suspected diagnosis, if you were to microscopically exam the nasal secretions, you would expect to see an excess amount of which of the following?

Correct answer: Eosinophils Explanation This patient is presenting with allergic rhinitis. This condition typically exhibits nasal secretions that are rich in eosinophils, which are the specific type of leukocytes that are involved in allergic reactions. Neutrophila would be seen in excess in the case of infection. Lymphocytes would be seen in excess in the case of an immune reaction. Monocytes and histiocytes would be seen in excess if bacteria were present, requiring their phagocytic activity.

Case A full-term Caucasian infant is a product of an uncomplicated pregnancy, labor, and delivery. Birth weight is 8 lb and length is 21 inches; head circumference is 14 inches. As you examine the newborn's mouth, you identify 6 raised yellowish-white raised lesions, each approximately 1 mm in size; these lesions are located along the mid-palatine raphe. The remainder of the physical examination is normal. Question What is the most likely diagnosis?

Correct answer: Epstein's pearls Explanation Epstein's pearls are 1 of 2 forms of cystic, or fluid-filled, lesions of the palate found in approximately 60% of newborns and more commonly in Caucasian than in African American babies. Epstein's pearls are located within the fusion of the posterior palatal segments and result from the inclusion of epithelial cells during palatal fusion. Bohn's nodules occur along the junction of the hard and soft palate or adjacent to the mid-palatine raphe and are derived from epithelial remnants of developing palatal salivary glands. The lesions are without symptoms and regress spontaneously within a few weeks to months. Alveolar cysts are small grayish-white or yellow cysts, similar in appearance to palatal cysts. They are also known as dental lamina or gingival cysts of the newborn and are also referred to occasionally as Bohn's nodules or Epstein's pearls. They occur on the crests of the maxillary or mandibular alveolar ridges and on the facial or lingual borders of the ridges. They occur in approximately 25-50% of Caucasian newborns and 11-40% of African American newborns. These asymptomatic lesions regress spontaneously also. Congenital epulis is a rare tumor of the newborn, occurring most commonly on the anterior alveolar ridge of the maxilla. It occurs more commonly in girls than boys (10:1). Size ranges from a few mm to several cm. It is generally a solitary firm, pedunculated lesion, but in 10% of cases, there are more than one. They may lead to difficulty with feeding and/or respiration. Fordyce granules are small slightly elevated yellow spots that may occur in clusters or coalesce to form yellow plaques. They are typically symmetrical on each side of the mouth and occur on the buccal mucosa of the cheeks, the inner surface of the lips, and on the mucosa distal to the mandibular molars. They are not found in the newborn, generally appearing after age 10 and increasing in size and number during puberty. 80-90% of the adult population has these. They are heterotopic collections of sebaceous glands in the oral mucosa and require no treatment. Mucoceles are raised, fluid-filled, well-circumscribed lesions several millimeters to over a centimeter in diameter, usually found in the labial mucosa of the lower lip, less commonly in the upper lip, and rarely on the palate, cheek, or tongue. These occur as a result of trauma to a submucosal salivary duct, usually from blunt trauma or lip biting.

Case A 12-year-old girl, diagnosed using a quick antigen test as having a pharyngeal infection due to Streptococcus pyogenes, also presents with a rash that is on the upper part of the chest and trunk. Question This rash is caused by what product of Streptococcus pyogenes?

Correct answer: Erythrogenic toxin Explanation Erythrogenic toxin (streptococcal pyrogenic exotoxin) is produced by Streptococcus pyogenes. The toxin is responsible for the rash of "scarlet fever". The toxin has been shown to exhibit pyrogenicity and cytotoxicity. It usually appears at the 2nd day of infection, on the upper part of the chest, spreading to the rest of the trunk out towards the rest of the body, with the palms, soles, and face being spared. Exfoliative toxin, produced by Staphylococcus aureus, is responsible for scalded skin syndrome (SSS), wherein the patient appears to have acquired a burn of the skin. There is extensive scaling and flaking desquamation of the epidermis. The syndrome is especially common in infants and small children. Elastase is an extracellular protease that is produced by Pseudomonas aeruginosa. The protease is associated with the organism's virulence due to tissue destruction and bacterial invasion. Elastase is necrotizing to the skin, cornea, and lung; it is capable of producing hemorrhage. Enterotoxin is produced by various bacteria. In Clostridium perfringens, the production of this toxin causes the symptoms of food poisoning. Clostridium perfringens type A is most associated with this toxin production. Enterotoxin is thought to act as a superantigen, which causes a massive release of inflammatory mediators and induces a calcium ion-dependent breakdown of permeability. Exotoxin (diphtheria toxin) is produced by Corynebacterium diphtheriae. The toxin inhibits protein synthesis in mammalian cells, but not bacteria. It affects all cells in the body; the heart, nerves, and kidney are impacted the most. Streptolysin is a streptococcal hemolytic exotoxin. There are two types produced by Streptococcus pyogenes: Streptolysin O (SLO), which is oxygen-labile, and Streptolysin S (SLS), which is oxygen-stable. The production of this hemolytic exotoxin causes the infected host immune response to produce anti-streptolysin O, which can be used to detect Streptococcus pyogenes infections.

Case A 22-year-old man was in your clinic 4 days ago for evaluation of a 101.8° F fever; he was diagnosed with acute pharyngitis. You prescribed Penicillin VK 250 mg TID for 10 days. The patient returns today because his sore throat has worsened. He has not been able to drink fluids, and he has excruciatingly severe pain with swallowing. You recognize the patient speaking with a muffled 'hot potato' voice. Upon re-examination today, you identify right medial deviation of the soft palate with 4+ right tonsillar swelling. Question Ico-delete Highlights What is the most likely diagnosis?

Correct answer: Peritonsillar abscess Explanation The patient scenario above is describing a classic presentation of peritonsillar abscess. This occurs when the acute infection of the pharyngeal area penetrates the tonsillar capsule and then invades the surrounding tissue. This is potentially a complication from acute bacterial pharyngeal infections, which can include streptococcus pharyngitis. These patients will describe a severe sore throat, odynophagia, and trismus. On examination, they will have a medial deviation of the soft palate and peritonsillar fold; they will have an abnormally muffled voice, which is often referred to as a "hot potato" voice. Candidiasis is an incorrect choice. Although this pathology is painful, it presents as creamy-white, curd-like patches on an erythematous base within the mucosa in the oral cavity and/or the oropharynx. Streptococcal pharyngitis (strep throat) is also not the correct choice in this scenario. Patients do presents with a sudden onset of fever, sore throat, pain with swallowing, and tender anterior cervical adenopathy; there is also erythema, edema, and even exudate of the pharynx, soft palate, and tonsils. The edema is much more pronounced on 1 side when a patient has a peritonsillar abscess, to the point that deviation of surrounding structures occur. This is not typical in a patient with strep throat. Mononucleosis has such manifestations as fever, sore throat, fatigue, malaise, anorexia, or myalgias. On physical exam, there may be posterior cervical lymphadenopathy, Hoagland's sign, splenomegaly, or even uvular edema. The patient in this case does not have characteristics consistent with mononucleosis. A tooth abscess would have a more distinct pattern or symptoms and signs that point more towards the teeth being involved. A severe throbbing toothache, sensitivity to hot and cold, and sensitivity to chewing and biting are some of the symptoms that a patient may experience. None of the aforementioned symptoms are consistent with this patient scenario.

Case A 35-year-old woman presents with a 24-hour history of purulent drainage and erythema of her right eye. After a brief physical examination, cultures of the drainage are taken and she is started on a medication prophylactically that would cover the most common bacterial causes of conjunctivitis (including sexually transmitted diseases). Question On what medication (in either an oral or topical form) would she most likely be started?

Correct answer: Erythromycin Explanation Erythromycin ophthalmic ointment applied 2-4 times daily is a treatment option for non-sexually transmitted bacterial conjunctivitis. If trying to cover all bacterial etiologies of conjunctivitis, then erythromycin can be given in the oral form in order to include good coverage for both gonococcal conjunctivitis and chlamydial conjunctivitis. If the erythromycin ophthalmic ointment were to be prescribed in a patient with a sexually transmitted bacterial conjunctivitis, there may still be a partial or complete resolution of symptoms. Erythromycin, in either the topical or oral form, has a good chance of treating any bacterial cause of conjunctivitis until the culture results confirm the etiologic agent. Tetracycline 250 mg po 4 times daily for 3 weeks is a good treatment choice for chlamydial conjunctivitis if the cultures reveal this as the cause, but it would not be a good prophylactic choice while waiting for lab results. Bacitracin ophthalmic ointment applied 2-4 times daily for 5 days is a good treatment option for patients with bacterial conjunctivitis that is not from a sexually transmitted disease. In these cases, the most common etiologic agent is Staphylococcus aureus. Olopatadine is an antihistamine ophthalmic solution that is used in the treatment of ocular itching associated with allergic conjunctivitis. It would not be of any help in a patient with bacterial conjunctivitis, regardless of the etiology. Acyclovir is an antiviral that is prescribed 400 mg po 5 times a day for 7 days in cases of herpetic viral conjunctivitis. It would not be of any help in a patient with bacterial conjunctivitis, regardless of the etiology.

Case A 9-year-old girl presents with a sore throat. The mother states that she began to run a fever a few days ago, and she complained that her throat hurt. On physical exam, you note a red throat, a red and beefy tongue, tonsillar exudates, and swollen anterior cervical lymph nodes. It is noted in the patient's records that she has had a severe anaphylactic reaction to penicillin. Question What antibiotic would treat this infection while minimizing the risk of invoking an allergic reaction?

Correct answer: Erythromycin Explanation The clinical picture is suggestive of a streptococcal bacterial infection. Penicillins are the first-line antibiotics in the treatment of strep pharyngitis. Since the patient is allergic to penicillins, erythromycin is an effective alternative and has no allergic cross-reactivity with the penicillins. Augmentin contains amoxicillin, a member of the penicillin family with allergic cross-reactivity; it is contraindicated in patients allergic to penicillin. Cephalexin, a cephalosporin, can be used to treat strep throat, but approximately 7% of patients who are allergic to penicillin are also allergic to cephalosporins. Compared to erythromycin, use of this agent poses a slightly higher risk of causing an allergic reaction in this patient. Ciprofloxacin is effective against gram-negative organisms. Since streptococcal species are gram-positive, it would not be an appropriate treatment in this scenario. Mupirocin is a topical antibiotic and is not indicated for the treatment of strep throat.

Case During your newborn nursery rounds, a young new mother tells you that there is a family history of eye problems that run on her dad's side. She's not sure what problem it is exactly, but many relatives have had to wear glasses. On exam, the infant's eyes seem to be deviated toward the nose. Corneal light reflex testing confirms your suspected diagnosis. Question Which of the following conditions is most likely present?

Correct answer: Esotropia Explanation The most likely diagnosis is congenital esotropia. This condition causes the eyes to deviate toward the nose, giving the appearance that they are crossed. The corneal light reflex test, also called the Hirschberg test, can reliably diagnose the condition. The corneal light reflex or small white dot on the front of the eye should be in the same position in each pupil. It is normally just slightly nasal to the center of each pupil. If the position is different in each pupil, then some type of strabismus is present. A prism diopter (PD) is the unit measuring the deflection of light passing through a prism equal to a deflection of 1 cm at a distance or 1 meter. Infantile esotropia characteristically presents as a constant, moderate-to-large angle measuring approximately 25 to 60 PD with alternate fixation. Infants presenting at 2 to 4 months of age with constant esotropia of 40 PD or greater are valid candidates for surgical repair. Cataracts present as an opacification or clouding of the lens. Many cataracts in children are actually acquired within the first several years of life. A normal and equal red reflex in each eye will exclude cataracts. This is best done with the room lights dimmed and an ophthalmoscope held about 2-3 feet away from the infant so both pupils can be seen simultaneously, making comparisons easier. Dark lens opacities signify possible cataracts that will eventually produce a gray or white pupillary reflex, or leukocoria. It is then best evaluated by a slit-lamp after pupillary dilation. Glaucoma is a common, progressive disease characterized by elevated intraocular pressure causing progressive damage to the optic nerve that results in atrophy and blindness. This is due to improper development of the eye's aqueous outflow system. Diffuse corneal haze will obscure the pupil and iris markings; in addition, the symptomatic triad of epiphora or excessive tearing, photophobia, and blepharospasm is evident. The corneal light reflex is normal in glaucoma. Retinoblastoma usually presents with leukocoria and strabismus. It can also be detected with an abnormal red reflex test. In the absence of a white pupil, this diagnosis is unlikely in the case presented. Dacryocystitis is an inflammation of bacterial infection of the lacrimal sac. It will present with swelling and erythema of the inner portion of the eye, near the tear duct

Question A 16-year-old boy was at his team basketball practice when a team member forcefully jabbed his fingers into the 16-year-old's left eye while trying to block his shot. He felt sharp, blinding pain and has had much tearing. He was taken out of practice, ice applied, and sent to you for evaluation. Which examination should always be performed initially following direct eye trauma?

Correct answer: Evaluate visual acuity with corrected vision in place Explanation The initial goal of management is to provide timely recognition of injury to allow for management and stabilization of the condition. On the sidelines, this should always begin with an evaluation of visual acuity with corrected vision in place, while taking a thorough history. A deficit in visual acuity unexplained by a previous abnormality or refractive error (astigmatism, hyperopia, myopia, or presbyopia) strongly suggests a serious globe injury. This can be done by having the player read words or letters or count fingers held at a 2-5 foot distance. Painful maneuvers and palpation should be left to the end, such as examining surrounding structures of the eyes, including the orbital bones, eyelids, and medial canthus. Assess the lacrimal drainage system for swelling or lacerations. Topical anesthesia may be required with severe pain or photophobia. The usual mechanism of eye injury is direct trauma. When the eye is struck by an object that is smaller than the orbit, the globe may become compressed. Objects larger than the orbital opening directs force through the skull to the brain that may result in optic nerve, cerebral, or bony injury. Other injuries following optical trauma include corneal abrasions, traumatic rupture of the globe, hyphema or hemorrhage into the anterior chamber of the eye, introduction of a foreign body, lacerations to the eye and surrounding structures, dislocation of the intraocular lens, retinal detachment, and orbital "blow-out" fractures. After transfer to a fully equipped facility, evaluation of intraocular pressure, anterior adnexal, and the interior eye by means of biomicroscopy and ophthalmoscopy should be done. After assessing visual acuity, surrounding structures may be examined, including the globe and sclera, for hyphema as well as corneal defects and lacerations. These findings might further suggest globe injury. Extraocular movements should also be observed for symmetry and nystagmus by having the patient follow a finger in all visual fields. Checking the pupil for size, shape, symmetry, and reaction to light should also be done after visual acuity testing. Sensation over the cheek checks for infraorbital nerve injury that would suggest a blow-out fracture.

Case A 25-year-old man presents with a 1-day history of redness, itching, and swelling in the right upper eyelid. The redness has increased; there has been no eye discharge or watering. On examination, a pustular swelling at the root of the lashes is found. It is surrounded by redness, and it is tender on palpation. The lesion is shown in the image. Question What is the most likely diagnosis?

Correct answer: External hordeolum Explanation There are 2 types of hordeolum. External hordeolum is an acutely inflamed stye caused by an infection of the glands of Zeis (sebaceous glands near the eyelashes) or glands of Moll (apocrine glands of the lid margin). This causes redness, irritation, and itching, followed by a white dot surrounded by erythema. Warm compresses, plus erythromycin cream, is usually effective in most cases. The other type is internal hordeolum, which is a secondary infection of the meibomian glands in the tarsal plate. This is caused by obstruction of the gland orifice, and causes a lid swelling that typically diminishes in size with voluntary squeezing of the eyelid. It may be acutely inflamed and cause diffuse lid swelling and erythema. Commonly, there is remission and exacerbation. The treatment of the acute form is systemic antibiotics, hot fomentation, and topical erythromycin cream; the chronic persistent form usually requires surgical removal and curettage from the conjunctival side. Chalazion is a granuloma of the meibomian gland, and it is usually painless. It may occur spontaneously, or may develop from an internal hordeolum. Blepharitis is inflammation of the lid margin, either due to allergy or infection. It is mostly associated with dry secretions, scales, and redness; there may be itching. Infective cases are treated with antibiotic drops; allergic blepharitis is usually treated with local steroids. Conjunctivitis is inflammation of the conjunctiva and is always associated with red eye and copious eye discharge. Topical antibiotics are used to treat infective conjunctivitis, while steroids and mast cell stabilizers are used to treat the allergic type. Herpes zoster ophthalmicus is an infection due to herpes zoster; it affects the ophthalmic division of the trigeminal nerve. It causes a rash on the upper eyelid that is extremely painful, unilateral, and may affect the cornea causing keratitis which presents with photophobia, lacrimation, and severe pain. It is typically treated with topical antivirals and steroids.

Case A 21-year-old Caucasian man reports symptoms of sneezing, runny nose, itchy nose and eyes, and occasional cough occurring intermittently. His symptoms typically occur following exposure to pollen or pet dander. He describes his symptoms as mild and intermittent; they do not negatively impair his quality of life. Past medical history: Fractured collarbone in childhood. No other significant past medical history. He denies any history of high blood pressure or enlarged prostate. Previous surgeries: None. Medication allergies: None known. Usual home medications: Occasional Tylenol for headache. Social History: The patient is a full-time engineering student. He does not use alcoholic beverages or illicit drugs. Family history: Both parents are alive and in good health. He has one brother who has asthma and eczema. He has no children. Review of systems: Occasional headaches, otherwise noncontributory. Physical Examination: General-Alert white male with dark circles under both eyes. Vital Signs—Temperature 98.4°F, pulse 72 and regular, respirations 12, blood pressure 124/76. HEENT: Normocephalic. EOMs intact. PERRLA. Erythematous injection of the conjunctiva is noted. Pale, boggy nasal mucosa is present. Oral mucosa is pink with a small amount of post-nasal drainage present. Heart: Normal S1 and S2 without rub, murmur, or gallop. Lungs: Clear to auscultation and percussion. No rales, rhonchi, or wheezes. The remainder of the physical examination is unremarkable. Treatment options are discussed with the patient. Patient requests medication that does not make him drowsy, as he wishes to be alert in his classes and when studying for exams. Question What is the best medication choice for this patient?

Correct answer: Fexofenadine (Allegra) Explanation Fexofenadine is the least sedating antihistamine and is the correct answer. In clinical trials, fexofenadine was found to produce no more drowsiness than placebo. Fexofenadine is the only antihistamine that carries no package insert warning regarding sedation, driving, or operating machinery. It is indicated for allergic rhinitis and chronic idiopathic urticaria. Interestingly, fexofenadine is the active metabolite of terfenadine (previously marketed as Seldane), which was removed from the US market in 1997. Unlike terfenadine, fexofenadine is not known to be associated with QT prolongation or cardiac arrhythmias. Diphenhydramine and hydroxyzine are first-generation antihistamines and both are incorrect answers. Early antihistamines (now called "first-generation antihistamines") are non-selective, highly lipophilic, and readily cross the blood-brain barrier. They have antihistaminic, antimuscarinic, anti-α-adrenergic, and anti-serotonin effects. They have prominent side effects of sedation and drowsiness. Of these, diphenhydramine is the most sedating. In addition to its indications for allergic conditions, diphenhydramine is also approved for use as a nighttime sleep aid and, because of its relatively strong anticholinergic activity, for use in motion sickness and parkinsonism. Hydroxyzine has relatively low anticholinergic effects but a strong central sedative effect and is approved for pruritus, anxiety, nausea/vomiting, insomnia, and as a supplement to anesthesia. Cetirizine and loratadine are not the best answers. While they are less sedating than first-generation antihistamines, they both carry warnings regarding drowsiness and sedation, unlike fexofenadine. Second-generation antihistamines are highly selective for the H-1 receptor, lipophobic (rather than lipophilic), and have limited penetration of the blood-brain barrier. They have no anticholinergic effects. As a result, second-generation antihistamines are minimally sedating or nonsedating. Cetirizine, fexofenadine, and loratadine are second-generation antihistamines. All are indicated in the treatment of allergic rhinitis or chronic idiopathic urticaria. Loratadine was one of the earliest non-sedating antihistamines to be developed and released. Cetirizine is an active metabolite of hydroxyzine. Compared to loratadine, cetirizine is more likely to cause sedation but has a faster onset of symptom relief (1 hour rather than 3 hours).

Case A 49-year-old woman presents with a neck mass. On examination, a firm nontender, 3-cm thyroid nodule is felt. TSH, T3, and T4 levels are all normal. Question What is most appropriate next step?

Correct answer: Fine needle aspiration Explanation Fine needle aspiration is the correct response. The patient presents with a thyroid nodule. TSH, T3, and T4 are all normal, which raises the suspicion of thyroid malignancy. Of the above options, fine needle aspiration is the most appropriate next step; it will determine if there are malignant cells present. Levothyroxine treatment is an incorrect response. The patient has normal TSH levels and does not require thyroid replacement. Potassium iodine treatment is an incorrect response. The patient has normal thyroid function studies. Total thyroidectomy is an incorrect response. If the patient's fine needle aspiration yields malignant cells, the patient may require a partial or total thyroidectomy as treatment. At this point, it is unknown if the thyroid nodule contains malignant cells, so a total thyroidectomy would be inappropriate. Observation is an incorrect response. Due to the size of the nodule and the presence of normal thyroid function studies, malignancy must be ruled out.

Case A 25-year-old man is on a skiing vacation. After a few hours of sleep the third night, he awakens with severe bilateral eye pain associated with lacrimation and photophobia. Question What procedure or ocular maneuver would give you the most information to assist in your diagnosis?

Correct answer: Fluorescein staining Explanation The clinical picture is suggestive of ultraviolet keratitis (snow blindness). Slit lamp examination after instillation of fluorescein will show damage to surface cells on the cornea. Visual acuity should be done prior to any testing, but it does nothing to confirm the diagnosis. Testing extraocular movements (EOM) will not confirm the diagnosis. Everting the eyelids is generally done to look at the tarsal plates. Acute angle-closure glaucoma, tested with the Schiotz tonometer, would cause unilateral globe pain and a firm globe.

Case A 17-year-old boy presents with a 1-day history of an extremely painful and pruritic left ear after returning from a weekend trip to the beach. Physical examination of the left ear reveals an erythematous external canal without clear visualization of the tympanic membrane. The patient grimaces and expresses a painful sensation when the left pinna is manipulated. Physical examination of the right ear is benign. Question What is the most appropriate treatment for the above condition?

Correct answer: Fluoroquinolone ear drops Explanation Otitis externa is an inflammation of the external ear canal, usually caused by organisms such as Pseudomonas aeruginosa, S. epidermidis, and S. aureus. In uncomplicated cases, treatment is with oral antianalgesics (e.g., ibuprofen) plus topical agents aimed at cleaning and drying the ear canal and treating the infection. These include 2% acetic acid and fluoroquinolone ear drops. Topical corticosteroids, such as hydrocortisone, can also be used to reduce inflammation. Oral antibiotics are generally used for otitis media, not otitis externa. IV antibiotics covering Pseudomonas are used for malignant otitis externa, as it is a life-threatening condition. An antiviral agent, such as acyclovir, can be used in combination with oral corticosteroids for Ramsey-Hunt Syndrome (herpes zoster affecting the geniculate ganglion). Saline flushes are not helpful in the treatment of otitis externa.

Case A 22-year-old woman presents to her primary care physician. Over the last few weeks, she has had nasal congestion and sneezing. She denies coughing, fever, rhinorrhea, and malaise. Examination reveals pale nasal turbinates. Question What is the most effective treatment in this patient?

Correct answer: Fluticasone spray Explanation Fluticasone spray is correct. Based on the patient's history and the presence of pale nasal turbinates on examination, the patient is suffering from allergic rhinitis. Fluticasone is a corticosteroid spray that decreases nasal mucosal inflammation. Corticosteroid sprays have proven to be more effective than oral antihistamines in the treatment of allergic rhinitis. Oral diphenhydramine and oral loratadine are both incorrect. These agents are anti-histamines. Diphenhydramine is a first generation antihistamine, whereas loratadine is a newer agent that causes less sedation than diphenhydramine. While antihistamines may alleviate symptoms of allergic rhinitis, corticosteroid sprays are more effective. Afrin spray is incorrect. Afrin is a nasal decongestant. The use of nasal decongestants long-term can actually cause rebound congestion, thereby worsening symptoms in the long run. Oral azithromycin is incorrect. The patient does not demonstrate signs of bacterial upper respiratory infection, so antibiotics should not be given.

Question A white male comes into the ER with the complaint of pain in his right eye. He was whittling and felt like something went into his eye. This happened four hours ago. He has washed his eye with water, but it still feels funny. What is your working diagnosis?

Correct answer: Foreign Body Explanation Foreign Body sensation is most commonly due to corneal or conjunctival foreign bodies. Other causes are disturbances of the corneal epithelium and rubbing of eyelashes against the cornea (trichiasis). Blepharitis may be seborrheic or ulcerative. Seborrheic (non-ulcerative) blepharitis is commonly associated with seborrhea of the face, eyebrows, external ears, and scalp. Inflammation of the eyelid margins occurs, with redness, thickening, and often the formation of scales and crusts, or shallow marginal ulcers. Ulcerative blepharitis is caused by bacterial infection (usually staphylococcal) of the lash follicles and the meibomian glands. Chalazion is a mildly painful swelling of the eyelid margin, due to granulomatous inflammation, and usually resolves spontaneously with warm soaks and time. Multiple chalazion may be seen in diabetes, alcoholism, and malnourishment. Curettage may be required in extreme cases. Conjunctivitis can be caused by bacterial, viral, allergic, and irritant etiologies. Clients usually complain of red eyes, and a sticky or watery discharge. The type of discharge helps to determine the etiology-watery in viral, and sticky, green, or yellow, in bacterial conjunctivitis. Irritation is common, but severe pain and photophobia are not. Bacterial or viral conjunctivitis is usually self-limited, but it may be treated with a topical antibiotic, without steroids, such as sulfacetamide (10% 3-4 times/day). Topical aminoglycoside, gentamycin drops, should be reserved for more refractory disease. Allergic conjunctivitis may be effectively treated with a new class of non-steroidal topical, anti-inflammatory agents. Irritant conjunctivitis, including dry eyes, may be treated with topical, non-preserved lubricants. Ectropion is the outward turning of, usually, the lower lid occurring in older people. Surgery is indicated if ectropion causes excessive tearing, exposure, keratitis, or a cosmetic problem.

Case A 17-year-old boy was in your clinic 4 days ago for evaluation of a 101.8°F fever and was diagnosed with acute pharyngitis. You prescribed penicillin VK 250 mg TID for 10 days. The patient returns today because his sore throat is now worse. He has not been able to drink fluids and he has excruciatingly severe pain with swallowing. You recognize the muffled "hot potato" voice. On re-examination, you identify a right medial deviation of the soft palate with a 4+ right tonsillar swelling. Question What is the most likely diagnosis?

Correct answer: Peritonsillar abscess Explanation This patient is clearly suffering from a peritonsillar abscess. This occurs when an active infection penetrates the tonsillar capsule and then involves the surrounding tissue. These patients will have a severe sore throat, odynophagia, trismus, deviation of the soft palate, and an abnormally muffled voice (hot potato voice). Oral candidiasis (or thrush) does not present with the symptoms described in the scenario. Typically, oral candidiasis is painful and appears as creamy-white, curd-like patches; overlying erythematous mucosa can be found virtually anywhere in the oral cavity. The white patches can easily be wiped off when attempted. Laryngitis is lower on the differential diagnosis list because it usually presents with the primary symptom of hoarseness. Laryngitis frequently occurs approximately 1 week after the occurrence of an upper respiratory viral infection that has since resolved. A dental abscess would cause severe, persistent, throbbing toothache, sensitivity to hot and cold temperatures, sensitivity to biting or chewing, fever, possibly noticeable swelling in the face or cheek, or even lymphadenopathy relating back to the site of the abscess. The symptoms of a dental abscess do not match the clinical scenario presented. Mononucleosis also presents somewhat differently from the scenario above, making it a less likely diagnosis. Malaise, fever, sore throat (sometimes exudative), lymphadenopathy, palatal petechiae, and even splenomegaly are found in patients with mononucleosis.

Case An 8-year-old girl presents to the emergency department with her mother due to pain and itching in her right ear. The patient denies hearing loss and dizziness. She just came back from camp yesterday and told her mom that her ear "really itched inside." In attempt to irrigate the right ear, you can see a red object in the ear. Question What is the most likely diagnosis?

Correct answer: Foreign body Explanation The clinical picture is suggestive of a foreign body because the patient complained of pain and itching, and a red object was present in the ear after irrigation. Acoustic neuroma is not correct because the patient would present with ringing in the ear, hearing loss, and dizziness, all of which are not described in the stem of the problem. Barotrauma is not correct because the patient did not undergo any change in pressure to cause a barotrauma. Cerumen impaction is not correct, because although cerumen impaction can cause ear fullness, the patient in the stem of the problem presents with something moving around in the ear. Otitis externa is not correct because the patient does not present with hearing loss, which is associated with swimmer's ear. Additionally, swimmer's ear is not associated with an object in the ear, but is instead caused by water remaining in the ear after swimming.

Case A mother has brought her 3-year-old daughter in to see you because she believes she may have a problem with her sinuses; the mother herself has a history of airborne seasonal allergies. She states that for the past 4 days, the child has been sneezing frequently and has noticeably increased nasal congestion, nasal discharge, and even begun to be extremely irritable. She denies noticing any shortness of breath, problems breathing, or fever; the child has not been in close contact with any sick contacts in the last few weeks. Physical examination reveals unilateral purulent nasal drainage from the left nare, as well as an obvious foul odor; this helps confirm your suspicions. Question What is the most likely diagnosis?

Correct answer: Foreign body Explanation The most likely cause of this patient's signs and symptoms is a foreign body. Pediatric patients are notorious for sticking objects in orifices and places that they do not belong; nose, ears, and mouth being a few examples. Adult patients with psychiatric or behavioral problems may also do this. Common objects include beads, paper, rocks, toy parts, and organic material (peas, corn, seeds, nuts, legumes). Sometimes the action of inserting the item is witnessed, but other times it is not. This is when the history and the physical examination together will reveal if there is a foreign body. A presence of unilateral purulent nasal drainage and a four odor should create a high index of suspicion of a foreign body. It can also create symptoms such as sneezing, epistaxis, or even pain or irritability. A cooperative patient is critical in terms of visualizing and removing any foreign body. Allergic rhinitis would develop nasal congestion, sneezing, rhinorrhea, itchy nose/palate/throat/eyes, and potentially be related to airborne seasonal or perennial allergens. This is not consistent with the patient above. Bacterial rhinosinusitis is a possibility but not as likely. The duration of the chief complaint is the main component ruling this diagnosis out. Typically, this diagnosis is made if a child seems to have viral rhinitis (common cold) that is lasting longer than 10 days, or if it worsens significantly after 5-7 days. Symptoms include nasal congestion, drainage, postnasal drainage, facial pain, headache and fever; some of these symptoms match, but not enough to make this the correct diagnosis. Nasal polyps are benign nasal tumors that are commonly seen in patients with allergic rhinitis. These can result in chronic nasal obstruction and diminished sense of smell, both of which are not consistent with the patient history and physical examination above. Viral rhinitis, or the common cold, is also a possibility but is not correct. Patients will experience clear or mucoid rhinorrhea, but not purulent drainage and/or foul odor. It is possible to also have nasal congestion, sore throat, or even a fever (kids under 6 years old). Both nares will be affected.

Case A 4-year-old boy presents with a 4-day history of frequent sneezing, noticeable congestion, nasal discharge, and irritability; his mother has a history of airborne seasonal allergies and believes her son may have a problem with his sinuses. The mother denies noticing any shortness of breath, problems breathing, or fever; the boy has not been in close contact with sick individuals for the last few weeks. Physical examination reveals unilateral, purulent nasal drainage from the left nare, as well as a foul odor. Question What is the most likely diagnosis?

Correct answer: Foreign body Explanation The most likely cause of this patient's signs and symptoms is a foreign body. Pediatric patients, as well as adult patients with psychiatric or behavioral problems, are notorious for sticking objects in orifices and places that they do not belong (e.g., nose, ears, and mouth). Common objects include beads, paper, rocks, toy parts, and organic material (e.g., peas, corn, seeds, nuts, legumes). Sometimes the patient is witnessed inserting the item, but other times the history and the physical examination together will reveal if there is a foreign body. The presence of unilateral, purulent nasal drainage and a four odor should create a high index of suspicion of a foreign body, which can also create symptoms such as sneezing, epistaxis, pain, and irritability. A cooperative patient is critical in terms of visualizing and removing any foreign body. With allergic rhinitis, a patient will develop nasal congestion, sneezing, rhinorrhea, and itchy nose/palate/throat/eyes; it can potentially be related to airborne seasonal or perennial allergens. Bacterial rhinosinusitis is a possibility, but the duration of the chief complaint is the main component ruling out this diagnosis. Typically, this diagnosis is made if a child seems to have viral rhinitis (common cold) that is lasting longer than 10 days, or if it worsens significantly after 5 - 7 days. Symptoms include nasal congestion, drainage, postnasal drainage, facial pain, headache, and fever. Nasal polyps are benign nasal tumors that are commonly seen in patients with allergic rhinitis. These can result in chronic nasal obstruction and diminished sense of smell, both of which are not consistent with this patient's history and physical examination. Viral rhinitis (common cold) is also a possibility, but patients will experience clear or mucoid rhinorrhea, not purulent drainage and/or foul odor. It is possible to also have nasal congestion, sore throat, or even a fever (childen under 6 years old). Both nares will be affected

Case A 3-year-old girl presents with a 5-day history of purulent, foul-smelling nasal discharge. She is otherwise well-appearing, and her mother reports that she has not had any cough, fever, or other illness symptoms recently. She has been eating and sleeping normally. She is playful and "acting like herself". She stays with a babysitter during the day. Her mother also mentions that the discharge seems to only come from one nare. Question What is the most likely diagnosis?

Correct answer: Foreign body Explanation This young patient has most likely inserted a foreign body into her nose. It is worth noting that the majority of foreign body insertions (71 to 88%) are completely asymptomatic. However, a child with sinusitis would appear more systemically ill than this child does (would probably have a low-grade fever and some appetite loss), while a child with a viral upper respiratory infection (while often well-appearing) would likely have a reported history of cough and congestion, in addition to bilateral nasal discharge (not unilateral). A child with seasonal allergies would have a history similar to that of a viral URI, but might also mention itchy watery eyes and sneezing. Nose-picking is more likely to lead to epistaxis than to purulent discharge

Case A 34-year-old man presents due to something being "wrong" with his left ear. He reports his hearing has been gradually declining, but he recently noticed some discomfort and malodorous discharge draining from this ear. He denies any trauma to the ear and any symptoms in his right ear. Upon further questioning, he admits to some tinnitus and mild vertigo. He otherwise feels well. He denies nasal symptoms, headache, sore throat, and fevers. His past medical history is unremarkable; he has no known medical conditions or history of surgery; he takes no medications and has no allergies. He lives with his wife and 2 children; he works as an office manager, and he denies the use of alcohol, tobacco, and drugs. On physical exam, his vitals are normal. Examination of the left ear reveals mucopurulent drainage within the external auditory canal. The tympanic membrane is disrupted by a retraction pocket within the upper portion, with some thick yellow debris and a polyp protruding from the pocket. Hearing tests are not performed. The right ear reveals mild tympanosclerosis on the tympanic membrane, but it is otherwise normal. The remainder of the patient's exam is normal. Question What risk factor is likely in this patient's history?

Correct answer: Frequent otitis media Explanation This patient is presenting with a cholesteatoma, a benign neoplasm of the tympanic membrane. It is considered a complication of chronic otitis media, so this patient could be expected to report a history of frequent otitis media (OM). (The tympanosclerosis on the right is another clue.) The cholesteatoma is an epidermal inclusion cyst. Complications can include infection, and more significantly, erosion into bone and nerve damage. The mechanism of cholesteatoma is not fully understood, but so far, no link has been associated with chronic use of immune-suppressants. The primary mechanism is neither infectious nor immune-mediated. Likewise, a family history of ear tumor is not linked with development of cholesteatomas. Cholesteatomas can be acquired (most commonly linked with chronic OM) or congenital. The congenital form is thought to arise during fetal ear development and is not hereditary. This patient's cholesteatoma should not be confused with simple otitis externa. Frequent swimming can be a risk factor for otitis externa, which may present with ear pain and discharge (but not the pocket and polyp on the tympanic membrane). Use of alternative medicine ear candling has not been linked with development of cholesteatomas. Ear candling involves the use of a hollow, lit candle held above the ear to create suction. Ear candling has been used for a variety of ear and systemic health issues, but no claims have proven it effective.

Case A 62-year-old woman with a history of breast cancer and rheumatoid arthritis presents with stiff neck and severe headache that started a couple of days ago; symptoms are worsening. Upon awakening the morning of presentation, she felt nauseated and vomited twice. Your neurological examination shows right-sided hemiparesis and dilated and nonreactive left pupil. Question What will be your next step?

Correct answer: Fundoscopic examination Explanation Having a history of breast cancer and signs of possible intracranial hypertension, your patient may have metastatic tumor in the brain. Right-sided hemiparesis and dilated nonreactive pupil on the left point to the compression from the left side. While in the office, she should should be examined for the presence of papilledema. Papilledema will confirm your suspicion that she has transtentorial herniation, in which brain tissue bulges out of the cranium through the tentorial notch because of the increased intracranial pressure. You should schedule Magnetic Resonance Imaging (MRI) of the endocranium. While waiting for the MRI, undertaking a simple fundoscopic examination in the office will contribute to your diagnosis. Lumbar puncture is contraindicated in focal lesions, particularly when you suspect transtrentorial herniation, because by taking a sample of cerebrospinal fluid, brain tissue can press vital centers in the brainstem and cause the death of the patient. Doll's eye maneuver is a test of brainstem integrity and is performed in comatose patients; when the patient's head is quickly moved to one side and then to the other and if the brainstem is intact, the eyes will move conjugately away from the direction of turning and revert back to the midline. Having the history of rheumatoid arthritis and a stiff neck, your patient may have atlantoaxial subluxation, and moving the neck can cause serious damage to the cord. Your patient might have metastases in the lungs and you may think to obtain chest X-ray. However, her symptoms are symptoms of intracranial hypertension, not a process in the lungs.

Case An 18-year-old female college student presents with a 2-day history of severe left ear pain. In the last 5 hours, the pain has become intolerable. Initially, the ear had an intense period of itchiness. Her history is significant for being a member of the college swim team. An examination of the ear canal is remarkable for the presence of edema and redness. A culture swab of the ear canal is performed. The patient is discharged with a course of treatment consisting of polymyxin with a steroid in an acid vehicle, and she is told to return if the symptoms do not subside within the next day. The next day, the microbiology laboratory isolates a gram-negative bacillus; it is oxidase positive and citrate positive. It does not ferment carbohydrates, and it produces a blue-green pigment. Question What is the cause of this patient's external otitis?

Correct answer: Pseudomonas aeruginosa Explanation Pseudomonas aeruginosa is a gram-negative facultative bacillus that grows on MacConkey as a non-lactose fermenter (colonies are not pink). Colonies can appear to have a blue to purple hue due to the production of pigments, and they may also produce a grape-like odor. The organism is oxidase positive, citrate positive, and does not ferment carbohydrates. The organism is a major pathogen and a common cause of nosocomial infections. Because it is a common nosocomial pathogen, they tend to multiply resistant organisms when isolated in the hospital setting. The organism can cause infections in all areas of the body, and these infections can range from mild to life-threatening in severity. Pseudomonas aeruginosa infections of the ear are associated with swimming ("swimmer's ear"), an external otitis. Injury, wet humid conditions, inflammation, and maceration can predispose the external auditory canal to Pseudomonas aeruginosa infections. External otitis due to Pseudomonas aeruginosa is common in humid southern climates. Moraxella catarrhalis is a gram-negative coccus that is relatively plump and occurs in pairs. The organism used to be called Branhamella catarrhalis. Moraxella catarrhalis is oxidase positive, catalase positive, non-motile, oxidation and fermentation glucose negative, and nitrate negative. Growth is good on blood and chocolate agar but variable on MacConkey agar. Moraxella catarrhalis is generally resistant to penicillin due to the overwhelming incidence of B-lactamase production among various strains. The organism is a saprophytic organism of the upper respiratory tract and occasionally of the female genital tract. It is a commonly isolated pathogen in pediatric patients and immunocompromised or debilitated adults (especially in the hospital setting). Moraxella catarrhalis plays an important role in otitis media and sinusitis, as well as nosocomial pneumonia and various other infections. Streptococcus pneumoniae is a gram-positive coccus that occurs in pairs and is "lancet" shaped. The organism is catalase negative, alpha-hemolytic on blood agar, grows best at 35°C with a 5% CO2 atmosphere, is negative for bile esculin as well as failing to grow in 6.5% NaCL, and is bile soluble. Streptococcus pneumoniae is the number one cause of acute otitis media (the second being Haemophilus influenzae, and the third being Moraxella catarrhalis). In children, it is the cause of otitis media in 40-50% of the cases in which an etiological agent is isolated. In adults, it is found to also be the major cause of otitis media. Prior viral respiratory infections are thought to be contributory to the predisposition and development of Streptococcus pneumoniae otitis media due to the congestion of the opening to the eustachian tube. The organism can cause a variety of other infections including bacteremia and meningitis. Haemophilus influenzae is a gram-negative coccobacillus that is small and light staining. It will only grow on chocolate agar due to the organism requiring the presence of growth factors hemin and NAD for growth. In chocolate agar media, the growth factors (found in red blood cells) are released in the media, whereas in blood agar media, the growth factors are trapped within the red cells and thus are unavailable to Haemophilus influenzae. Haemophilus influenzae is the second most common cause of otitis media. Otitis media due to Haemophilus influenzae most commonly occurs between the ages of 6 months and 5 years. The organism can cause a variety of other infections, most importantly meningitis in children as well as sinusitis. Bacteroides fragilis is a gram-negative anaerobic bacillus. Biochemical reactions of significance are growth at 20% bile, indole positive, and resistance to kanamycin/vancomycin/colistin. The organism will only grow under strictly anaerobic conditions. It can be a cause of chronic otitis media infections and is frequently found concomitantly with other organisms such as streptococci, staphylococci, and Pseudomonas aeruginosa.

Case A 48-year-old Caucasian woman with multiple comorbidities presents with worsening hearing loss and tinnitus in her right ear. She states this first began about 3 months ago and was initially bearable; it has now progressed to where she cannot hear anything out of her right ear, and the tinnitus is unrelenting and constant. The patient is worried because she is now experiencing balance and coordination issues. An MRI is ordered on the patient, and it reveals the following results. Question Considering the diagnosis, as well as the fact that this patient is a poor surgical candidate, what would be a reasonable treatment option at this time?

Correct answer: Gamma knife radiosurgery Explanation Vestibular schwannoma, which also sometimes is referred to as an acoustic neuroma, are one of the most common intracranial tumors encountered in clinical practice. More than likely these occur as unilateral lesions; only rarely can these occur as bilateral masses. Although growth of these lesions is overall slow, the increased size can eventually cause such symptoms as unilateral hearing loss and deterioration of speech discrimination. Tinnitus will also be seen in these patients and as the tumor increases in size more central nervous system components are affected. This will cause loss of balance, coordination, vertigo, facial numbness, facial weakness, or even dysphagia. Typically these lesions are diagnosed via MRI or even a CT scan. Treatment options usually are initially centered on surgical removal; however, our patient is not a candidate for a surgical procedure at this time. Other options would be radiosurgery, or specifically Gamma Knife radiosurgery is recommended. Gamma Knife radiosurgery is seen as an acceptable alternative for microsurgery for non-surgical candidates with similar tumor control rates to those having the surgical intervention. Linear accelerator radiation therapy is another type of radiotherapy used to treat cancers; however, this type is not indicated in the treatment of a vestibular schwannomas. Proton therapy, or proton beam therapy, is yet another type of radiotherapy; however, it is not a type that is specifically used in the treatment of schwannomas. Observation only would not be appropriate for this patient currently as she is having significant symptoms currently that are interfering with her every day and quality of life. Chemotherapy is not used for treating schwannomas.

Question A 35-year-old man presents with what he describes as a "weird-looking tongue." He denies any soreness, tenderness, or recent injury to the tongue. He is not on any medications. On exam, the tongue has erythematous areas that are smooth and appear to be without papillae. There are also areas that have not been denuded and are still rough to the touch. No lesions, white patches/areas, or ulcerations are appreciated, and the tongue protrudes symmetrically. What is the most likely diagnosis?

Correct answer: Geographic tongue Explanation A geographic tongue is a benign condition with unknown cause; it is characterized by a map-like pattern of smooth red areas that do not have papillae as well as rough areas that still have papillae. Atrophic glossitis (or smooth tongue) presents as having a smooth surface due to papillae loss. The loss may indicate deficiency in riboflavin, niacin, folic acid, vitamin B12, pyridoxine, or iron. Candidiasis is an infection that may cause the tongue to have a white coating, which can be scraped; a sample can be analyzed for the presence of Candida. Hairy leukoplakia may be seen in people infected with HIV and AIDS. It is characterized by raised areas that are whitish-tan in color that have a feathery appearance. It is different from candidiasis of the tongue in that hairy leukoplakia cannot be scraped off. A "hairy tongue" is not actually due to hair growth on the tongue; it consists of elongated papillae that have the appearance of grayish-black hair to the naked eye. This condition may be caused by antibiotic use, or there may not be any reason.

Case A 20-year-old woman presents with a 3-day history of fever, sore throat, and enlarged glands in her neck. She denies any cough or runny nose, but she has malaise, body aches, and headaches. She has no other medical problems and does not take any medications. She works in a daycare center; she takes care of children 3 to 4 years of age. On examination, she has a temperature of 101.5° F; pulse is 102/min, and BP 110/70 mm Hg. Oral exam reveals swollen tonsils with plenty of exudates. There is no nasal congestion, and lungs are clear. Cervical lymph nodes are enlarged bilaterally and tender. Abdomen is unremarkable. Throat swab is obtained by the physician. Question What organism is most likely to be isolated in culture of the swab?

Correct answer: Group A streptococcus Explanation Based solely on the clinical features, the patient is suffering from acute streptococcal pharyngitis, which is characterized by fever, sore throat, exudates, and cervical adenopathy without cough or rhinorrhea. Group A streptococcus is the most likely causative organism among the choices. It is especially seen in children or adults exposed to children in schools, daycare centers, etc. Treatment with penicillin is recommended if rapid antigen test in the office is positive, the throat culture comes back positive, or the patient has all four clinical criteria, including fever, exudates, cervical lymphadenopathy, and absence of cough. Corynebacterium diphtheriae causes diphtheria and is characterized by pharyngitis, low-grade fever, malaise, and a gray membrane over the pharyngeal area that bleeds on stripping. Throat swab has to be cultured on a specific medium since regular media will not grow this bacterium. Treatment is with penicillin or erythromycin. The Epstein-Barr virus causes infectious mononucleosis that is characterized by fever, respiratory congestion, splenomegaly, hepatomegaly, severe fatigue, as well as anterior and posterior cervical adenopathy. Definitive diagnosis is by noting atypical lymphocytes in the peripheral smear and heterophile antibody test. Serology is needed only in a few cases. Treatment is supportive because antibiotics are useless. Haemophilus influenzae is a rare cause of pharyngitis and causes lower respiratory tract infections; it is usually found in smokers. Group C streptococcus can cause similar symptoms, but it is much less prevalent than group A streptococcus.

Case A 16-year-old girl presents with throat pain. Her sore throat was first noticed when she woke up this morning. The patient has a temperature 102 degrees F. On examination, the pharyngeal mucosa is erythematous, with yellow exudate. She has tender anterior cervical lymphadenopathy. Question What organism is the most likely cause of her condition?

Correct answer: Group A streptococcus Explanation Group A streptococcus is the correct response. Group A streptococcus is the most common cause of bacterial pharyngitis. The patient has presence of fever, exudate and tender anterior cervical adenopathy, which are all highly suggestive of bacterial pharyngitis. Corynebacterium diphtheriae is an incorrect response. Corynebacterium diphtheriae is an uncommon cause of sore throat. It would present with a gray pseudomembrane on examination of the throat. Adenovirus and rhinovirus are incorrect responses. Viral pharyngitis does not typically present with exudate or tender anterior cervical adenopathy. Candida albicans is an incorrect response. Candida albicans of the throat typically presents with thin exudate, but it is not common unless a patient is immunocompromised or has had recent antibiotic treatment.

Case A 42-year-old man presents for evaluation of a growth on his tongue. He thinks the lesion has been present for a few months, and it has not changed; however, he generally prefers to avoid healthcare, and he has not been concerned. He is only here at the urging of his family member. The patient denies oral symptoms and changes in taste sensation; he states that he generally feels fine. The patient denies the use of chew tobacco and cigarettes. On physical exam, there is a white patch of tissue, which does not scrape off; there is a "shaggy" appearance on the left lateral tongue. No erythema is noted. No other lesions are identified. The remainder of his exam is normal. A biopsy of the lesion is obtained. The pathology shows hyperkeratosis and "balloon" cells in the upper cell layer. Refer to the image. Question What additional test should be ordered on this patient?

Correct answer: HIV Elisa Explanation This patient has oral hairy leukoplakia, which is often associated with HIV-infection or other immunocompromised states (such as post-transplant). It is rare in immunocompetent individuals. Therefore, this patient should have an HIV Elisa test in order to screen for HIV. The oral hairy leukoplakia is a benign neoplasm of the tongue; in and of itself, it is not of great significance. However, due to its association with an immunocompromised state, it can be a harbinger of a more significant condition. A C-reactive protein test is a non-specific indicator of inflammation. Oral hairy leukoplakia is not associated with elevations in serum inflammatory markers. A fungal culture may be considered based on the initial appearance, which resembles thrush. However, the biopsy is consistent with oral hairy leukoplakia and not Candida. No further testing for fungal organisms is needed. Heterophile antibodies are helpful in the diagnosis of infectious mononucleosis as a result of the Epstein-Barr virus (EBV). The oral hairy leukoplakia is caused by EBV, but it is not associated with mononucleosis. Testing for heterophile antibodies is not helpful. An MRI of the head and neck would be appropriate if the patient presented with a malignant oral growth, suspicious lymph nodes, and/or a history suspicious for metastasis. He had none of these. The oral hairy leukoplakia is benign and does not metastasize.

Case A 37-year-old man presents with headache, malaise, and nasal congestion. He reports his symptoms started with what he believed to be a common cold, which occurred about 2 weeks ago. However, he has begun feeling much worse over the last 5 days. He describes severe facial pain when he bends to tie his shoes. He admits a very mild nonproductive cough, but he denies shortness of breath, fever, and chills. He has tried multiple over-the-counter cold remedies without relief. His past medical history is unremarkable, with no known medical conditions, no history of surgery, no regular medications and no allergies. He works as a welder, but denies known occupational respiratory exposures. He denies use of alcohol, tobacco, and drugs. On physical exam, his temperature is mildly elevated at 99.9 °F (37.7 °C), and other vitals are normal. His voice has a nasal quality and an occasional mild cough is noted throughout the exam. On HEENT exam, he demonstrates some tenderness over the left frontal region. Green nasal discharge with boggy nasal mucosa is present bilaterally. Mild injection is seen in the posterior nasopharynx. The remainder of his exam is unremarkable. Question Ico-delete Highlights What organism is most likely implicated with this patient's current condition?

Correct answer: Haemophilus influenzae Explanation This patient presents with a classic acute bacterial rhinosinusitis (sinus infection). The most common causative organisms are Streptococcus pneumonia, Haemophilus influenzae, and Moraxella catarrhalis. Viruses and anaerobes can also be implicated as causes. Haemophilus influenzae is a Gram-negative bacterium. It is important for clinicians to understand the basic underlying etiology of common infections in order to select appropriate empiric antibiotic therapy. Because of difficulty in obtaining sinus cultures without nasal contamination, culture and identification of sinusitis-causing organisms is rarely done in clinical settings. Candida albicans is a common fungal organism. It is typically responsible for vaginal yeast infections and thrush. It does not usually cause sinusitis. Clostridium difficile is a Gram-positive, anaerobic bacillus. It can be a causative organism in diarrheal illnesses and colitis, and it is often considered a nosocomial (hospital-acquired) infection. It is not linked with sinusitis. Escherichia coli is a Gram-negative bacterium commonly found in the gastrointestinal tract. It is part of the normal GI flora, but can contribute to infection in other systems, such as the urinary tract. It is not usually found in the sinuses. Francisella tularensis is a Gram-negative bacterium that causes tularemia (also known as "rabbit fever"). It is carried by various vectors (ticks, rabbits, and rodents). Tularemia can cause various systemic symptoms, such as fever, malaise, fatigue, aches, and swollen lymph nodes. Pseudomonas aeruginosa is a Gram-negative bacterium that can be found in infections throughout several body systems. It can cause pneumonia, skin infections, as well as gastrointestinal and urinary tract infections. However, it is typically associated with some type of inoculation or immunocompromising condition (e.g., burns, post-catheter, puncture wounds, neutropenia, ventilator use, etc.). This patient history does not suggest any unusual etiology for his sinusitis. Pseudomonas aeruginosa is an unusual and rare cause of sinus infection.

Question A 12-year-old female presents to the office with an acute sore throat. The patient is experiencing great pain when swallowing. Even the swallowing of her own saliva causes intense pain. An examination of the throat reveals an enlarged and erythematous epiglottis. A special throat culture request was made in consideration to the finding of acute epiglottitis. The patient was discharged and treated with ampicillin. The next day the culture was significant for 4+ of a gram-negative coccobacillus that only grew on chocolate agar media. The cause of this acute epiglottitis is:

Correct answer: Haemophilus influenzae Explanation Streptococcus pyogenes is a gram-positive coccus, catalase negative, beta hemolytic on blood agar, appearing as chains on gram stain. Definitive identification to distinguish it from other beta hemolytic streptococci is the detection of its specific "A" antigen by latex agglutination techniques. It is associated with streptococcal pharyngitis, scarlet fever, streptococcal pyoderma, necrotizing fascitis, and streptococcal toxic shock syndrome. Bacteremia is uncommon. They are universally sensitive to penicillin. Streptococcus pneumoniae is a gram-positive lancet-shaped coccus that is catalase negative and occurs in pairs. It is a common cause of otitis media in children. It is also a major cause of meningitis in elderly people and especially those that have underlying conditions, are malnourished, or are alcoholics. The organism is alpha hemolytic on blood agar. Staphylococcus aureus is a gram-positive coccus, catalase positive, and coagulase positive, predominantly beta hemolytic on blood agar, appearing in characteristic grape clusters on gram stain. Staphylococcus aureus can cause a variety of infections. In children with reactive tonsils, this can be a source of infection, leading to severe tonsillitis. A semisynthetic penicillin is the treatment of choice. Haemophilus influenza is a gram-negative coccobacillus. It is also a major cause of meningitis. It occurs mostly in young infants and children where it can also cause a severe epiglottitis that can necessitate intubation. When it occurs in adults, it is usually due to an underlying condition, such as paranasal sinusitis, remote head trauma, or otitis. The organism will not grow on blood agar and requires the presence of growth factors (hemin and NAD) for growth. Candida albicans is a yeast. Yeast appears on gram stain as large gram-positive organisms approximately 3-5 times larger than gram-positive cocci. They are aerobic and generally grow well on most nonselective agar media. The organism is a major cause of throat infections in the immunocompromised, such as patients with HIV. When causing an infection in the throat, it is called "thrush."

Question A 32-year-old new male patient presents for a routine physical exam. The patient confides that he is HIV positive; he has not sought any medical help. During examination of his oral cavity, raised areas that are whitish-tan in color with feathery appearance on his tongue are noted. Attempts to scrape the area off for sampling are unsuccessful. What is the most likely diagnosis?

Correct answer: Hairy leukoplakia Explanation Hairy leukoplakia may be seen in people infected with HIV and AIDS. It is characterized by raised areas that are whitish-tan in color and have a feathery appearance. It is different from candidiasis of the tongue in that hairy leukoplakia cannot be scraped off. A "hairy tongue" is not actually due to hair growth on the tongue; it consists of elongated papillae that have the appearance of grayish-black hair to the naked eye. This condition may be caused by antibiotic use, or there may not be any reason. Atrophic glossitis (or smooth tongue) presents as having a smooth surface due to papillae loss. The loss may indicate deficiency in riboflavin, niacin, folic acid, vitamin B12, pyridoxine, or iron. A geographic tongue is a benign condition with an unknown cause; it is characterized by a map-like pattern of smooth red areas that do not have papillae as well as rough areas that still have papillae. Although candidiasis is also common with people with AIDS and the infection also causes the tongue to have a white coating; the coat can be scraped and a sample can be analyzed for the presence of Candida.

Case An 8-year-old boy in the 3rd grade is referred to you by his school doctor to be evaluated for poor speaking and reading ability, failure to follow directions in class, and classroom disruptiveness. Despite these problems, he appears to be alert and interactive with other children and there is no demonstration of aggressive behavior or rage. He does not appear to be preoccupied with internal stimuli, and IQ testing results are within normal range. Question What is the most likely cause of his symptoms?

Correct answer: Hearing impairment Explanation Hearing impairment is the most likely cause for this boy's poor communication and reading ability and his classroom problems. Children with hearing impairment appear to learn more slowly because they miss many important cues and information. They often become frustrated and develop other behavioral disturbances such as classroom disruptiveness. Thus, sensory impairment is an important consideration in the differential diagnosis of any child with symptoms that might suggest MR/ID or learning and communication disabilities. The 4 most common causes of speech or language delay are developmental language disabilities (i.e., normal cognition (IQ), impaired intelligibility, and delayed emergence of phrases, sentences, and grammatical markers), intellectual disability, hearing impairment, and autistic spectrum disorders. That being said, research has shown that approximately one-third of hearing impaired children will also be found to have at least 1 other disability that affects the development of speech and language (e.g., intellectual disability, cerebral palsy, craniofacial anomalies). Any child who shows developmental warning signs of a speech or language problem should have a hearing assessment by an audiologist and an examination by a geneticist as part of a comprehensive evaluation. Autism is unlikely in this case because the boy is interactive with classmates and his environment. Reading disorder (or "Developmental Dyslexia") is the most common learning disability. The clinical picture is consistent with a learning disability in that the IQ score is higher than academic performance would suggest (poor speaking and reading ability, failure to follow directions). However, before this boy can be given a diagnosis of a learning disability, hearing impairment must be ruled out with audiometric studies. Childhood schizophrenia is a rare condition that can cause learning disabilities and is characterized by inattention and disruptive behavior; however, it is unlikely in this case, as there appears to be no impairment in social skills, no preoccupations with internal stimuli, and no aggression or rage behavior. Seizure disorders (epilepsy) can cause cognitive disturbances, but these should be detectable with IQ testing. The boy, in this case, has a normal IQ. Although a topic of much discussion and debate, it is well known that chronic epilepsy has an association with neuropsychological impairment.

Question A 2-year-old girl is brought to an otolaryngologist by her mother for chronic ear infections. The patient is otherwise healthy, with the exception of recurrent episodes of otitis media (OM). Examination and history show that the child has had average growth and development; she has not had invasive infections, skin disorders, or hospitalization. The child's mother is concerned about the risk of hearing loss and its effects on development. What statement about hearing loss and OM is most accurate?

Correct answer: Hearing loss during OM may adversely affect cognition and language Explanation Conductive and sensorineural hearing loss are complications of chronic otitis media (OM). Acute and chronic suppurative OM usually results in conductive hearing loss. Chronic infection may result in conductive hearing loss from a perforation of the tympanic membrane; however, sensorineural hearing loss can occur, especially when herpes zoster is the etiologic agent. Cholesteatoma increases the probability of labyrinthitis, which carries a high risk for sensorineural hearing loss. Van der Hoeve syndrome is a constellation of symptoms including hearing loss, but is unrelated to OM. Hearing impairment is a risk factor for impaired speech and language development, particularly if it occurs early in life. In cases where hearing loss due to chronic OM is reversed surgically, it is likely that young children will compensate and catch up to peers. In cases where OM is either undiagnosed or untreated, long-term developmental and social problems may result. Because otitis media is often associated with hearing loss, most clinicians have been eager to treat the condition to restore hearing to normal, thereby preventing any long-term problems.

Case A woman presents with an otherwise healthy 12-month-old boy because she noticed that his eyes that appear to be crossed as well as each one being different colors. The mother sheepishly admits to smoking for about 10 years and she smoked during her entire pregnancy. She is worried because she had a brother die when he was 3 because he had a tumor in both his eyes. Physical exam reveals heterochromia iridis and leukocoria bilaterally, as well as evidence of severe strabismus. Question What is the main risk factor for the most likely diagnosis of this child?

Correct answer: Heredity Explanation The correct response is heredity. The child most likely has inherited retinoblastoma. Retinoblastoma is a rapidly developing cancer that generally affects children under 6. It is most commonly diagnosed in children 1-2 years old. Genetic counseling is especially important when more than one family member has had the disease or if the retinoblastoma occurs in both eyes. Leukocoria is seen as a whitish color behind the pupil, which is usually black. It is a sensitive test best done by looking at the "red reflex." Normally, red reflection occurs in people's eyes when taking flash photographs. Dimming the room lights and using a flashlight to shine light directly into the child's eyes can also elicit the red reflex. With leukocoria, also known as "cat's eye," red reflex is absent. This abnormality is present in approximately 60% of all children with retinoblastoma. Keeping in mind that retinoblastoma is the third most common cancer overall affecting children, red reflex is a useful screening tool. It is a sign and not a risk factor for the disease. Although smoking can affect pregnancy and a child, in this particular case, heredity is probably the main risk factor. Heterochromia iridis is a relatively late symptom caused by the tumor invasion and/or neovascularization. It is a symptom, not a risk factor. Crossed eyes, or strabismus, which occurs as a result of visual loss, is a common sign of retinoblastoma. That is the reason funduscopic examination through a well-dilated pupil must be performed in all cases of childhood strabismus. Strabismus is usually secondary to macular involvement. It is also a sign and not a risk factor for the disease.

Case A 19-month-old boy presents with a 3-day history of fever, irritability, and poor feeding. His mother noticed white patches on his tongue yesterday. He also began to drool, and she can hardly get him to eat anything. His past history is significant for recurrent ear infections. He had tympanostomy tubes placed about 1 month ago. On examination, he is febrile with a temperature of 39.8C (103.6F). He appears unwell and is uncooperative and irritable when touched. He is noted to be drooling and has several shallow ulcers on his tongue and palate. The ulcers have a yellowish base with surrounding erythema. There are no lesions on any other areas of his body. Question What is the most likely diagnosis?

Correct answer: Herpetic gingivostomatitis Explanation Herpetic gingivostomatitis is characterized by a prodrome of flu-like symptoms such as fever and irritability followed by vesicles that coalesce and rupture to form painful ulcers of the oral and perioral tissues. The pain can be severe, and refusal to eat or drink is a clue to the diagnosis. There may be accompanying cervical lymphadenopathy. Diagnosis is usually clinical, but viral cultures, PCR, or serology can be done if the diagnosis is in doubt. Thrush is caused by overgrowth of candida albicans and is characterized by white plaques, not ulcers. Herpangina is usually caused by group A coxsackie virus and characteristically involves the soft palate and tonsillar fauces. It may be accompanied by fever and sore throat. It usually affects children 3-10 years old. Treatment is symptomatic. Apthous ulcers are painful shallow ulcers with a similar appearance to herpetic ulcers, but they are not accompanied by fever and the patient is otherwise not ill. They are self-limited and no treatment is usually necessary. Topical steroids may be used if needed. Hand, foot, and mouth syndrome is caused by coxsackie viruses. It is characterized by fever and oral vesicles on the palate, tongue, and buccal mucosa, which progress to ulcers. Vesicles are also noted on the palms and soles and sometimes on the buttocks. Treatment is symptomatic.

Case A 20-month-old boy presents with a 1-week history of fever up to 104°F and irritability. Four days prior to his visit, his mother noted sores in his mouth; she states that she has noted him to be drooling and that his appetite is quite diminished. His past medical history is unremarkable. He has no medical allergies and his only current medication is acetaminophen. He is current on his immunizations. His physical exam reveals normal vital signs except for a temperature of 103.5°F. On examination of his oral cavity, you note swollen, erythematous gingiva with ulcerations present. The ulcerations appear yellowish-white and friable. White-gray lesions approximately 3 mm in diameter are seen on the anterior tongue. The tonsils appear erythematous without exudates. His lips are slightly cracked, and his mucous membranes are slightly tacky. Neck examination reveals bilateral anterior cervical adenopathy. He has no skin lesions. The remainder of his exam is normal. Question What is the most likely cause of this patient's condition?

Correct answer: Herpetic gingivostomatitis Explanation Herpetic gingivostomatitis, caused by herpes simplex virus type 1, is the most common cause of stomatitis in children 1 - 3 years of age. Symptoms may appear abruptly, with high fever, drooling, fetid breath, and refusal to eat, as noted in the above vignette. However, the fever may precede the oral lesions by 2 - 3 days and presage to a more insidious onset of the disease. The tongue, cheeks, and gingiva are most commonly affected, but the entire oral cavity may be involved. These areas can present with ulcers that are yellowish-gray in color, and the gingiva may be quite friable. Drooling may be present secondary to the pain associated with chewing and swallowing, and dehydration is a real concern in the management of the patient. Cervical and submaxillary adenitis is common. The acute phase may last up to 1 - 2 weeks. Treatment consists of measures to relieve the pain and facilitate the intake of fluids for adequate hydration. Oral candidiasis (thrush) typically presents in the infant period and is usually caused by the yeast Candida albicans. This common affliction presents with white curd-like plaques on the oropharyngeal mucosa and tongue. Scraping the plaques may reveal an erythematous base. Treatment is usually accomplished with oral nystatin. It may be seen in older infants and children on antibiotic treatment or with immunodeficiencies. Herpangina is usually associated with a prodrome of fever, headache, and occasional emesis. Lesions are characteristically 1 - 3 mm in diameter, present as vesicles and ulcers, and are yellow-white in color. Each lesion is surrounded by an erythematous halo up to 10 mm in diameter. The lesions are typically found on the anterior tonsillar pillars, as well as the uvula, soft palate, and tonsils. The anterior mouth is rarely affected. The illness is caused by members of the Enterovirus family and affected children do not appear as toxic as those that have herpes gingivostomatitis. Treatment is supportive and the acute phase lasts 3 - 6 days. Nursing bottle caries is relatively common, and it is seen in patients that sleep with a bottle in their mouth. These children present with significant erosion of the enamel of the anterior dentition. Treatment typically consists of extraction of severely affected teeth to prevent pain and spread of infection to contiguous tissues. Impaction of a foreign body would lead to pain, edema, and erythema of only a localized portion of the gingivae and not the widespread inflammation noted in the vignette. Difficult cases should be referred to a dentist for further treatment.

Case A 68-year-old man with a 50-pack/year smoking history presents to his primary care provider complaining of a 4-month-history of progressive dysphagia. His review of symptoms is notable for intermittent ear pain, especially upon swallowing, an involuntary weight loss of 12 pounds over the past 4 weeks, and occasional hemoptysis. He denies chills, abdominal pain, shortness of breath, chest pain, vomiting, or skin changes. His physical exam is remarkable for nontender, immobile lymphadenopathy of the cervical nodes, and stridor upon auscultation of the trachea. A nasopharyngoscope revealed the following image. Question Which of the following additional findings is most likely in this patient?

Correct answer: Hoarseness Explanation This patient is demonstrating signs and symptoms consistent with laryngeal cancer. In the accompanying image, cancer along the right vocal cord, causing hoarseness and sore throat, is evident. Squamous cell carcinoma of the larynx, the most common malignancy of the larynx, occurs almost exclusively in patients with a history of significant tobacco use. A change in voice quality is most often the presenting complaint; new and persistent hoarseness of more than 2 weeks duration in a smoker is typical. Other manifestations, including persistent throat or ear pain, especially with swallowing, hemoptysis, lymphadenopathy, a neck mass, dysphagia, weight loss, and airway compromise with stridor, may occur. Weight gain is unlikely with laryngeal cancer. Tonsillar exudates and tympanic membrane abnormalities are not expected in laryngeal carcinoma and suggest infectious etiologies. Fever and posterior pharyngeal edema are physical exam findings more commonly associated with a retropharyngeal abscess; this process is infectious and of an acute progression.

Case A 33-year-old man presents with a 1-day history of a painful left upper eyelid. He denies any change in vision, discharge, trauma, or foreign body. The pain started after the patient was cleaning out the garage. On physical exam, the visual acuity is OD/OS/OU = 20/20. The lateral aspect of the left upper eyelid is swollen, erythematous, and tender to palpation. The rest of the eye exam is normal. Question What is the most likely diagnosis?

Correct answer: Hordeolum Explanation A hordeolum is an acute localized red, swollen, and tender area on the upper or lower eyelid. A chalazion is a chronic granulomatous inflammation of a meibomian gland that is characterized by a hard, nontender swelling of the upper or lower eyelid. It is usually not erythematous. Blepharitis is a chronic bilateral inflammation condition of the eyelids. Dacryocystitis is an infection of the lacrimal sac located in the nasolacrimal area. Uveitis is an intraocular inflammation characterized by photophobia, blurred vision, and moderate pain in the eye.

Case A 43-year-old woman presents to a walk-in clinic with a complaint of a painful, red lump on her left eyelid. Examination shows an oozing erythematous pustule surrounded by hyperemia edema. Question What is the most likely diagnosis?

Correct answer: Hordeolum Explanation Hordeolum is an inflamed sebaceous gland near a hair follicle which appears as a red, painful pustule and is sometimes known as a sty. Hordeola are usually self-limited, spontaneously improving in 1-2 weeks. Medical therapy for hordeola includes eyelid hygiene, warm compresses and massages of the lesions for 10 minutes 4 times per day, and topical antibiotic ointment in the inferior fornix if the lesion is draining. Chalazion is a granuloma of a meibomian gland and often causes lid protrusion. Pterygium is an extension of the pinguecula over the limbus toward the center of the cornea. It is neither painful nor red. Interstitial keratitis is a cloudy, painful, cornea caused most commonly by syphilis, though it has also been associated with tuberculosis. Staphyloma is protrusion of the sclera near the cornea resulting from injury to the sclera or increased intraocular pressure.

Case An 8-year-old child is brought to your office because of swelling of the left upper eyelid; the swelling is associated with redness and tolerable pain. No fever is noted. Physical examination shows a localized swelling and redness on the upper middle lid of the left eye; there is slight tenderness on palpation. Vital signs are within normal limits. Question Ico-delete Highlights What is the most likely diagnosis?

Correct answer: Hordeolum Explanation The clinical picture is suggestive of hordeolum, which is an infection of the lid glands. The most common causative agent is Staphylococcus aureus, which may either be acute or subacute. When the meibomian glands are infected it is referred to as internal hordeolum, wherein the lesion tends to be large and extend to the skin or conjunctival surface. If it affects the glands of Zeis and Moll, it is referred to as an external hordeolum or stye. It is smaller, more superficial, and points to the lid margins. Treatment, like any abscess, is warm compresses and surgical drainage (if needed). Topical antibiotics may also sometimes be used. If left untreated, it may progress to cellulitis of the lid or orbit, which requires systemic antibiotics. Recurrence is also frequent, and children with recurrent styes should be evaluated for an immunologic problem. Blepharitis is an inflammation of the lid margins characterized by redness and a scaling or crusting lesion. It is initially manifested by itching, irritation, and burning sensation. It is recurrent, chronic, and usually bilateral. In cases of the seborrheic type, the scales are greasy, erythema is less, and ulceration seldom occurs. In cases of the staphylococcal type, ulceration is common; lashes may fall out, and it is often accompanied by conjunctivitis and superficial keratitis. Most of the blepharitis is of mixed type. Application of antistaphylococcal agent or sulfonamides directly to the lids daily is the treatment of choice. Daily cleaning of the lid with a moist cotton applicator to remove scales and crusts is very helpful. Chalazion is an inflammation of the meibomian glands characterized by a firm, non-tender nodule on the upper eyelid. It differs from hordeolum because it does not have the presence of inflammatory signs. Excision is recommended if the nodule is large enough to cause astigmatism by exerting pressure on the globe. Some cases subside spontaneously. Entropion is a condition in which the lid margin is directed inwards. It usually causes discomfort and corneal damage because the eyelashes are also turned inwards. It is most commonly caused by scarring due to inflammation seen in trachoma; it may also result from Steven-Johnson syndrome. Surgery is effective. Ectropion is the opposite of entropion, in which the lid margin is turned outwards or everted; it is associated with an overflow of tears, maceration of the lid skin, inflammation of exposed conjunctiva, and/or superficial exposure keratopathy. Scarring from inflammation, burns, trauma, or orbicularis muscle weakness from facial palsy are the common causes. Surgical correction is necessary to protect the cornea.

Case A 35-year-old woman presents with a painful swelling of her left eyelid. On physical exam, there is tenderness to palpation and erythematous swelling present on the lid margin involving the eyelashes. Question What is the most likely diagnosis?

Correct answer: Hordeolum Explanation The correct answer is hordeolum (or stye), which is an infection that occurs at the lid margin. It is most often caused by bacteria and is treated with warm compresses and sometimes antibiotic ointment. An internal hordeolum is an infection of the meibomian gland that expands onto the lid conjunctiva. An external hordeolum is typically smaller than an internal one and found on the lid margin. Xanthelasma is a raised yellowish plaque in the skin of the eyelids. It is not painful and is classically associated with high cholesterol. A chalazion is a chronic inflammation of the meibomian gland inside the eyelid, not the lid margin, and is characteristically hard and nontender. Pinguecula is a nodule in the bulbar conjunctiva. Dacryocystitis is an inflammation of the lacrimal sac that leads to pain, swelling, and erythema around the tear sac.

Case A patient has recurrent aphthous ulcers. He wants to know if there is any medication that can help him. Question What are you most likely going to prescribe?

Correct answer: Hydrocortisone Explanation Hydrocortisone hemisuccinate pellets is a topical anti-inflammatory used to help speed healing of an aphthous ulcer and are popular because it does not cause significant adrenal suppression. It does not, however, stop the ulcers from recurring. Acyclovir is used to treat herpes labialis. Mupirocin is used to treat impetigo. Colchicine is used to treat Behcet's disease. Methotrexate can cause Aphthous Ulcer.

Case A 17-year-old baseball player presents to the clinic after being struck in the eye with a baseball. On examination, you note bright red blood in the anterior chamber. Question What is your initial diagnosis?

Correct answer: Hyphema Explanation The clinical picture is suggestive of a hyphema. Hyphema is defined as hemorrhage into the anterior chamber. Patients with a corneal abrasion note severe pain and photophobia following a history of a traumatic event to the affected eye. Pinguecula is a yellow, elevated conjunctival nodule; it is commonly located on the nasal side of the eye. Most retinal detachments occur spontaneously, and blood in the anterior chamber is not found. Hypopyon is described as pus (white and cloudy fluid) in the anterior chamber; it usually follows a fungal infection.

Case A 16-year-old girl has just been diagnosed with severe allergic rhinitis caused by ragweed and dust mite. She is a candidate for allergy immunotherapy, which will involve weekly subcutaneous delivery of the offending allergens in increasing concentrations. Question What is the ultimate goal of this type of immunotherapy for this patient?

Correct answer: Hyposensitization Explanation Allergy injections are a type of immunotherapy that is also known as hyposensitization. Exposure to a gradually increasing amount of allergen results in various cellular effects that lead to a decrease in the production of mast cells by the immune system. It does not result in complete immunity and does not suppress the immune system but rather decreases the reactivity of the immune system. This treatment does not serve to treat infections, as the condition that it treats is not infectious in nature.

Case A 30-month-old boy presents with what his parents describe as "chronic ear infections." The patient appears otherwise healthy with the exception of recurrent episodes of otitis media (OM). The boy's parents are concerned about sequelae, and they are anxious about taking time off work. They ask if their son can be referred to an otolaryngologist for tympanostomy tubes. Question What criterion would be the best indicator of the need for tympanostomy?

Correct answer: Impending or actual complication of OM Explanation The correct response is impending or actual complication of OM. A child with otitis media (OM), if referred to an otolaryngologist, will be extensively assessed. Evaluation should include anatomic and pathologic findings, speech and language development, and hearing. Summary of indications for tympanostomy tubes: Refractory infection with moderate to severe symptoms Unresponsiveness to at least 2 antibiotics Hearing loss of 20-30 dB or worse with effusion = 3 months Impending or actual complications (mastoiditis, labyrinthitis, etc.) Persistent infections (4-6 months) Advanced middle ear disease (e.g., cholesteatoma) Craniofacial anomalies that predispose to middle ear dysfunction

Case Ico-delete Highlights A 28-year-old man presented 4 days ago for evaluation of a 101.8° F fever; he was diagnosed with acute pharyngitis. You prescribed penicillin VK 250 mg TID for 10 days. The patient returns today because his sore throat has worsened. He has not been able to drink fluids or eat, and he has excruciatingly severe pain when swallowing. You recognize the patient speaking with a muffled "hot potato" voice. Upon re-examination today, you identify a right medial deviation of the soft palate with 4+ right tonsillar swelling. Question Ico-delete Highlights Based on the most likely diagnosis, what is next step in treatment?

Correct answer: Incision and drainage Explanation The patient scenario above is describing a classic This occurs when the acute infection of the pharyngeal area penetrates the tonsillar capsule and then invades the surrounding tissue. This is potentially a complication from acute bacterial pharyngeal infections, which can include streptococcus pharyngitis. These patients will describe a severe sore throat, odynophagia, trismus, and upon examination will be noted to have a medial deviation of the soft palate and peritonsillar fold, as well as have an abnormally muffled voice, often referred to as a "hot potato" voice. The correct response is aspiration via incision and drainage (I&D) or needle aspiration. This provides immediate symptoms improvement. The abscess is localized and blunt dissection is done to break loculations. The incision it is left open to drain. After cultures are obtained from the abscess, antibiotic therapy should begin. Because of the severity of symptoms this patient is presenting with, the treatment for this condition at this point usually will consist of intravenous amoxicillin (1 g), amoxicillin-sublactam (3 g), or clindamycin (600 - 900 mg). Intravenous antivirals would not be an appropriate choice, as the organisms that are more likely to cause this include aerobic (Streptococcus pyogenes, Staphylococcus aureus, Haemophilus influenzae, Neisseria species) and anaerobic (Fusobacterium, Peptostreptococcus, Prevotell, and Bacteroides) organisms. Observation and reassurance is not appropriate due to the substantial signs and symptoms the patient is currently displaying. Immediate tonsillectomy is not a recommended plan of action for acute treatment of a peritonsillar abscess. Experts have concluded that it is safest to complete a tonsillectomy approximately 3 to 6 months after the abscess has resolved in patients who have either recurrent tonsillitis or peritonsillar abscess. There is no indication of this being a recurrent event in this patient.

Case A 25-year-old woman presents with watery eyes and a runny nose. The symptoms get worse after she has been outside, especially if she plays in the grass with her 2-year-old. Allergy testing reveals that she is highly allergic to several grass pollens. Allergy shots using grass pollens as the antigens might be helpful. Question Ico-delete Highlights What mechanism describes how allergy shots help to reduce allergy symptoms?

Correct answer: Increased production of allergy-specific IgG Explanation The correct response is increased production of allergy-specific IgG. Allergy shots are given to individuals who have an allergic reaction to common allergens (e.g., mold or pollen from grasses, ragweed, and trees). A small amount of the allergen is injected into the patient and the body starts making antibodies to the allergen; this allows the body to fight the allergen and relieves the symptoms of the allergic reaction. The body's main response is increased production of IgE-blocking antibodies and allergy-specific IgG. They also increase IL-10 production, which has anti-inflammatory effects. Allergy shots do not decrease IgE or IgA production. They also do not degrade allergy-specific antibodies.

Case Ico-delete Highlights A 1.5-year-old boy presents with a squint in the left eye. His mother informed you that the child's eyes were quite normal until about 2 months ago when she noticed asymmetric movements of her son's eyes. She also felt that the child could not see properly with his left eye. There is no history of trauma to the eye. The child was born at full term and his growth and development have been within normal limits. On exam, the pupils are equal in size. There is loss of vision in the left eye and a convergent squint in the same eye. Fundus examination showed absence of red reflex in the left eye, and instead a white pupillary reflex (leukocoria) was seen. X-ray of the skull shows calcification within the globe. Question Ico-delete Highlights What is the most likely diagnosis?

Correct answer: Retinoblastoma Explanation The most likely diagnosis is retinoblastoma, as it is the most common primary ocular tumor in children below 5 years of age. 90% of cases are diagnosed below 3 - 4 years of age. The index case is a 1.5-year-old boy who has presented with a recent appearance of squint and absence of normal red reflex in the left eye, replaced instead by a white pupillary reflex (leukocoria). This is due to reflection of light from the white-colored tumor and loss of vision in that eye. The diagnosis is further supported by calcification seen in the globe in the X-ray of the skull. Fundoscopy may show the tumor as a white mass, which may be small and flat or may be large and protuberant. Orbital inflammation, hyphema, and irregular pupil are seen in advanced stages of the disease. Retinoblastoma gene is a recessive gene located on the chromosome13 at the 13q 14 regions, and the tumor may arise from any of the nucleated layers of the retina. Retinal detachment in infants and children more commonly occurs due to trauma, secondary to other abnormalities like myopia, or after cataract surgery. It can also occur in diabetes, sickle cell disease, and retinopathy of prematurity. Presenting signs can be loss of vision, secondary strabismus (squint), nystagmus, and leukocoria (white pupillary reflex). Calcification seen on an X-ray of the skull in retinoblastoma is absent in retinal detachment. Also ultrasonography and neuroimaging may be required to establish the cause of detachment. Congenital glaucoma (elevated intraocular pressure) usually manifests during the first 3 years of life. The classical triad of symptoms of congenital glaucoma are epiphora (excessive lachrymation), photophobia (sensitivity to light), and blepharospasm (squeezing of the eyelids). These symptoms are due to corneal irritation. As the cornea and sclera are more elastic during early childhood, the elevated intraocular pressure therefore leads to expansion of the eyeball, including the cornea, and development of buphthalmos (ox eye), which means a large eye. This leads to corneal edema and conjunctival congestion. The cornea may become cloudy. There is no white pupillary reflex or calcification in the globe seen on an X-ray of the skull. A cataract is an opacity in the lens and may cause significant impairment of vision. It may be an isolated defect or may be a part of a generalized disorder. Common causes are intra uterine infections like rubella, cytomegalovirus infection, toxoplasmosis, metabolic disorders like galactosemia, and chromosomal disorders like trisomy 13, 18, and 21. Trauma to the eyeball is a major cause of cataract in children. The red reflex may be absent or may be irregular or there may be a white pupillary reflex. The retina and the blood vessels may not be visualized due to the lenticular opacities. Nystagmus may be present. Poor visual fixation, squint, and poor social smile may be seen later on. Calcification in the globe is not present in cataract. Persistent hyperplastic primary vitreous (PHPV) is caused by persistence of portions of the fetal hyloid vascular system and the associated fibromuscular tissue. The condition is usually unilateral, and the affected eye is smaller than normal. The anterior chamber is shallow, and the lens is also smaller than normal. Other presenting signs are white pupillary reflex (leukocoria) strabismus and nystagmus. The course is progressive and outcome is poor.

Case A 25-year-old woman presents due to constant sneezing episodes that have progressively worsened over the last few months. Further questioning reveals she also suffers from severe bilateral eye irritation, excessive tearing, and pruritus. On physical examination, you note hypertrophic nasal mucosa that is boggy in appearance, with the turbinates appearing pale. The patient states that this is especially bothersome during spring months; however, it also occurs to some degree during the fall. Question Based on the history and physical examination findings, what is the most appropriate first-line clinical intervention at this time?

Correct answer: Intranasal corticosteroids Explanation The scenario above is describing a common presentation of allergic rhinitis, most likely leaning more towards the seasonal rhinitis. Based on this diagnosis, the most appropriate first-line treatment is intranasal corticosteroids (ICS). The use of ICS for treatment of allergic rhinitis has essentially been a revolution; evidence-based reviews have found ICS to be much more effective and much less expensive than oral antihistamines. They are also less sedating. Key components of ICS therapy that must be addressed with all patients when prescribing them include appropriate training on usage as well as educating the patient that it will typically take 2 or more weeks to note any improvement. Oral antihistamines are a viable alternative for treatment of allergic rhinitis, but they should not be considered first line. If the patient is unable to tolerate ICS or proper usage cannot be executed, oral antihistamines could potentially be initiated. Oral steroids and oral decongestants are not indicated for the treatment of allergic rhinitis. Intranasal decongestants should not be prescribed to any patients who are diagnosed with allergic rhinitis, and they should be discouraged in general. These over the counter nasal sprays will lead to a condition known as rhinitis medicamentosa after around 5 - 7 days of treatment; a characteristic of this condition is severe rebound nasal congestion. Increased and longer use of intranasal decongestants will further worsen this condition, creating a viscous and endless cycle.

Case A 22-year-old woman presents due to mild year-round symptoms of nasal blockage, sneezing, rhinorrhea, and excessive tearing. She reports a family history of allergies to various substances. A nasal smear reveals many eosinophils. Immediate skins tests show positive reactions to house dust, grass, and ragweed pollens. Question What is the most appropriate therapeutic intervention?

Correct answer: Intranasal corticosteroids Explanation This patient has signs and symptoms of perennial allergic rhinitis. Intranasal corticosteroids is the correct response. Perennial rhinitis is a type 1 hypersensitivity reaction. Patients are symptomatic throughout the year and usually present with chronic nasal obstruction, rhinorrhea, and excessive lacrimation. Sinusitis, nasal polyps, or aspirin sensitivity may complicate it. On examination, the nasal mucosa may be edematous and is usually pale or bluish in color with thin nasal secretions. Nasal polyps may be visualized. Conjunctival findings include injection and swelling with occasional chemosis. The diagnosis is made by a positive history of atopic disease, the characteristic bluish red nasal mucosa and numerous eosinophils in a nasal smear stained with Wright stain. Positive skin tests help to identify the particular allergen. Perennial allergens include: House dust mites Epidermal antigens produced by pets such as cats and dogs Indoor molds, such as spores of Aspergillus and Penicillium Occupational allergens, such as platinum salts and wood dust Cockroach skin casts Treatment should be based on the patient's age and severity of symptoms. Patients should be advised to avoid known allergens and be educated about their condition. Intranasal corticosteroids are the most effective treatment and should be first-line therapy for mild to moderate disease. Moderate-to-severe disease not responsive to intranasal corticosteroids should be treated with second-line therapies, including antihistamines, decongestants, cromolyn, leukotriene receptor antagonists, and nonpharmacologic therapies (e.g., nasal irrigation). With the exception of cetirizine, second-generation antihistamines are less likely to cause sedation and impair performance. Immunotherapy should be considered in patients with a less than adequate response to usual treatments. Antigen avoidance is most effective when a single agent is responsible for the symptoms. When complete avoidance is impossible (as with house dust), exposure may be reduced by such measures as wet mopping and dusting frequently. Environmental measures, such as air conditioners, only reduce indoor mold and pollen counts but would not be the most effective measure for this patient who is allergic to house dust, grass, and ragweed pollen. Immunotherapy might be indicated if a patient continues to experience clinically significant symptoms after appropriate environmental measures, antigen avoidance, and pharmacologic measures have been taken. This is done by injecting an extract of the allergen subcutaneously in gradually increasing doses.

Question A previously healthy 13-year-old boy presents with a 2-day history of sore throat and fever. On examination, his temperature is 102.4°F. He demonstrates a 'hot potato voice', and he is drooling. Examination of the throat is initially difficult because of trismus, but reveals a fluctuant left tonsil that is displaced medially, with erythema and edema of the soft palate. The uvula is deviated to the right. Tender cervical adenopathy is noted on the left. What is the most appropriate treatment option?

Correct answer: Intravenous antibiotics and surgical consultation Explanation A peritonsillar abscess results from a complication of acute tonsillitis. Usual etiologic agents include group A streptococci, Staphylococcus aureus, Streptococcus pneumoniae, and oral anaerobes. The infections have a rapid onset, and they are accompanied by fever, severe sore throat, trismus, drooling, alterations in speech and dysphagia. The tonsil is unilaterally displaced medially with erythema and edema of the soft palate. Uvular deviation may be deviated to the opposite side. The infection is usually unilateral, but can be seen bilaterally in up to 10% of cases. Therapy consists of intravenous antibiotics and surgical drainage of the abscess. Some surgeons may opt for serial aspirations of the abscess. Occasionally the infection may be seen early in the course and intravenous antibiotics are all that is required. However, this patient has frank abscess formation and, therefore, would require surgery. Oral or intramuscular antibiotics are not indicated initially. Throat culture may help guide therapy, but should never be a replacement for hospitalization and surgical consultation. Complications include upper airway obstruction, aspiration pneumonia due to rupture of the abscess, and spread to the retropharyngeal or mediastinal spaces. Tonsillectomy may be done in an elective basis 3 to 4 weeks after the inflammation resolves. Acute tonsillectomy may be performed for significant airway obstruction or other complications.

Case A 40-year-old man presents with burning and pain of his oral cavity; the burning and pain have been associated with a pruritic rash of the flexor aspect of his left wrist. He denies a history of smoking, drinking, or illicit drug use. The physical exam is remarkable for violaceous, shiny, and polygonal papules that are arranged as lines and circles on his wrist. These papules range in size from 1 mm to 1 cm in diameter, and they have fine, white lines on them. In the oral cavity, a reticular, white, lacy pattern is visualized, as seen in the image. Question What is correct regarding this condition?

Correct answer: It is a cell-mediated immune response that is associated with hepatitis C and primary biliary cirrhosis Explanation This patient most likely has lichen planus with cutaneous and oral involvement. It is an inflammatory mucocutaneous condition that usually exhibits a distinctive morphology; it is associated with hepatitis C virus infection, chronic active hepatitis, and primary biliary cirrhosis. The classic appearance of skin lesions includes violaceous polygonal flat-topped papules and plaques, commonly occurring at t the wrist. Wickham's striae may also be found; these are white lines found within papules. No imaging studies are necessary for lichen planus. Lichen planus is not a fungal disease process; therefore, antifungal agents are not appropriate treatment. Unlike leukoplakia or erythroplakia, the oral finding in this patient is not a premalignant finding. Nutritional deficiencies commonly contribute to angular chelitis and atrophic glossitis, not lichen planus.

Case A 12-year-old boy presents with fatigue and jaundice. History obtained from the patient and his mother is negative for recent illness, fever, infectious exposures, medication, alcohol, or drug use. He denies gastrointestinal (GI) symptoms and a history of GI disease. On physical examination, he appears ill; the liver edge is palpable and slightly tender. Skin and sclera are icteric, and there is corneal discoloration. On eye examination using a slit-lamp, you note brown-yellow rings encircling the iris in the rim of the cornea bilaterally. You order a serum ceruloplasmin level, which is reported as low. Question What is this diagnostic corneal pigmentation known as?

Correct answer: Kayser-Fleischer rings Explanation Kayser-Fleischer rings are the result of the accumulation of copper in the cornea; they are the most unique sign of Wilson's disease. Wilson's disease is an inherited disorder of copper toxicity due to a genetic defect in copper transport. Beginning at birth, copper is not secreted into the bile or incorporated into the copper protein ceruloplasmin, resulting in low serum levels of ceruloplasmin. Symptoms and signs develop age 5-40 as copper accumulates in the liver, brain, cornea, kidney, and reproductive organs. 50% of patients present with hepatitis, 40% present with neurological manifestations (tremor, speech disorders, dysphagia, incoordination), and 5-10% first present with Kayser-Fleischer rings, amenorrhea, miscarriages, or hematuria. Diagnosis is confirmed by Kayser-Fleischer rings on slit lamp examination in the presence of a low serum ceruloplasmin. AST and ALT levels are often elevated; serum copper is low; 24-hour urinary copper excretion is elevated. Treatment is lifelong chelation or oral zinc and a low copper diet. Keratoconus is a bulging of the cornea to form a cone, and the classic sign is Fleischer's rings, which are iron-colored rings surrounding the cone. This progressive bulge is due to a weakness in the cornea and often occurs bilaterally at age 10-20. There are frequent changes in visual acuity, necessitating repeated prescription changes; contacts provide better correction than glasses. Corneal transplant may be necessary if corrective lenses are not adequate. Arcus juvenilis is a gray or white arc around the peripheral cornea similar to arcus senilis in adults. It occurs in younger adults and is often associated with high blood cholesterol. A metallic foreign body lodged in the cornea can quickly result in a single small-diameter rust ring that requires ophthalmologic intervention with a rust ring drill for removal. A pinguecula is a raised, yellowish discoloration on the bulbar conjunctiva at the 3 o'clock or 9 o'clock position of the scleral-corneal junction. It is a benign growth caused by an accumulation of conjunctival tissue that can be the result of chronic actinic irritation.

Case Ico-delete Highlights A 56-year-old man presents with a history of persistent and progressive unrelenting hoarseness for the last few months. He is a 50 pack-year smoker but quit 1 year ago. Physical examination demonstrated a 2-cm firm non-tender right anterior cervical lymph node. Question What is the most likely diagnosis?

Correct answer: Laryngeal cancer Explanation The correct answer is laryngeal cancer. Tobacco abuse is a common predisposing factor in laryngeal cancer and affects men more often than women. Persistent hoarseness in this population should cause suspicion of cancer. Many patients with laryngeal cancer present with palpable anterior cervical lymphadenopathy. Acute laryngitis lasts for about 1 week and typically follows a viral infection. Chronic laryngitis—often due to irritants, vocal abuse, or gastroesophageal reflux—does not typically have accompanying non-tender lymphadenopathy. Thyroid cancer may present with anterior cervical lymphadenopathy but is rarely seen with progressive hoarseness. Vocal cord nodules are typically found in patients who overuse their voices and is not related to tobacco abuse. Strep pharyngitis typically causes tender cervical lymphadenopathy unrelated to tobacco abuse and does not cause progressive laryngitis.

Case A 70-year-old man with a 50-pack/year smoking history presents to his primary care provider complaining of a 4-month history of progressive dysphagia and hoarseness. His review of symptoms is notable for intermittent ear pain, especially upon swallowing, an involuntary weight loss of 12 pounds over the past 4 weeks, and occasional hemoptysis. He denies fever, chills, abdominal pain, shortness of breath, chest pain, vomiting, or skin changes. His physical exam is remarkable for nontender, immobile lymphadenopathy of the cervical nodes and stridor upon auscultation of the trachea. He is referred for a CT scan of the neck, which reveals the following image. Question What is the next most appropriate diagnostic test in the definitive diagnosis of this patient?

Correct answer: Laryngoscopy Explanation This patient is demonstrating signs and symptoms consistent laryngeal cancer. The CT scan image reveals tumoral involvement of the right vocal cord. In the accompanying image, cancer along the right vocal cord, causing hoarseness and sore throat, is evident. Direct laryngoscopy also provides an opportunity for biopsies of the tumor to be obtained. When coupled with appropriate imaging, such as a CT scan, the direct laryngoscopy provides the best information for tumor staging and surgical planning. A pharyngeal culture is of no diagnostic value in the assessment of this patient's presentation and CT-scan findings. A culture may be useful in the evaluation of infectious etiologies. Pulmonary function tests are necessary before one decides whether the patient is a suitable candidate for radical surgery that involves airway function. Plain films of the chest may be useful in planning surgery. If metastases are present in the chest, the therapeutic decision tree changes entirely. However, chest CT or PET-CT are more sensitive for metastasis that plain films. Tracheal sonogram may be used in the initial assessment of a neck mass; however, it is unable to definitively diagnosis laryngeal cancer.

Case A 70-year-old man presents with paralytic strabismus with maximal esotropia as he gazes to the left. Question What nerve is most likely affected in this case?

Correct answer: Left 6th cranial nerve Explanation The correct answer is the left 6th cranial nerve. This nerve abducts the eyeball and paralysis of this nerve results in the inability of the left eye to gaze laterally, causing maximal esotropia with left gaze. The right 6th cranial nerve will cause the same symptoms but as the patient gazes to the right. Cranial nerves III and IV are responsible for other movements of the eyeball, such as the ability to gaze upward and downward as well as pupil reaction to light and accomodation, which will not result in the defects noted in this patient.

Case A 42-year-old man presents with a 10-day history of worsening headache, stuffy nose, greenish nasal discharge, and a low grade fever. He has body aches and facial pain, as well as a dry cough. He denies shortness of breath, abdominal pain, nausea, or vomiting. He is a non-smoker, has no significant past medical history, and is only taking acetaminophen. On exam, he has a temperature of 100.9° F taken orally. Pulse is 86/min, BP is 120/76 mm Hg left arm sitting, and SPO2 is 94% on room air. Lungs are clear and abdomen normal. Nasal mucosa appears boggy, and there is tenderness with palpation over the facial bones (maxillary area). Pharynx is without exudates. Question What component of the history prompts you to consider giving antibiotics for treatment of this condition?

Correct answer: Length of time the symptoms have been present Explanation The condition being described in this clinical scenario is highly indicative of acute bacterial rhinosinusitis (acute sinusitis). Acute sinusitis clinically is described as including symptoms such as green/yellow purulent appearing discharge, nasal obstruction, congestion, facial pain, or pressure over the affected sinus, and may also include cough, malaise, fever, or even headache. Acute sinusitis has an acute onset of symptoms, ranging from 1-4 weeks in length of duration by the time the patient presents clinically. More commonly, the origin of sinusitis is viral; however, symptoms relating to this will resolve as time passes, not intensify or worsen.

Case A 60-year-old man presents with bad mouth odor that has been present for months. He states that he has smoked 2 packs per day for the last 20 years and has chronic bronchitis with colorless sputum. He recently visited a dentist, who assured the absence of caries and sent him to be examined for a patch (please see image), which can't be rubbed or removed by cotton tipped applicator. Question Ico-delete Highlights What is the most likely diagnosis?

Correct answer: Leukoplakia Explanation Leukoplakia is a white patch on the tongue or oral mucosa that cannot be removed by rubbing. It is usually found in heavy smokers and those with chronic irritation, such as patients with bad teeth or alcoholism. It is premalignant and consists of dysplastic cells and hyperkeratosis. Erythroplakia is similar but has definite erythematous component. Lichen planus occurs as lacy or erosive lesions on the buccal mucosa. Oral candidiasis presents similarly to leukoplakia (white patches on the mucosa); however, it can be removed, leaving a bleeding surface. Hairy leukoplakia occurs in patients with symptomatic AIDS, usually in the lateral aspect of tongue, as a white corrugated patch; it is sometimes bilateral.

Question A 66-year-old male presents to the clinic with a complaint of not being able to hear the beeping of his microwave oven when it signals completion of it's heating cycle. Knowing that the "beeping" is high pitched and the age of the individual, you suspect hearing loss in this patient that is typically associated with aging. This type of hearing loss is related to which one of the following alterations in the ear? Answer Choices 1 Fibrosis of the tympanic membrane 2 Hypersecretion of cerumen in the external auditory meatus 3 Ankylosis of the stapes at the oval window 4 Loss of cochlear hair cells 5 Loss of otoconia in the otolithic membrane

Correct answer: Loss of cochlear hair cells Explanation Hearing loss may be the result of one of two basic problems. Auditory disorders may be related to either conductive disorders, or sensorineural disorders. Conductive disorders are those that result from the mechanical impedance of sound waves from reaching the auditory sensory receptors. Sensorineural disorders are those that result from the loss of the ability to transduce or convey the mechanical signal into the neural signal. Fibrosis of the tympanic membrane, excessive secretion of cerumen in the external auditory meatus or ankylosis (bone deposition) of the stapes at the oval window are all examples of conductive disorders leading to hearing loss.Furthermore, conductive disorders such as these would result in a clinical situation with the loss of sound at all frequencies, rather than only a high frequency or selected frequency. Loss of the cochlear hair cells, particularly at the beginning of the basal turn of the cochlea, typically result in the loss of high frequency sounds. This is due to a sensorineural disorder which results in the loss of a specific frequency due to inability to transduce or convey the mechanical signal to a neural signal. This selective hearing loss of high frequency sounds, such as that of a beeping microwave oven, can be associated with hearing disorders during the process of aging.Loss of neurons from the spiral ganglion would be another example of a sensorineural disorder. The loss of otoconia in the otolithic membrane would probably have little effect on auditory responses.

Case A 55-year-old woman presents with episodic vertigo, tinnitus, hearing loss, and ear fullness. Her ear and eye physical examination are unremarkable. You perform a Dix-Hallpike maneuver which is negative. There are no carotid bruits noted on auscultation. Question Ico-delete Highlights What is the best initial recommendation to make?

Correct answer: Low sodium diet Explanation The clinical picture is suggestive of Meniere's disease. The classic syndrome consists of episodic vertigo lasting 20 minutes to several hours associated with fluctuating low-frequency sensorineural hearing loss, tinnitus, and a sensation of aural pressure. The Dix-Hallpike maneuver is a diagnostic maneuver for benign paroxysmal positional vertigo. Treatment involves a low sodium diet and diuretics. Restricting water intake will lead to dehydration and an increase in sodium levels, worsening the symptoms of Meniere's disease. Diazepam can be used for Meniere's disease but is usually used for severe vertigo. The question is indicating initial treatment. Diazepam is used in the treatment of vestibular neuronitis. In vestibular neuronitis, a paroxysmal, usually single attack of vertigo occurs without accompanying impairment of auditory functions and will persist for several days to weeks before clearing. Examination reveals nystagmus. These symptoms are not present in this patient. Antibiotics are not indicated for Meniere's disease. Fluticasone propionate is an anti-inflammatory nasal spray used to treat the nasal symptoms of indoor and outdoor nasal allergies and year-round nonallergic nasal symptoms. Fluticasone helps reduce the inflammation that leads to nasal symptoms that include congestion, sneezing, and itchy, runny nose which are not indicated in this patient.

Case A 29-year-old woman presents with slowly progressive right-sided hearing loss, tinnitus, and continuous vertigo. Her Weber test reveals lateralization to the left ear. Question What is the recommended next step?

Correct answer: MRI Explanation MRI is the correct response. The presentation of slowly progressive sensorineural hearing loss, vertigo, and tinnitus is suggestive of acoustic neuroma, which is a benign tumor on the 8th cranial nerve. Diagnosis is made through MRI. Observation is an incorrect response. While acoustic neuroma is a benign tumor, its growth can impinge on vital structures. MRI should be obtained in this patient. Tympanostomy tube placement is an incorrect response. Tympanostomy tubes are placed for recurrent otitis media. Angiography is an incorrect response. Angiography has no role in this patient's diagnosis. Referral for hearing aid is an incorrect response. While the patient is suffering from hearing loss, the cause should be investigated before referring her for a hearing aid.

Case A 58-year-old Caucasian man presents due to a growing mole on his face. The mole is located on his left cheek and has been present for the past several years but has appeared bigger and darker in the last 3 months. The patient denies any other moles with the same characteristics and just wants it taken care of so it is not as bothersome. The patient denies weight loss, night sweats, or fevers; he has no recent changes in his appetite or sleeping issues. He is a farmer and owns more than 100 acres that he plants and harvests yearly and has for the past 40 years. On physical examination, you utilize the ABCD rule and find a 7.2 mm, asymmetrical, irregular bordered, varying colored, mostly flat lesion with evidence of ulceration. Question Given the history and physical exam findings, what is the most likely diagnosis for this patient?

Correct answer: Malignant melanoma Explanation The patient in this clinical case scenario most likely has Malignant Melanoma of the skin. It is commonly described as being a flat or raised pigmented lesion. The mnemonic of the "ABCD" rule is what is used to further aid in screening suspicious lesions: Asymmetry, Border irregularity, Color variegation, and Diameter > 6 cm. It is suggested that an "E" could be added to this widely used and accepted mnemonic, which stands for Evolution. The history of evolution or the history of a changing mole is the single most important historical reason for exquisitely close evaluation and even prompt referral. Bleeding and ulceration of these lesions are also considered extremely ominous signs. Basal cell carcinoma (BCC) is not the correct choice. BCC clinically presents in four or so different clinical types, the nodular form being the most common (making up 60% of the BCC cases). Nodular BCC is described as a pink or flesh-colored papule that is pearly or translucent with evidence of a telangiectatic vessel within the papule. Ulceration is common and is sometimes referred to as a "rodent ulcer." The other types of BCC include superficial, morpheaform, other subtypes, and even several BCC syndromes. Squamous cell carcinoma of the skin typically appears as small, red, conical, hard nodules that occasionally will ulcerate. The presence of a pearly appearance helps distinguish BCC from squamous cell, although the two malignancies present in the same patterns. Benign nevi are common, but any skin lesion that has an ulcer and tendency to bleed should be urgently evaluated to rule out the worst-case scenario. Actinic keratosis is also an incorrect choice. These are generally small (0.2 cm-0.6 cm) macules or papules that could be flesh color, pink, or even slightly hyperpigmented; however, they will feel like sandpaper and are generally tender when palpated. The clinical scenario does not match this description.

Case A 32-year-old man presents with a 1-week history of fever and pain over his cheeks, which radiate to his teeth and are provoked by bending forwards. He also has right-sided facial pain, nasal congestion and discharge with hyposmia, and postnasal drip with a persistent, nonproductive cough. His physical examination was remarkable for a febrile patient with purulent nasal and posterior pharyngeal secretions, tenderness overlying the right sinus, air-fluid levels on transillumination of the right sinus, periorbital edema, mucosal, and facial erythema. A facial x-ray in Waters view displayed the following image. Question Ico-delete Highlights Which of the following is correct regarding this patient's condition?

Correct answer: Mechanical obstruction of ostia and impaired ciliary function are known risk factors. Explanation This patient most likely has rhinosinusitis. Obstruction of the natural sinus ostia prevents normal mucus drainage. Retained mucus, when infected, leads to sinusitis. The ostia can be blocked by mucosal swelling or local causes (trauma, rhinitis), as well as by certain inflammation-associated systemic and immune disorders (such as Sarcoidosis, Wegener's granulomatosis AIDS, diabetes, and chemotherapy). Systemic diseases that result in decreased mucociliary clearance, including cystic fibrosis, respiratory allergies, and primary ciliary dyskinesia (Kartagener syndrome), can be predisposing factors for acute sinusitis. Any mass lesion with the nasal air passages and sinuses, such as polyps, foreign bodies, tumors, and mucosal swelling from rhinitis, may block the ostia and predispose to retained secretions and subsequent infection. Mechanical obstruction because of nasal polyps, foreign bodies, deviated septa, or tumors can also lead to ostial blockage. Conditions that impair ciliary function include Kartagener syndrome, exposure to bacterial toxins, or exposure to cold air. The vast majority of rhinosinusitis episodes are caused by viral infection. The most common pathogens isolated from maxillary sinus cultures in patients with acute bacterial rhinosinusitis include Streptococcus pneumoniae, Haemophilus influenzae, and Moraxella catarrhalis Streptococcus pyogenes, Staphylococcus aureus. Anaerobes are less commonly associated with acute bacterial rhinosinusitis; they have been found in fewer than 10% of patients with acute bacterial sinusitis, despite the ample environment available for their growth. Women have more episodes of infective sinusitis than men because they tend to have more close contact with young children. The rate in women is 20.3%, compared with 11.5% in men. Sinusitis is more common from early fall to early spring. Radiographic findings in patients with acute sinusitis include diffuse opacification, mucosal thickening (>4 mm), or an air fluid level. These findings, in conjunction with clinical features of acute sinusitis, are helpful in confirming the diagnosis.

Case Ico-delete Highlights A 73-year-old man presents with a 1-month history of left-sided hearing loss. He finds it especially difficult to hear higher-frequency sounds, but he is still able to appreciate tones. The hearing loss has been accompanied by a "ringing and flushing" sensation of the affected ear and facial numbness. For 1 week, he has noticed vertigo and balance impairments during standing and walking. He denies any history of falls, fever, chills, headaches, vision changes, weakness, speech deficits, paresthesias, otalgia, otorrhea, swollen glands, or sore throat. His physical exam is remarkable for left sensorineural hearing loss, facial hypoesthesia, and a diminished corneal reflex. The patient undergoes MRI imaging, with the image demonstrated. Question What pharmacotherapeutic agents would be most appropriate for this patient's vertigo?

Correct answer: Meclizine Explanation This patient's most clinical presentation and radiographic findings support a most likely diagnosis of acoustic neuroma. Meclizine (Antivert) is an antihistamines useful in preventing the histamine response in sensory nerve endings and blood vessels, and is effective in treating vertigo. Meclizine decreases the excitability of the inner-ear labyrinth and blocks conduction in inner-ear vestibular-cerebellar pathways. Its effects are associated with therapeutic effects in relief of nausea and vomiting. It is most effective if used as needed for 2-3 days with episodes of true vertigo. Scopolamine is an anticholinergic agent thought to work centrally by suppressing conduction in the vestibular-cerebellar pathways. It blocks the action of acetylcholine at parasympathetic sites in smooth muscle, secretory glands, and the CNS, antagonizing the actions of histamine and serotonin. Severe adverse effects preclude its use in elderly. Facial sensory disturbances occurs only with large neuromas. Facial sensory disturbance may respond to carbamazepine or oxcarbamazine medication for neuralgia. Nortriptyline is the most helpful pharmacologic treatment of tinnitus who endure related depression. There is no known indication in patients with tinnitus in the setting of acoustic neuroma. Propranolol, a beta-blocker, offers no known therapeutic effect in the management of symptoms associated with acoustic neuroma.

Question A man cannot hear any normal voice sounds spoken from more than 3 feet away. This is consistent with what type of hearing?

Correct answer: Moderate hearing loss Explanation The correct response is moderate hearing loss. Using a normal voice for testing, someone with normal hearing should hear sounds from at least 18 feet away. Someone with slight hearing loss will not generally hear sounds from more than 12 feet away. An individual with moderate hearing loss is limited to approximately 3 feet of hearing. Severe hearing loss is associated with sound perception only immediately around the meatus. Profound hearing loss is near-total or complete loss of one's hearing

Question A woman presents with a non-tender lesion on her tongue that she noticed a few days ago. Past medical history includes positive RPR "several years ago." Physical exam reveals the lesion is gray in color, raised, and oval-shaped. 3 similar patches are noted in her oral cavity. What is the most likely diagnosis?

Correct answer: Mucous patch due to syphilis Explanation Second-stage syphilis presents as a mucous patch that is gray in color, raised, oval-shaped, and may be distributed in multiple areas in the oral cavity. The patient's history of a positive RPR should put this diagnosis at the top of the differential list. Caviar lesions are varicosities found on the dorsal side of the tongue only. This type of lesion does not have any significance clinically and no intervention is needed. An aphthous ulcer, or canker sore, is a painful round whitish-gray ulcer that has a red surrounding halo. Tori mandibulares are benign, painless bony projections found on the inner side of the lower teeth. The mucosa covering these tori are of normal color and appearance. Leukoplakia is characterized by non-tender white patches found on the inside of the mouth. Biopsy is necessary to determine malignancy.

Case A 44-year-old Caucasian man presents with chronic rhinitis, nasal congestion, and a decreased sense of smell. He has had these symptoms for several years, but this is the first time he has sought care for them. The patient thought he may have seasonal allergies and has intermittently tried numerous over-the-counter allergy treatments, some of which diminish symptoms temporarily. On physical exam, the patient's vitals are normal. His HEENT exam is positive for yellowish, boggy nasal mucosal masses protruding bilaterally in the nares. Surrounding tissue is relatively pale, without erythema. Some clear nasal discharge is noted. Posterior oropharynx is mildly inflamed without exudate. Ear exam is normal. Nodes within the neck and auricular and occipital regions are not enlarged. Heart, lung, and abdominal exams are normal, with the exception of a dry, nonproductive cough noted a few times. Question Based on this patient's history and physical, which of the following is the most likely diagnosis?

Correct answer: Nasal polyps Explanation This patient most likely has nasal polyps, a consequence of long-term rhinosinusitis. Nasal polyps are benign mucosal masses that protrude into the nasal canal and may be unilateral or bilateral. They are associated with chronic allergies and a diminished sense of smell. A deviated nasal septum can produce decreased nasal airflow and present with some congestion-like symptoms, but the physical exam would reveal the septum lying off midline, without the mucosal masses characteristic of polyps. A nasal foreign body could present with a similar history of nasal congestion and rhinorrhea but is more frequent in young children at risk for putting objects into their noses. The physical exam would readily identify and distinguish a foreign body from nasal polyps. Neoplasms, such as nasopharyngeal carcinoma, are rare in comparison to the prevalence of nasal polyps and are typically asymptomatic until late in the disease process. A nasopharyngeal carcinoma is most common in patients of southern Chinese ethnicity. Symptoms might include rhinitis or nasal obstruction, similar to the presentation of this patient. However, a neoplasm may be more likely to include pain, nasal hemorrhage/bleeding, and unilateral symptoms if symptoms were present. Rhinitis medicamentosa is a condition resulting in severe rebound nasal congestion after prolonged and/or frequent use of nasal decongestants, such as phenylephrine. There are typically no distinguishing physical exam findings from this condition.

Case A 7-year-old boy is brought to the emergency department due to a 2-hour history of persistent bleeding from the nose. He has a history of several nosebleeds, which usually respond to pinching of the nose, but this episode has continued despite pinching the nose. His father reports that he is known to pick his nose and was noted to have some cold symptoms more recently. He did not experience excessive bleeding at circumcision, and there is no family history of bleeding disorders. On physical exam, he is alert and responsive to questions. His heart rate is 120 bpm; respiratory rate is 20/min; blood pressure is 105/64 mmHg; and oxygen saturation is 97% on room air. There is continuous active bleeding from his left nostril. On examination of the nose, no obvious bleeding site can be discerned in the anterior part of the nasal cavity. He receives phenylephrine and nasal packing after the initial evaluation, and his bleeding finally stops. Question What is the most appropriate next step in his assessment?

Correct answer: Nasopharyngoscopy Explanation The correct response is Nasopharyngoscopy. Nosebleeds are fairly common in children; they are usually associated with trauma (nose-picking), mucosal friability due to upper respiratory tract infection, and mucosal drying related to environmental conditions. Most episodes are self-limited and require simple measures, such as stopping the bleeding with pressure application (pinching) and comfort care. Recurrent nosebleeds are rarely noted to be due to an underlying anatomic or hematologic abnormality. Patients who require further evaluation are those who: have very frequent nosebleeds have bleeding that is difficult to control or localize have a positive family history of bleeding disorder have other signs suggestive of a bleeding disorder In most cases, initial evaluation of patient history and physical examination identifies the source and likely cause of the bleeding. About 90% of nosebleeds occur due to injury to the anterior vascular plexus of Kiesselbach in the anterior nasal septum; this site is usually visible on nasal exam. In this case, the inability to localize the source of the bleeding to the anterior vestibule suggests a more posterior source (bleeding from anterior or posterior ethmoidal or sphenopalatine arteries), which is more difficult to control. Nasopharyngoscopy helps identify the site and direct treatment. If a mass lesion or vascular anomaly is suspected, then CT or MRA may be considered as the next step in evaluation. In this case, chest X-ray would not provide any meaningful information that would help guide management. For a suspected bleeding disorder, the initial evaluation should include complete blood count with platelet counts, peripheral smear evaluation, prothrombin, time, partial thromboplastin time, and type and cross match sample to be kept if transfusion is anticipated. Based on the initial results, further testing can then be directed towards evaluation for platelet function abnormalities, factor assays, or von Willebrand factor/ristocetin cofactor assay as required

Question Ico-delete Highlights A 44-year-old man presents for follow-up of poorly controlled type I diabetes mellitus, which was diagnosed 32 years ago. What change on his funduscopic examination would indicate a need for urgent referral to an ophthalmologist?

Correct answer: Neovascularization Explanation Neovascularization is the hallmark of proliferative diabetic retinopathy. New vessels can appear at the optic nerve and the macula as a result of retinal hypoxia. They are susceptible to rupture, resulting in vitreous hemorrhage, retinal detachment, and blindness. Proliferative retinopathy requires urgent referral to an ophthalmologist and is usually treated with pan retinal laser photocoagulation. The risk of developing diabetic retinopathy is related to the extent of glycemic control and the duration of diabetes. It is classified as nonproliferative and proliferative. Blot hemorrhages, cotton wool spots, and microaneurysms are indicative of nonproliferative diabetic retinopathy, which is usually seen 10 to 20 years after the onset of diabetes. Nonproliferative retinopathy does not always progress to proliferative retinopathy, but if it becomes extensive, it can result in retinal ischemia, which increases the likelihood of proliferative disease. Flame-shaped hemorrhages are indicative of hypertensive retinopathy.

Question Ico-delete Highlights A 36-year-old woman presents with a small and irregular right pupil. On exam, you note that the pupil does not respond to direct or consensual light stimuli; however, it becomes smaller during an accommodation testing. What is the most likely diagnosis?

Correct answer: Neurosyphilis Explanation The clinical picture is suggestive of neurosyphilis; more specifically, it is likely tabes dorsalis. The pupil described here is the Argyll Robertson pupil. The pupil reacts poorly to light, but it reacts well to accommodation. Signs and symptoms seen in a TIA include temporary weakness and heaviness of the contralateral arm, leg, or face. There may be monocular vision loss in the eye contralateral to the affected limbs, which are not described in this patient. Retinal vein or artery occlusion will produce sudden vision loss, which is not described in this patient. Herpes simplex can involve the eyes, but the patient would develop keratitis (corneal inflammation) with impaired vision, and dendritic ulcers can be seen with fluorescein stain.

Question Ico-delete Highlights A 55-year-old man presents with intermittent vertigo, tinnitus, and progressive hearing loss over the last 4 years. There is no facial numbness. Based on the most likely diagnosis, what will an MRI of the head most likely show?

Correct answer: No abnormalities Explanation The clinical presentation is most consistent with Ménière's disease, which is thought to be caused by excess endolymph in the labyrinth. This condition causes no MRI abnormalities. The incidence of Ménière's disease ranges from 10-150 cases per 100,000 people each year. Acoustic neuromas may cause similar symptoms, but facial numbness is also usually present. In addition, vertigo is less commonly reported, as the symptom onset is more gradual. Acoustic neuromas are also fairly rare, occurring about 10 times per 1 million people each year.

Case A 33-year-old man with no significant past medical history presents with a 2-day history of an acutely painful sore in his mouth. He states that the area is tender to touch, and pain is exacerbated upon eating food. He denies any known history of trauma, sexually transmitted infections, fever, chills, swollen glands, sore throat, otalgia, otorrhea, or rhinitis. His oropharyngeal exam revealed the findings in the image. Question Ico-delete Highlights What is correct regarding this patient's condition?

Correct answer: Nutritional deficiencies are implicated in up to 20% of cases Explanation Nurtritional deficiencies are implicated in up to 20% of cases. This patient's most likely diagnosis is an aphthous ulcer. They are found mainly on the non-keratinized mobile mucosa of the lips, cheeks, floor of the mouth, sulci, or ventrum of the tongue; they are uncommonly seen on the keratinized mucosa of the palate or dorsum of the tongue. Most relevant studies have found hematinic (e.g., iron, folic acid, vitamin B12) deficiencies in as many as 20% of patients with recurrent ulcers. In addition, deficiencies of vitamins B-1, B-2, and B-6 have been noted in some patient cohorts. A minority of patients exhibit precipitating factors such as use of toothpaste containing sodium lauryl sulfate (SLS), trauma, stress, cessation of smoking, menstruation, and food allergies. Little evidence suggests an etiologic association between viruses and aphthous ulcers. Human herpesviruses (HHV)-6 and HHV-7 DNA have not been demonstrated in aphthous ulcers, but HHV-8 DNA is present in HIV-related oral ulcers. Diagnosis of aphthous stomatitis is based on history and clinical features. No specific tests are available. Rarely, biopsy may be indicated in cases in which a different diagnosis is suspected. The natural history of aphthous ulcers is of amelioration with age. The natural history is typically of resolution in the 3rd decade of life.

Case A mother has brought her 10-year-old son in to see you because she believes he has a lesion on his gums. She states that for the past 2 days he has been complaining about the tissue around his teeth and the inside of his cheeks being very sore. When she looked closely in his mouth with a flashlight, she noted bright red areas as well as occasional patches of white curd-like lesions. She denies that the patient has ever had this issue before. She also denies that the patient has had any shortness of breath or problems breathing; he has not been in close contact with sick contacts in the last few weeks. She denies any fever. The patient's past medical history is positive for severe seasonal allergies and mild persistent asthma. Medications the patient takes daily include: pediatric multivitamin, cetirizine hydrochloride 10 mg 1 tablet at bedtime, montelukast sodium 5mg 1 chewable tablet in the morning, beclomethasone disproportionate HFA 40 mcg 2 puffs twice daily in the am and pm, and albuterol sulfate inhalation powder 2 inhalations every 4-6 hours as needed for wheezing and 15 minutes just prior to physical activity. Physical examination reveals adherent thick white plaques with underlying erythematous tender mucosa on the gingival and buccal surfaces. Question Ico-delete Highlights Considering the patient's most likely pathology, what clinical intervention would be most appropriate at this time?

Correct answer: Nystatin oral suspension Explanation The lesions being described and seen are very highly likely to be caused by Candida albicans, which commonly causes oral candidiasis, commonly known as thrush. Adherent creamy white plaques on the buccal, gingival, or lingual mucosa is what the lesions look like; they may be asymptomatic or be painful. Thrush is a fairly common condition in infants, especially during the first few weeks of life. Other reasons pediatric patients will develop this condition includes recent antimicrobial therapy and daily inhaled corticosteroid therapy, such as for asthma maintenance. It is highly likely the cause of this patient's thrush. Simple steps can very easily eliminate this risk, which is to educate both patient and mom about washing/swishing his mouth immediately after using his beclomethasone either with water or even mouthwash. Less commonly but still possible is the patient potentially has a positive HIV status, although this is not a high possibility in this patient. Treatment in uncomplicated cases such as this one would typically be Nystatin suspension 200,000-500,000 units to be swished like mouthwash and spit out 5 times daily. Infants with oral candidiasis usually only need a suspension of 100,000 units and it can be given between 4-6 times daily. Other treatment options that have been utilized in pediatric patients include gentian violet, clotrimazole troches, or fluconazole. Azithromycin or amoxicillin are both incorrect, as these are antibiotics for bacteria and the patient's diagnosis is of a fungal origin. Chlorhexidine gluconate is most commonly used as a treatment for gingivitis. Fluticasone propionate is also an incorrect option, as this class of medication (corticosteroid) is what most likely caused the issue to begin with.

Case A 68-year-old man with a past medical history of hyperlipidemia, hypothyroidism and hypertension presents to his local medical office for a routine follow-up. He offers no complaints at this time. Further questioning reveals an 88-pack/year smoking history, and chronic alcohol use; he admits to drinking hard liquor "at least 3 days a week for most of his life." Upon physical exam, he had normal vital signs, unremarkable ear, nose, lymph node, and cardiopulmonary exams. His oropharyngeal exam revealed a slightly elevated plaque with an irregular outline that is opaque-white, and has a fine, granular texture. Of note, the medical provider was unable to scape this lesion. Question What is the most appropriate next intervention for this patient?

Correct answer: Obtain a biopsy Explanation Biopsy obtainment, repeated as necessary, is essential in this case. This patient most likely has oral leukoplakia. It manifests as patches that are bright white and sharply defined that cannot be rubbed off. The surfaces of the patches are slightly raised above the surrounding mucosa. Individuals are not symptomatic. Factors most frequently blamed for the development of leukoplakia include tobacco use, alcohol consumption, chronic irritation, candidiasis, vitamin deficiency, endocrine disturbances, and possibly a virus. It is considered a premalignant lesion. Topical retinoids are ineffective in this case. Diagnostic assessment of this lesion should be prompt and without delay. Observation for 6 months is not appropriate. This lesion is not characteristic of oral candidiasis, making initiation of an antifungal agent inappropriate. The oral lesions of thrush, when scraped with a tongue blade, leave behind an inflamed base; it may be painful, and it may bleed. Oral leukoplakia is managed exclusively in an outpatient setting.

Case A 73-year-old man presents with a nosebleed that will not stop. The bleeding has been present for over 2 hours. The patient's nose began dripping blood at breakfast with no known trauma. He denies pain. He has tried applying nasal pressure and laying down to rest. Until the bleeding began, the patient had not been experiencing any nasal symptoms, such as congestion, impaired nasal patency, or rhinitis. Because he felt too dizzy to drive, his wife drove him to the emergency department. The patient has no diagnosed medical conditions and takes no medications. He denies prior episodes of severe nosebleeds, easy bruising, and any known bleeding disorder. The patient's vitals are shown in the table. Weight 148 lbs. Height 69" Pulse 120 Respiratory rate 18 Blood pressure 90/66 mm Hg Temperature 97.4 °F/36.3°C On physical exam, the man is holding a large blood-soaked towel to his nose, with continued brisk bleeding. He is otherwise in no apparent distress, but he seems somewhat confused. On rhinoscopy, bleeding is observed from bilateral nares. No foreign body, mass, lesions, or abrasions are visualized. The bleeding site cannot be identified. Question What intervention is the next most appropriate step for this patient's current condition?

Correct answer: Obtain intravenous (IV) access and begin normal saline infusion Explanation Based upon this patient's history, and physical, he most likely has a posterior epistaxis. Posterior bleeds are less common than anterior bleeds, but should be suspected when the bleeding is large in volume and unresponsive to nasal packing/tamponade. The cause of a posterior bleed is not readily evident in this patient and many are idiopathic. However, with this patient's confusion, tachycardia, tachypnea, hypotension and blood loss, the provider must recognize that this patient is presenting with shock (hypovolemic) and needs to address that prior to the epistaxis. Of the choices listed, the healthcare personnel should obtain intravenous (IV) access and begin normal saline infusion. If a patient presents with shock and obvious blood loss, control of bleeding, crossmatching of blood and infusion of fluids and blood products are first-line actions. When a patient is hemodynamically stable and the provider can visually identify a blood vessel site for epistaxis, it is reasonable to apply electrocautery to visible nasal blood vessels. This patient is not hemodynamically stable, and no bleeding site can be identified, so electrocautery is not a viable option as the next step. It is common practice to apply nasal packing when bleeding points cannot be identified and an otolaryngologist is not available. Anterior packing alone, or a posterior balloon plus anterior packing may stabilize the bleed. Nasal and mucosa trauma can occur with packing. In this patient's case, packing could be performed once basic shock measures are completed. If this patient was hemodynamically stable and a trained surgeon was available, it would be reasonable to refer for surgical ligation of the nasal arterial supply. Vessel ligation success is quite high, but this hypotensive patient would not tolerate surgery in his present condition. For some nasal procedures, the provider may spray 4% topical cocaine intranasally for its decongestant and anesthetic effects. This may be used prior to electrocautery. For this patient, this intervention is inappropriate.

Case A 64-year-old African-American man presents to the emergency department after he went blind in his right eye "out of the blue" 20 minutes ago. There is no pain associated with his symptoms and he is not nauseated. Past medical history is positive for DMII for the past ten years. The pupil reaction on the left side is normal with pressure of 17mmHg. Right pupil evaluation reveals no reaction to light or accommodation with pressure of 20mmHg. Right eye ophthalmoscopy reveals arteriolar narrowing, vascular stasis, and "boxcar" pattern. Question What is the most likely diagnosis?

Correct answer: Occlusion of the central retinal artery Explanation The symptoms described above are typical for occlusion of the central retinal artery, which is a branch of the ophthalmic artery, in turn a branch of the internal carotid artery. The "boxcar" pattern is segmentation of the venous blood column, bilateral boxcar ring is a useful sign of circulatory arrest and death. Acute central artery occlusion is an emergency, since it results in permanent blindness if circulation is not restored within 30-60 minutes. An acute glaucoma attack is accompanied by severe pain with decreased vision. The patient usually reports seeing halos around light. The pupil is fixed in a mid-dilated position, and the eyeball is firm to pressure since the intraocular pressure is elevated. Subconjunctival hemorrhage onsets spontaneously and shows a painless, bright red patch on the sclera. It usually is caused by overexertion, is benign, self-limited and has no influence on the visus. Retinal detachment starts with the patient seeing dark, vitreous floaters, light flashes, and blurred vision, which progresses to blindness if not treated. Macular degeneration causes painless loss of visual acuity. There is altered pigmentation in the macula.

Case A 10-year-old boy presents with chronic otitis media with accompanying purulent discharge occurring over the past 6 months. He is very irritable, and his mother has been increasingly frustrated that he has not completely responded to the treatments. Question Which of the following organisms is most likely associated with this patient's condition?

Correct answer: Pseudomonas aeruginosa Explanation The correct answer is Pseudomonas aeruginosa, as it is one of the most common organisms causing chronic otitis media. Other common organisms include Proteus sp. and Staphylococcus aureus. The hallmark sign of chronic otitis media is purulent discharge from the ear. The condition is most often resolved by tympanoplasty. Haemophilus influenza, Streptococcus pneumonia, and Moraxella catarrhalis are common organisms found in acute otitis media, but Enteroccoccus faecalis is not typically found in chronic or acute otitis media.

Case A man has had 4 diagnosed sinus infections in the last 5 months, so you order a CT scan of his sinuses. He does not believe that any of the episodes completely resolved. The patient has a long history of excessive seasonal allergies; he treats them with daily oral OTC anti-histamines and he has repeatedly declined beginning allergy immunotherapy. He also has a history of asthma that is well controlled with daily inhaled corticosteroids. Question Ico-delete Highlights What imaging discovery do you expect to find?

Correct answer: Opacification Explanation This patient is most likely suffering from chronic rhinosinusitis (CRS); there may or may not be involvement of nasal polyps. The CRS would be classified as chronic due to the fact that he has had symptoms for over 12 weeks. CRS without nasal polyps accounts for up to 65% of cases of CRS. Risk factors that contribute to this condition include the presence of allergic rhinitis and asthma. On computed tomography (CT) studies, patients suffering from CRS without nasal polyps will generally present with sinus opacification (or sinus ostial obstruction) as well as mucosal thickening of the affected sinus cavity. Mucosal thinning and translucency are not associated findings on CT in CRS. Hypoattenuation generally describes an area whiter than usual and is used when referring to organs, such as the liver or kidneys.

Case Ico-delete Highlights A 22-month-old girl presents with a 3-day history of decreased appetite. Her mother has also noticed a decrease in the child's sleeping pattern and an increase in irritability; she is favoring her right ear, and she had a fever of 102.4°F taken rectally the night before presentation. All of the aforementioned factors led to the girl's mother making an appointment with you. The patient does not have any significant medical history. Question What signs found during the physical examination would prompt a bacterial-induced diagnosis and necessitate treatment?

Correct answer: Opaque, erythematous tympanic membrane with decreased mobility Explanation The correct response is opaque, erythematous tympanic membrane with decreased mobility. This is a classic example of a pediatric patient presenting with acute otitis media (AOM). A rapid onset of high fever, change of sleeping habits, alteration in appetite, and irritability are keys in regards to the child's medical history. She also is most likely having otalgia due to the fact that is she is favoring that ear. Both the American Academy of Pediatrics and the American Academy of Family Physicians define 3 criteria that must be present in order to make the diagnosis of AOM: acute onset of symptoms the presence of fluid in the middle ear (evidenced by the presence of a fluid line or visibility of purulent fluid) signs and symptoms of middle-ear inflammation (e.g., erythema, bulging, decreased mobility via pneumatic insufflation) Any otoscopic examination revealing a shiny, translucent membrane with no fluid line present and the evidence of tympanic membrane mobility would be considered a normal finding. Cerumen is also commonly found in the external portion of the ear, but not in the middle ear, and it is also normal for it to be visible in various amounts in the canal. Ventilation tubes, sometimes called ear tubes, tympanostomy tubes, or myringotomy tubes, are literally cylinders placed in the tympanic membrane to allow air into the middle ear. Ventilation tubes are usually recommended for patients who experience repeated AOM, have hearing loss due to otitis media with effusion, a malformation of the tympanic membrane or Eustachian tube, or barotrauma. More commonly, tubes are necessary in the pediatric population due to the common occurrence of AOM, but this need can be seen in adolescents or even adults.

Case A 14-year-old boy is seen in the office for a sports physical for the freshman basketball team. Past history is significant only for a high degree of myopia bilaterally, first diagnosed at age 4 years, and a dislocated shoulder at age 10 years that was easily reduced. Grades are A's and B's. Family history is significant for several unidentified ancestors having died in their forties of an unidentified cardiovascular disorder. Physical examination reveals normal vital signs. Height is 6'1" and weight 145 lbs. The upper to lower segment ratio is 0.65 (decreased). Arm span is 76". The palate is highly arched. Mild pectus excavatum was present. A 2/6 early diastolic murmur is present and is best heard at the second intercostals space at the right sternal border. Arachnodactyly of the fingers and toes and generalized loose jointedness and pes planus are also present. Echocardiography reveals a tricuspid aortic valve with grade 1 out of 4 aortic regurgitation with a normal aortic root diameter. Question In addition to echocardiography, which evaluation would be most productive?

Correct answer: Ophthalmology evaluation Explanation Ectopia lentis, retinal detachment, and other ocular anomalies are frequent in Marfan syndrome. CNS anomalies are unusual in Marfan syndrome, although intracranial aneurysms are described. Growth hormone is not elevated in Marfan syndrome. In growth hormone excess, the body habitus is proportionate, as opposed to the disproportionately long limbs seen in this patient. No specific findings will be seen on muscle biopsy. Further, this patient is not weak. The presentation does not suggest a collagen vascular disorder warranting a rheumatoid factor

Case A 33-year-old man presents with acute left eye pain. He was working in his garage on a woodworking project, and as he hammered in a nail, he felt that something hit him in the left eye. On examination, you note that the left pupil has a teardrop appearance. Question What is the next best step in the management of this patient?

Correct answer: Ophthalmology referral Explanation The clinical picture is suggestive of an intraocular foreign body or penetrating injury to the eye. This is commonly seen in individuals with a history of pounding on metal or using grinding equipment. The patient may give a history of "something hitting my eye" or "something was pulled out of my eye." His pupil is teardrop shaped, indicating penetration of the globe. Patients with suspected intraocular foreign body must be referred emergently to an ophthalmologist. Fluorescein staining is indicated for corneal abrasions/foreign bodies. To avoid extrusion of intraocular contents, EOMs should not be performed. Visual acuity should be tested, but alterations in visual acuity will not confirm your diagnosis. Testing intraocular pressure is indicated if you suspect glaucoma.

Question The client makes an appointment with your office having the complaint of a huge sore on his mouth that hurts terribly. In talking with the client he relates that his lip felt itchy a couple of days ago and then he had these bumps that looked like blisters. The blisters broke open and were extremely painful. The blisters are now drying and crusting over. He has never had this problem before, and the rest of his history is noncontributory. What is the most likely diagnosis?

Correct answer: Oral Herpes Simplex Explanation Oral herpes simplex are recurrent small grouped vesicles on an erythematous base, especially in the orolabial and genital areas. Herpes may follow minor infections, trauma, stress, or sun exposure; regional lymph nodes, usually the sub-mandibular glands, may be swollen and tender. Tzanck smear is positive for multinucleated epithelial giant cells; viral cultures and direct fluorescent antibody tests are positive. The principal symptoms are burning and stinging. Neuralgia may precede or accompany attacks. The lesion usually heals and crusts in 1 week. Patients can be educated to recognize attacks that they previously did not identify as recurrences of herpes simplex. Herpes simplex is the most common cause of painful genital ulcerations in patients with HIV infection. Genital herpes is treated with antiviral drugs Acyclovir 200mg po 5 times/day for 10 days, or valacyclovir one gram po bid for ten days. For comfort measure the client can use a warm water wash while voiding. Laryngeal Papillomas are common lesions of the larynx where ciliated and squamous epithelia meet. Unlike oral papillomas, laryngeal papillomas are likely to be symptomatic with hoarseness that progresses to stridor over weeks to months. The disease is more common in children. Treatment is by surgical removal of the papilloma. Laryngitis is probably the most common cause of hoarseness, which may persist for a week or so after other symptoms of an upper respiratory infection have cleared. Other causes include voice abuse, GERD , allergies, and exposure to chemicals. The patient should be warned to avoid vigorous use of the voice (singing, shouting), and advised voice rest for at least a week, while laryngitis is present, since this may foster the formation of vocal nodules. Although thought to be usually viral in origin, both Moraxella catarrhalis and Haemophilus influenzae may be isolated from the nasopharynx at higher than expected frequencies. Erythromycin may reduce the severity of hoarseness and cough. Oral Candidiasis (thrush) is usually painful and looks like creamy-white curd-like patches, overlying erythematous mucosa. Because these white areas are easily rubbed off (e.g., by a tongue depressor) - unlike leukoplakia or lichen planus - only the underlying irregular erythema may be seen. Oral candidiasis is commonly encountered among denture wearers; in debilitated patients, diabetes patients, and anemia patients. Also, patients undergoing chemotherapy or local irradiation; and in patients receiving corticosteroids or broad-spectrum antibiotics, may be affected. It may be the first presentation in HIV patients. Peritonsillar Abscess can begin with complication of tonsillitis with unilateral peritonsillar pain, and swelling of the anterior pillar and soft palate. There will be marked bulging of the tonsil into the oropharynx. This may occur when the patient is already on antibiotics.

Case A 52-year-old man presents with a concern of hearing changes. He has noticed a decreased ability to hear sounds for the past few months; he tested it at home by covering each ear, and he now thinks there is a hearing loss in only the left side. Furthermore, he hears a ringing sound all the time. He is a business manager, and he denies occupational exposure to loud noises. He denies head trauma, headaches, and prior ear problems. His wife thinks this is just normal age-related hearing loss. His review of systems is negative for other neurological symptoms. The patient's past medical history is unremarkable; he has no known medical conditions. He takes no medications. He has no allergies, and he has not had any surgeries. He denies alcohol, tobacco, and drug use. On physical exam, his vitals are normal. His HEENT exam is significant only for decreased auditory acuity and Weber test lateralizing to the right. Audiometry confirms a sensorineural hearing loss on the left. An MRI is performed; it shows a well-delineated intracranial mass. Further investigation reveals the origin of cells is from Schwann cells. Question What choice represents the best intervention for this patient's current condition?

Correct answer: Referral for surgery Explanation This patient is presenting with a vestibular schwannoma, or acoustic neuroma, that is affecting his vestibulocochlear nerve (cranial nerve VIII). This is one of the more common benign head and neck neoplasms. A common presentation is unilateral hearing loss and tinnitus. Treatment is typically surgical removal; another possibility is radiation therapy. Of the choices listed, referral for surgery is the best option; if he turns out to be a poor surgical candidate, radiotherapy should be discussed. This patient should not be referred for chemotherapy. Schwannomas are not typically responsive to chemotherapy, so doing so could delay appropriate treatment. If the patient was describing seizures, the provider should refer for electroencephalography (EEG), especially as some brain tumors are associated with seizures. However, this patient denied other neurological symptoms and has been clearly shown to have an intracranial mass. Referral for EEG would also delay definitive treatment. A referral for unilateral hearing aid would not fix this man's hearing loss; the treatment of his brain tumor would be delayed. A referral for ventricular shunt should be done for patients with hydrocephalus. Shunts have no role in schwannoma treatment.

Case A 19-month-old boy presents with a 3-day history of fever, irritability, and poor feeding. He also began to drool and she can hardly get him to eat anything. His past history is significant for recurrent ear infections. He had tympanostomy tubes placed about 1 month ago. On examination, he is febrile with a temperature of 39.8°C (103.6°F). He appears unwell and is uncooperative and irritable when touched. He is noted to be drooling and has several vesicles and shallow ulcers on his tongue and palate. The ulcers have a yellowish base with surrounding erythema. There are no lesions on any other areas of his body. Question What would be the most appropriate treatment for this child?

Correct answer: Oral acyclovir Explanation This child has symptoms and signs consistent with herpetic gingivostomatitis. Oral acyclovir is recommended for treatment of herpetic gingivostomatitis and leads to faster resolution of symptoms. Symptomatic treatment with oral analgesics and cold soothing foods, such as ice pops, is also helpful. Amoxicillin would not be used to treat a viral infection. Antibiotics may be indicated if there is evidence of secondary bacterial infection. Topical penciclovir is indicated for treatment of herpes labialis on the lips and face. The safety and efficacy on mucus membranes is unknown. Systemic steroids would likely worsen a viral infection. Nystatin is an antifungal and would be indicated for the treatment of fungal infections like oral thrush.

Case A 5-year-old boy presents with a sudden high fever and severe pain in the right ear. He has been very irritable since the initial onset of symptoms. He also seems to have some difficulty in hearing. The child has been suffering from a cough and cold for the last 2 weeks. He recently returned to the United States after visiting family abroad with his mother, where they spent time with an ill relative. The mother notes that he exhibited no irritability or signs of ear pain during the plane ride home. Question Ico-delete Highlights What is the best step in management of the boy's condition?

Correct answer: Oral amoxicillin Explanation The correct response is oral amoxicillin. The diagnosis in this case is acute otitis media (inflammation of the middle ear), which is a common childhood infection. Infants and children are at highest risk for otitis media; incidence rates are 15-20%, with peaks occurring from 6-36 months and 4-6 years of age. Children who develop otitis media in the first year of life have an increased risk of recurrent acute infection or chronic disease. In the usual course, a child suffering an upper respiratory infection for several days suddenly develops otalgia, fever, and hearing loss. The characteristic features include a bulging, opaque, erythematous tympanic membrane with impaired mobility. Purulent otorrhea may be present, but earache and fever are not always present. Any child with a "fever of undetermined origin" must also be evaluated for a middle ear infection. Bacteria are the primary agents of otitis media. The most common causes in all age groups are Streptococcus pneumoniae (25-40% of cases), followed by Haemophilus influenzae (15-25% of cases). Gram-negative bacilli cause about 20% of otitis media in neonates, but these bacteria are rarely found in older children with otitis media. Less common causes include group A Streptococci and Branhamella catarrhalis. Staphylococcus is a less common cause of chronic otitis media. Normally, children will improve clinically within 48 hours after antimicrobial therapy. If there is no improvement, the possibility of a resistant organism must be suspected; trimethoprim-sulfamethoxazole or erythromycin and sulfonamides may be given. The antibiotic of choice is amoxicillin orally; it is effective for both S. pneumoniae and H. influenzae. There is no added advantage of intramuscular injection over oral amoxicillin. An increasing percentage of H. influenzae and Moraxella catarrhalis strains have now become beta-lactamase producing and therefore ampicillin-resistant. Some resistant cases may benefit from a change of antibiotics to erythromycin or sulfonamides. Needle aspiration of the middle ear is only rarely necessary, as in the case of a critically ill child or a child who fails to respond to standard antimicrobial therapy.

Case Ico-delete Highlights A 25-year-old female college student presents with 2 lesions on the buccal mucosa on the right side of the oral cavity. You document these lesions with the following description: 2 round lesions, each measuring approximately 2 mm in diameter, with the presence of a white-yellow center that is surrounded by a red halo. Question Ico-delete Highlights What intervention would you recommend in order to help aid in healing?

Correct answer: Oral chlorhexidine Explanation The correct response is oral chlorhexidine. The patient above has an aphthous ulcer; they are also sometimes referred to as canker sores or aphthous stomatitis. These painful, open sores are found in the oral cavity, and they are the most common form of mouth ulcer. They are benign, noncancerous, and noninfectious; in most cases, the cause is unknown. The lesions are often described as having a white or yellow center that is surrounded by a bright red area. The majority of the time, aphthous ulcers will resolve without treatment. If they are causing the patient symptoms, a chlorhexidine-containing mouthwash may be prescribed. The mouthwash has been found to have an immediate bactericidal action and a prolonged bacteriostatic action, thereby relieving the pain of aphthous ulcers and aiding in healing. In fact, the clinical efficacy of chlorhexidine is well documented in review articles in post-op care following any dental pathologies or trauma. Neither oral antibiotics nor antivirals are indicated at this time; there is no direct evidence of a virus or bacteria being the etiology of aphthous ulcers. Acetaminophen and aspirin do not have any significant effects in terms of helping heal aphthous ulcers. These agents may aid in relieving some of the pain associated with them, but they will not aid in healing.

Case While performing a comprehensive oral exam on a patient, you note 3 lesions on the buccal mucosa on the right side of the oral cavity. You document these lesions with the following description: 2 round lesions, each measuring approximately 2 mm in diameter, with the presence of a white-yellow center surrounded by a red halo. Question What clinical intervention should you recommend to this patient in order to help aid in healing?

Correct answer: Oral chlorhexidine Explanation The patient above has an episode of aphthous ulcers; they are also sometimes referred to as a canker sores or aphthous stomatitis. These painful, open sores are found in the oral cavity, and they are the most common form of mouth ulcer. They are benign, noncancerous, noninfectious, and in most cases, the cause is unknown. Many times, the lesions are described as having a white or yellow center, and they are surrounded by a bright red area. The majority of the time aphthous ulcers will resolve without treatment. Patients that are experiencing symptoms (such as those in the patient above) can be prescribed chlorhexidine-containing mouthwash. It has been found to have an immediate bactericidal action and a prolonged bacteriostatic action, thereby relieving the pain of aphthous ulcers and aid in healing. In fact, the clinical efficacy of chlorhexidine is well documented in review articles regarding post-op care following any dental pathologies or trauma. Oral antibiotics and antivirals are not indicated at this time; there is no direct evidence of a virus or bacteria being the etiology of aphthous ulcers. Aspirin and acetaminophen do not have any significant effects in terms of helping heal aphthous ulcers. These agents may aid in relieving some of the pain associated with them, but they will not provide any aid in healing.

Case A 42-year-old man presents for evaluation of a growth on his tongue. He thinks that the lesion has been present for a few months, and it has not changed; however, he generally prefers to avoid health care, and he has not been concerned. He is only here at the urging of his family member. The patient denies oral symptoms and changes in taste sensation; he states that he generally feels fine. The patient denies the use of chew tobacco and cigarettes. On physical exam, there is a white patch of tissue, which does not scrape off; there is a 'shaggy' appearance on the left lateral tongue. No erythema is noted. No other lesions are identified. The remainder of his exam is normal. A biopsy of the lesion is obtained. The pathology shows hyperkeratosis, "balloon" cells in the upper cell layer, and Epstein-Barr virus (EBV) in the basal epithelial cells. Question What is the most likely diagnosis?

Correct answer: Oral hairy leukoplakia Explanation This patient has oral hairy leukoplakia, which is often associated with HIV-infection or other immunocompromised states (such as post-transplant). It is rare in immunocompetent individuals. The oral hairy leukoplakia is a benign neoplasm of the tongue; in and of itself, it is not of great significance. However, due to its association with an immunocompromised state, it can be a harbinger of a more significant condition. Geographic tongue is a benign condition of the tongue; it affects the epithelium. It typically presents with erythematous patches, with white, rounded borders. Lesions can change size, patterns, and locations. Patients may be asymptomatic or experience burning or oral discomfort. Oral candidiasis (thrush) is a fungal infection of the oral mucosa. Typically, affected individuals note oral pain or discomfort. There may be white plaque with surrounding or underlying erythematous tissue. Biopsy is not typically necessary for diagnosis; this patient's biopsy would have indicated fungal organisms if he had thrush. Oral lichen planus can present with white 'lace-like' patches, but it typically presents with oral pain. Squamous cell carcinoma can present in a variety of lesions (plaques, ulcerations, erosions, papules). Any persistent oral lesion should be biopsied to evaluate for possible malignancy. This patient's biopsy did not indicate malignancy.

Question A 34-year-old woman presents to your office to establish care. Her past medical history is significant for gastritis. She has no other medical problems. As part of your new patient assessment, you perform a neurological examination. On confrontation with visual field testing, you note bilateral temporal field defects, specifically a bitemporal non-homonymous hemianopsia. The remainder of your neurological evaluation is unremarkable. What would be your next step in the management of this patient?

Correct answer: Order an outpatient MRI of the brain Explanation Bitemporal visual field loss localizes to the optic chiasm. In a 34-year-old patient, the most likely cause is a pituitary tumor. The next step in management would be to obtain brain imaging to verify the presence of a lesion and to evaluate its extent. An urgent referral to the emergency room is not indicated; at the patient's age, there is nothing to indicate a stroke. Thyroid function tests may be abnormal with a pituitary lesion. Evaluating thyroid hormone levels may be important in characterizing a pituitary lesion if one is present. The first step in management is to obtain brain imaging. Checking an EKG in the office is not indicated based on the information presented. While glaucoma can cause visual field defects, a bitemporal hemianopsia suggests a chiasmal lesion. Glaucoma would be unlikely in this setting and with a patient of this age.

Case Ico-delete Highlights A 39-year-old Caucasian man presents with a "lump in his left ear canal." He just wants to confirm it is not a type of tumor. Otoscopic examination reveals a single, discrete, pedunculated, flesh-colored bony mass located at the 7 o' clock position in the left external auditory canal. The right external ear canal was unremarkable. You tell the patient that the lump in his ear canal is indeed classified as a tumor but is completely benign. Question What is the "lump" called?

Correct answer: Osteoma Explanation Osteoma is the most likely answer for this patient scenario.These will manifest as discrete, pedunculated bony massesthat arise in the external solitary canal, usually from the tympanosquamous suture line. Osteomas are typically solitary, unilateral, and slow growing. They are also the most common bony neoplasms of the temporal bone. Usually osteomas are asymptomatic, and no treatment is necessary; however, they could cause issues if the canal becomes obstructed. This lesion is also commonly referred to as "surfers ear" due to the possibility of chronic exposure of cold water/wind leading to their formation. Keratoacanthoma is a lesion that is more often linked to sun exposure, chemical carcinogens, trauma, and possibly a viral etiology.These are usually located more often on the skin of the face, forearms, and dorsal aspects of the hands. Actinic damage is evident in the surrounding skin around the lesion. There are usually 3 stages to these lesions: proliferative, mature, and resolving. Appearance can vary from smooth, firm, symmetric, erythematous, or skin-colored with central keratotic or even partly necrotic, depending on the stage the lesion is in. Pilomatrixoma is also incorrect.Characteristics of this lesion include necrosis, keratinization, and proliferation of small epithelial cells; calcification and ossification may also be evident. These are slow-growing, bluish, solitary, and well-circumscribed nodules, usually in the dermis of the auricle or pre-auricular area. Sebaceous adenoma is the incorrect choice. These lesions are extremely rare. As obvious from the name, these arise from the sebaceous glands and present as smooth, elevated, pedunculated, firm-to-soft lesions that are solitary and smaller than 0.5 cm. Their color varies from tan, to skin-colored, to pink, and red. Squamous papilloma is another benign external auditory canal tumor, but it is not the correct option for the patient above. These will present as wary, verrucous, elevated lesions that are pigmented and may have an overlying thickened keratin layer.

Case A 17-year-old boy presents with a 1-day history of an extremely painful and pruritic left ear after returning from a weekend trip to the beach. Physical examination of the left ear reveals an erythematous external canal without clear visualization of the tympanic membrane and large amounts of debris and discharge. The patient grimaces and expresses a painful sensation when the left pinna is manipulated. Physical examination of the right ear is benign. Question Ico-delete Highlights What is the most likely diagnosis?

Correct answer: Otitis externa Explanation Otitis externa, also known as swimmer's ear, is an infection (usually bacterial) of the external auditory canal. This condition is quite common after swimming, and the causative organism is most often Pseudomonas aeruginosa. Other factors such as irritation by other chemicals and trauma can also lead to this condition. Patients present with a painful, pruritic ear, sometimes accompanied by a discharge. On physical exam, these patients have a swollen, erythematous auditory canal. Manipulation of the tragus or the pinna of the affected ear elicits pain. This differentiates the condition from otitis media, where pain is not elicited. Malignant external otitis is also an infection of the outer ear, but it spreads to involve the skull, most often the temporal bone. This infection is life-threatening, and it is most often seen in diabetics and immunocompromised patients. Ramsey-Hunt syndrome (herpes zoster oticus) is due to a shingles outbreak that affects the facial nerve near the ear. In addition to the painful shingles rash, it may cause hearing loss and facial paralysis in the affected ear. Tympanic membrane perforation allows a middle ear abscess to drain, which relieves increased middle ear pressure. Once pressure has been relieved, the tympanic membrane usually heals quickly, in hours to days.

A 13-month-old girl presents in the urgent care center with a history of fever, increasing lethargy, and decreased appetite for 4 days. She has had watery rhinorrhea, an occasional dry cough, and progressive vomiting with 6 episodes in the prior day, but no diarrhea. Also noted are pain and difficulty when swallowing and noisy breathing. She has no active medical problems and her immunizations are up to date. On examination, her temperature is 39.5°C, respiratory rate 48, apical pulse 162, and blood pressure 90/66 mm Hg. Physical examination demonstrates decreased hydration of the oral mucosa, increased capillary refill time, and cool extremities. Tympanic membranes are clear. Pupils are equal, round, and reactive to light. Nasal mucosa appears mildly erythematous with some dried crusted secretions in both nares. Nasal flaring is noted. There are no oral or pharyngeal lesions visible; tonsils appear to be of normal size and without inflammation or exudates. Neck resists flexion. Shotty bilateral anterior cervical nodes are palpated. The chest is clear and without retractions. The remainder of the examination is normal. Initial laboratory studies provided the following results: CBC - WBC 26,500 with differential 24% polymorphonuclear leukocytes, 60% bands, 11% lymphocytes, 1% eosinophils, and 4% atypical lymphocytes. Hemoglobin 10.8 g/dL, platelets 268,000. Blood culture was drawn. Cerebrospinal fluid - 11 white blood cells and 1 red blood cell/mm3, protein 40 mg/dL, glucose 79 mg/dL. Gram stain showed no organisms. Liver function tests were normal. A basic metabolic panel was normal except for a CO2 of 20 and a BUN of 22. Initial treatment consisted of hydration and antibiotics with a presumptive diagnosis of aseptic meningitis. Fever, irritability, and nuchal rigidity persisted. The following radiograph was obtained. Refer to the image. Question Ico-delete Highlights After review of the tests, who is the most appropriate consultant to call?

Correct answer: Otolaryngologist Explanation The X-ray of the lateral neck demonstrates a retropharyngeal abscess. Definitive treatment consists of surgical drainage performed by an otolaryngologist. This is primarily a pediatric illness, rarely occurring in adolescents and adults. Physical examination may demonstrate nuchal rigidity and/or cervical lymphadenopathy. An X-ray of the lateral neck may reveal the diagnosis with findings of a widened prevertebral soft tissue shadow, the presence of air-fluid level, foreign body, and/or loss of the normal curvature of the cervical spine. The width of the prevertebral soft tissue should be less than or equal to the width of the adjacent vertebral body. Radiologically, it may be difficult to distinguish an abscess from cellulitis in the neck. A follow-up CT scan of the neck may help. A chest X-ray should be done to determine if there is mediastinal extension or aspiration pneumonia. Laboratory findings reflect an acute bacterial infection. White blood cells in the cerebrospinal fluid can result from irritation of the meninges by infection in adjoining tissues.

Question A 67-year-old woman presents saying her husband says she doesn't listen to anything he says. The patient states that occasionally she has to ask people to repeat themselves when sitting to her right. She denies any dizziness, headaches, or visual disturbances. Her current medication is furosemide. On physical examination, the Weber test reveals lateralization to the left ear. On the left ear air conduction lasted for 15 seconds and bone conduction lasted 10 seconds. On the right ear air conduction lasted for 22 seconds and bone conduction lasted 10 seconds. What do you suspect as the cause of this hearing loss?

Correct answer: Ototoxicity Explanation Ototoxicity secondary to furosemide is the correct answer. Loop diuretics can cause sensory hearing loss, as evidenced by this patient's physical exam finding of lateralization to the good ear and air conduction slightly longer than bone conduction. The Rinne test should reveal an air:bone conduction ratio of 2:1. Cerumen impaction will cause a conductive hearing loss with the lateralization to the affected ear and a negative Rinne test. Otosclerosis typically will result in conductive hearing loss. Meniere's Disease is incorrect because the patient does not exhibit any vertigo or tinnitus. Middle ear effusion is incorrect because it would cause a conductive hearing loss.

Case A 41-year-old woman initially presented to the local emergency department with a 10-day history of nasal drainage, congestion, fever, and cheek pain. She was prescribed a 10-day course of amoxicillin/clavulanate. After 10 days of therapy she presents again, reporting that she is no better. The fever continues and she now has upper tooth pain. On exam, she is in mild distress, with thick, purulent rhinorrhea and halitosis. Question Ico-delete Highlights What is the next step in the care of this patient?

Correct answer: Refer to ENT for sinus aspiration Explanation The patient has bacterial sinusitis that is amoxicillin/clavulanate resistant. The sinuses need to be aspirated and cultured, so that the correct antibiotic therapy can be instituted. This moderately ill patient does not warrant admission and intravenous therapy or surgical intervention. Such treatments are reserved for those who are severely ill. Since the amoxicillin/clavulanate has had no effect after 10 days, the infection is likely resistant to this medication. Clinical trials do not support the use of broad spectrum antibiotics for routine cases of bacterial sinusitis.

Case A 28-year-old woman with a past medical history of well-controlled asthma presents with recurrent sneezing episodes, nasal itching, congestion, and headache. Her physical exam reveals post-nasal drip, a transverse nasal crease, and bilateral infraorbital cyanosis. Question What additional finding(s) support an allergic etiology of this patient's presentation?

Correct answer: Pale bluish nasal mucosa upon speculum examination Explanation This patient's past medical history and current presentation are remarkable for allergic rhinitis. Signs and symptoms include sneezing paroxysms, clear rhinorrhea, nasal congestion, pale bluish nasal mucosa, transverse nasal crease, infraorbital cyanosis (allergic shiners), serous otitis media, along with nasal, ocular, or palatal itching. Viral rhinitis is often associated with other manifestations of viral illness, which can include headache, malaise, body aches, and cough. Nasal drainage in viral rhinitis is most often clear or white and can be accompanied by nasal congestion and sneezing. The nasal septum and turbinates are typically erythematous and edematous. Patients with vasomotor rhinitis present with symptoms of nasal obstruction and clear nasal drainage. The symptoms are often associated with changes in temperature, eating, exposure to odors and chemicals, or alcohol use. Some clinicians suggest that abnormal autonomic regulation of nasal function leads to vasomotor rhinitis. The presence of fever in conjunction with copious, purulent green nasal discharge should raise suspicion for underlying infection, especially bacterial rhinosinusitis.

Case A 17-year-old girl presents after waking up in the morning with a sore throat and fever but no cough. On examination, the pharyngeal mucosa is erythematous without exudate and she exhibits painful anterior cervical lymphadenopathy. She has no past medical conditions and no known allergies. A rapid strep test is positive. Confirmatory culture is sent to the lab. Question Ico-delete Highlights What is the most appropriate treatment for this patient?

Correct answer: Penicillin V Explanation Penicillin V is the drug of choice for the treatment of group A streptococcus (GAS) pharyngitis. It is the preferred treatment due to its narrow spectrum of activity. Cephalexin is effective against GAS pharyngitis but is typically reserved for those with allergies to penicillin. Clindamycin and clarithromycin are both effective against GAS pharyngitis but are reserved for those with penicillin allergy and accompanying anaphylactic sensitivity. Patient should not be treated until confirmatory culture results are received is incorrect. Patients with positive rapid strep test do not require confirmatory culture prior to treatment. Patients should be treated as soon as possible to avoid complications of GAS pharyngitis, including peritonsillar abscess and acute rheumatic fever.

Case A 68-year-old man with a 50-pack/year smoking history presents with a 4-month history of progressive dysphagia. His review of symptoms is notable for intermittent ear pain, especially upon swallowing; there has also been an involuntary weight loss of 12 pounds over the past 4 weeks, slowly progressive hoarseness, and occasional hemoptysis. He denies chills, abdominal pain, shortness of breath, chest pain, vomiting, skin changes recent travel, and sick contacts. His physical exam is remarkable for nontender, immobile lymphadenopathy of the cervical nodes; there is also stridor upon auscultation of the trachea. A nasopharyngoscope reveals the following imaging. Question What is the next most appropriate step in the management of this patient?

Correct answer: Perform a biopsy during laryngoscopy Explanation This patient is demonstrating signs and symptoms that are consistent with laryngeal cancer. In the accompanying image, cancer along the right vocal cord causing hoarseness and sore throat is evident. Squamous cell carcinoma of the larynx, the most common malignancy of the larynx, occurs almost exclusively in patients with a history of significant tobacco use. Diagnosis is made by biopsy at the time of laryngoscopy; it is when true fold mobility and arytenoid fixation, as well as surface tumor extent, can be evaluated. Most otolaryngologists recommend esophagoscopy and bronchoscopy at the same time to exclude synchronous primary tumor. A CT of the chest is not recommended at this time. This patient is not exhibiting signs or symptoms of a bacterial infection or abscess formation; therefore, neither antibiotics nor incision procedures are indicated, Voice rest and hydration may help to alleviate symptoms of vocal cord abuse; however, this patient's presentation is inconsistent with this diagnosis.

Case A 40-year-old man presents with a painless, circular, 7-millimeter spot inside his mouth; he noticed it 2 days ago. His medication list includes propranolol for hypertension. He is a known alcoholic. The diagnosis of leukoplakia of the buccal mucosa is made. The physician adopts a 'watch-and-wait' attitude; however, after 2 weeks the lesion is still present and unchanged. Question What is the best course of management?

Correct answer: Perform a biopsy of the lesion Explanation The correct response is that you should perform a biopsy of the lesion. Leukoplakia is a white keratotic lesion seen on mucous membranes. Irritation from various mechanical and chemical stimuli, including alcohol, favors development of the lesion. Leukoplakia can occur in any area of the mouth, and it usually exhibits benign hyperkeratosis on biopsy. On long term-term follow-up, 2% to 6% of these lesions will have undergone malignant transformation into squamous cell carcinomas. Oral nystatin would not be appropriate treatment because this lesion is not typical of oral candidiasis. Candidal lesions usually are multiple and spread quickly when left untreated. A fluorescent antinuclear antibody test is also not indicated, as the oral lesions of lupus erythematosus are typically irregular, erosive, and necrotic. An idiosyncratic reaction to propranolol is unlikely in this patient.

Case A 12-year-old Caucasian girl presents with a sore throat. The onset of symptoms was about 24 hours ago. The patient experiences pain in her throat, especially with talking or swallowing. She is also very fatigued because throat pain prevented her from sleeping last night. Throat lozenges have not been helpful. She attends public school with several other classmates who have been out sick recently. The patient denies nasal congestion, rhinorrhea, and cough and is unsure of fevers. This patient has no chronic medical conditions, takes no medications, and has no known drug allergies. Her vitals are: Weight 92 lb Height 56: Pulse 95 Blood pressure 102/60 Temperature 99.2°F/37.3°C On physical exam, the patient appears slightly ill and fatigued. Her HEENT exam is positive for bilateral cervical lymphadenopathy and inflamed posterior oropharynx without exudate. She does have normal range of motion of the neck without eliciting pain. Her heart, lung, and abdominal exams are normal. No other lymph nodes are palpable. Question Ico-delete Highlights What is the most appropriate next step in the care of this patient?

Correct answer: Perform rapid antigen testing for group A streptococcus. Explanation The next most appropriate step in the management of this patient with acute pharyngitis is to perform rapid antigen testing for group A streptococcus. While viral causes of pharyngitis are extremely common, management should focus on identifying those at risk for group A beta-hemolytic streptococcal infections to prevent compilations such as rheumatic fever and glomerulonephritis. This patient presents with acute pharyngitis, no cough, and cervical lymphadenopathy, which all support a likely diagnosis of streptococcal origin. Additional indicators would be temperature over 38°C and tonsillar exudate. Excusing the patient from school while symptoms are still present may limit the spread of the infection and allows the patient to rest, but this does not address the possible need for antibacterial treatment and would leave the patient at risk for developing complications if she indeed has group A streptococcal pharyngitis. Initiation of amoxicillin may be appropriate management once group A streptococcal pharyngitis is confirmed. (Penicillin has been the standard primary regiment.) However, if this patient's pharyngitis is not found to be caused by streptococcus (and therefore, most likely viral), use of antibiotics is unnecessary and only puts the patient at risk for adverse effects of the medication. In fact, if this patient's pharyngitis is caused by infectious mononucleosis, use of amoxicillin can precipitate a skin rash. A complete blood count (CBC) will likely show some characteristic changes in patients affected by acute pharyngitis. For example, the total white blood cell (WBC) count is likely to be elevated. Depending on the potential cause of the pharyngitis, the neutrophils and monocytes may be elevated for bacterial and viral causes, respectively. The CBC changes are rather unspecific, however, especially early in the infection. It makes more clinical sense to directly swab the patient's throat and get a result within minutes rather than to subject a 12-year-old to a blood draw with a typically longer wait time for results. The conservative approach of fluid, rest, and analgesics/antipyretics is reasonable if the patient is not likely to have group A streptococcal pharyngitis. Aspirin is not the best choice for a 12-year-old, though. Because of the link between aspirin use and Reye's syndrome in children, salicylate use is discouraged in children, especially if suffering viral illnesses.

Case A 32-year-old woman with a past medical history of multiple environmental allergies presents with bilateral ocular pruritis, eye redness, and excessive tearing over the last several days. She notes associated nasal congestion. She states that these symptoms are predictable, occurring at approximately the same time every year. She has had these symptoms for many years, and over-the-counter medications are not helpful. She denies ocular trauma, fever, chills, swollen glands, otalgia, otorrhea, headache, and sore throat. She further denies shortness of breath, cough, and chest pain. Her physical exam is remarkable for bilateral conjunctival erythema and chemosis, violaceous mucosal turbinates in the nasal cavity, and small nasal polyps. Question Ico-delete Highlights What health maintenance strategy is correct regarding this patient?

Correct answer: Pillow and mattress covers, air purifiers, and dust filters should be purchased. Explanation This patient's diagnosis is allergic rhinitis and conjunctivitis. Maintaining an allergen-free environment by covering pillows and mattresses with plastic covers, substituting synthetic materials for animal products, removing dust-collecting household fixtures, and using air purifiers and dust filters are all recommended. Warm compresses are useful in managing hordeolums and blepharitis, not allergic conjunctivitis. Numerous over-the-counter antihistamines offer the benefit of reduced cost but are associated with higher rates of drowsiness. When using intranasal corticosteroids, patients should be reminded that there may be a delay in the onset of relief for 2 or more weeks. Corticosteroid sprays may also shrink nasal polyps, delaying or eliminating the indications for endoscopic sinus surgery.

Case A 69-year-old woman presents with epistaxis. She has been applying pressure to her nose for the last hour, but she came to the emergency room when the bleeding did not subside. Past medical history is significant for type II diabetes, hypertension, and an acute myocardial infarction 2 years ago, which was treated with stent placement. On physical exam, the patient is afebrile, with pulse 92 beats per minute, respirations 23 per minute, and a blood pressure of 185/105. There is blood flowing through both nares, and blood is seen down the posterior oropharynx. Examination of the nasal cavity does not reveal a specific area of bleeding. Question Ico-delete Highlights What is the appropriate next step for this patient?

Correct answer: Posterior nasal packing Explanation Posterior nasal packing is correct. The presence of blood in the posterior oropharynx indicates posterior epistaxis. Posterior epistaxis is responsible for approximately 5% of cases of epistaxis, and it most commonly presents in patients with history of hypertension and atherosclerosis. Posterior epistaxis does not respond to direct pressure. Posterior nasal packing is the appropriate next step in treatment. Resume placing direct pressure, cauterization of the nasal cavity, and anterior nasal packing are incorrect. These are all treatments for anterior epistaxis, and they would not be effective in posterior epistaxis. Surgical ligation of the nasal artery supply can be used in the treatment of posterior epistaxis. However, it is reserved for patients who do not respond to posterior nasal packing.

Case A 32-year-old man presents with chronic runny nose and congestion; according to the patient, it seems to come and go. He reports getting similar symptoms each fall, which is when he goes hunting. His symptoms are just starting again this year. He describes the nasal discharge as clear and watery. He often awakes with congestion. He admits to some sneezing, "itchy nose", and an infrequent dry cough. He denies fever/chills, headache, itchy or watery eyes, and productive cough. He has tried over-the-counter allergy pills; however, they cause excessive sedation. His past medical history is unremarkable. He has no known medical conditions. He is not currently taking any medications; he has no known drug allergies, and he has not had any surgeries in the past. He lives with his girlfriend and works in customer service; he denies the use of alcohol, tobacco, and drugs. On physical exam, his HEENT exam is remarkable for violaceous nasal turbinates bilaterally, with clear nasal discharge. The rest of his exam is normal. Question What provides the most appropriate intervention for this patient's current condition and recurrent symptoms?

Correct answer: Prescribe intranasal corticosteroid Explanation This patient is suffering from seasonal allergic rhinitis and is experiencing primarily nasal symptoms. With a history of cyclic symptoms, including nasal congestion, rhinorrhea, pruritus, and clear nasal discharge, as well as no fevers or symptoms consistent with viral or bacterial sinusitis, the diagnosis can be made without tests. The next step is treatment. First-line treatment is to prescribe an intranasal corticosteroid or antihistamine. Nasal corticosteroids shrink hypertrophic nasal mucosa. Patients should be educated that noticeable efficacy may take approximately two weeks. This patient could be started on an intranasal steroid spray now and use it seasonally for a few months each fall. Although it likely would be effective for symptoms, it would be inappropriate to prescribe a first-generation oral antihistamine (such as diphenhydramine or chlorpheniramine). The patient gave a history of trying such medications (which are available over-the-counter) and experiencing excessive sedation. Some of the newer, second-generation antihistamines would be a preferable choice, especially if the patient was experiencing allergy symptoms in his eyes. An over-the-counter nasal saline spray is often recommended for various nasal conditions. In cases of allergic rhinitis, these sprays can mechanically flush allergens out of the nasal cavity; however, their role is more of an adjunct treatment, not a primary treatment. Continuous positive airway pressure (CPAP) use while sleeping is a treatment for obstructive sleep apnea (OSA). OSA is characterized by nighttime snoring, gasping for air, awakening unrefreshed, and excessive daytime sedation. There is no role for CPAP in allergic rhinitis. If this patient were presenting with an acute bacterial sinusitis (classically described as an acute episode of nasal congestion, discolored rhinorrhea, sinus pressure, headache), it would be appropriate to prescribe high-dose amoxicillin/clavulanate. However, there is no infectious origin to this patient's allergic rhinitis, and antibiotics do not have a place.

Case A 19-year-old woman presents with a painful sore in her mouth. It has been present for 3 days. She denies oral trauma. She describes a single, painful lesion that feels like a bump between her cheek and gum line. She has not tried any treatments at home. She denies history of similar lesions. She has otherwise been feeling well and denies recent flu-like symptoms. Her past medical history is unremarkable, with no known medical conditions or history of surgery; she takes no medications and has no allergies. She lives in an apartment with her boyfriend; she works at a convenience store, and she denies the use of alcohol, tobacco (including chew tobacco), and drugs. On physical exam, her vitals are normal. A single, tender lesion is identified on the buccal mucosa, just across from the lower right jaw. It is a small, shallow ulcer; it is approximately 3 mm diameter, with a yellow-gray center, and it is surrounded by a red halo. The remainder of her exam is normal. Question What is the best intervention for this patient's condition?

Correct answer: Prescribe topical corticosteroids Explanation This patient is presenting with an aphthous ulcer (canker sore), which is a very common oral lesion. Treatment is targeted at pain relief, and it would be most appropriate to prescribe topical corticosteroids. These have been shown to provide symptomatic relief. Aphthous ulcers tend to resolve on their own within 10 - 14 days. An aphthous ulcer may be confused with herpes; in fact, an association with human herpesvirus 6, has been found. However, because aphthous ulcers are self-limiting, prescribe oral antiviral medication is not recommended. This condition can be managed with any urgent care or family practice, so it does not require a referral to a dermatologist. Such referral would delay pain treatment and incur unnecessary health care costs. Likewise, aphthous ulcers are not malignant, and this presentation is not similar to any oral lesion that would require surgical removal; therefore, she should not be referred to an oral surgeon. Recurrent aphthous ulcers can be linked with celiac disease, HIV, and inflammatory bowel conditions. It would be reasonable to test this patient for autoimmune diseases if she were presenting with recurrent ulcers. However, with a single occurrence, no further testing is recommended.

Case A 30-year-old woman presents to the emergency room at 7 am with severe pain and swelling of her right eye. She was awakened early the previous evening due to the discomfort and swelling of the surrounding conjunctiva. She found it difficult to sleep due to the discomfort. She planned on going to work, but the swelling had closed her eye shut, and she developed excruciating pain in the eye that radiated internally. The patient denies recent swimming and does not recollect any previous trauma or injury to the eye. She uses contact lenses, but they were not in use due to the condition of her eye. The contact lenses were stored in a small pillbox container with some fluid that she later described as tap water. She ran out of sterile cleaning and soaking solution for the contact lenses, so she has been using tap water as a substitute for approximately 5 days. She frequently sleeps with her contacts in. The patient is afebrile. Pulse is 70/min, and blood pressure is 135/80 mm Hg. Lungs are clear, and there is no evidence of lymphadenopathy. The eye has profound conjunctivitis that is acute and follicular. Purulent drainage is present. The acute nature of conjunctivitis requires an ophthalmologist consult. The ophthalmologist obtains ocular fluid for culture and Gram stain. CBC results are unremarkable. The Gram stain reveals the following results (see image). Prompt and aggressive therapy is initiated. Question Ico-delete Highlights What is the most likely organism causing this acute eye threatening infection?

Correct answer: Pseudomonas aeruginosa Explanation Pseudomonas aeruginosa causes a supprative keratitis that is rapid in its development. It can be caused by previous injury to the eye, or possible, recurrent, minor corneal epithelial trauma, caused by contact lenses. The incidence is higher with soft lenses and extended overnight wear. Fever is usually absent, and leukocytosis is absent or minimal. The infection can lead to corneal ulceration, resulting in the rapid loss of ocular function; therefore, these infections need to be approached as a medical emergency. It is a very common opportunistic source of human infections, especially in the hospital setting. Pathogenesis is due to its minimal nutritional requirements, relative resistance to antibiotics, and a host of other invasive and toxicogenic substances that it produces. Scrapings from the floor of the ulcer exhibiting Gram-negative rods are strongly indicative of Pseudomonas aeruginosa and should necessitate treatment. Immediate initiation of combined topical and subconjunctival therapy with an animoglycoside antibiotic such as gentamicin or tobramycin is advised. Topical steroids are sometimes used to reduce ocular inflammation. Pseudomonas aeruginosa is a Gram-negative rod; it is a non-lactose fermenting, oxidase-positive motible bacteria. Growth on MacConkey agar is usually characterized by the production of a "grape-like" smell. A blue-green color, due to the production of the diffusible fluorescent pigments pyoverdin and pyocyanin, is characteristic of the colonies growing on MacConkey. Haemophilus aegyptius is indigenous to humans. It is an important cause of purulent conjunctivitis called "pink eye", and it can occur in outbreaks because of its contagious nature. The infection is not acute in presentation. The diffuse pink color of the sclera and the presence of a serous or purulent discharge are virtually diagnostic of Haemophilus aegyptius infection. Leukocytosis is absent. The infection is not acute in presentation. The treatment of Haemophilus aegyptius is with topical antibiotics. Because of the infectious nature of the infection, instructions should be provided to the patient to help prevent the spread of the infection to others. The organism is a Gram-negative coccobacillus; it is non-motile, fastidious bacteria requiring the presence of special factors for its growth on agar media. These factors are hemin and nicotinamide adenine dinucleotide, which are present in chocolate agar. Acanthamoeba is a corneal infection that occurs in healthy people and is often associated with contact lens wearers. Swimming in fresh water (where the organism is naturally found) with contact lenses can predispose the wearer to Acanthamoeba keratitis. To prevent infection, it is recommended that contact lenses be cleaned and stored with Benzalkonium chloride-preserved saline and solutions containing thimerosal with edetate. The keratitis is slow in developing and is often mistaken for herpes, bacterial, or fungal keratitis. The average delay to definitive treatment can range from days to months. Symptoms include blurred vision, conjunctivitis, tearing, severe pain to the eye, and photophobia. The keratitis achieves an advanced stage in several days to several months, and it can exhibit patchy stromal infiltrates and dendriform epithelial involvement without frank corneal ulceration in its early stages. A ring corneal infiltrate is characteristic of this keratitis in its late stages. Early diagnosis, aggressive surgical debridement, and medical management can prevent eye damage. High concentrations of topical antimicrobial drugs (1% miconazole, 0.1% propamidine isethionate, and Neosporin) for a minimum of 3 - 4 weeks is part of the antibiotic therapeutic regimen employed in the treatment of Acanthamoeba keratitis. Acanthamoeba keratitis is a free-living amebae that can cause granulomatous amebic encephalitis and keratitis. Detection is usually made by observing the free-living motile organisms in a wet prep preparation. Staphylococcus aureus is probably the 2nd most common bacterial isolate of human infections behind Escherichia coli and the most common cause of bacterial endophthalmitis. Conjunctivitis caused by Staphylococcus aureus is usually characterized as non-severe where there is little to no lid edema, scant purulent discharge, and normal cornea; however, in some cases, the presentation can be severe. Topical agents are usually used to treat this infection such as cephalosporins or semisynthetic penicillins. In suspected case of resistance, topical vancomycin should be considered. Staphylococcus aureus has a host of invasive and toxigenic characteristics that enhance the pathogenesis of the organism in the human host. Beside causing bacterial endophthalmitis, the organism has been described as an etiologic agent of many infections including but not limited to, endocarditis, septicemia, abscesses, and urinary tract infections. It is a Gram-positive staining cocci that is catalase positive and coagulase positive. Candida albicans is a yeast. Endophthalmitis due to yeast is generally a common and serious complication of intravenous drug use. Candida albicans is the most common fungal cause. It is usually of hematogenous origin, where the patient has infective endocarditis or some other infective process occurring. The symptoms are blurred vision, decreased vision, white cotton appearing exudative lesions in the choroid and retina with vitreous haziness, and eye pain. A definitive diagnosis is made by obtaining vitreous fluid for Gram stain and culture. The treatment consists of parenteral amphotericin B together with flucytosine. Intraocular amphotericin B administration as therapy is controversial. The incidence of permanent intraocular damage is high. Yeast appears on Gram stain as large Gram-positive organisms that are approximately 3 - 5 times larger than Gram-positive cocci. They are aerobic and generally grow well on most non-selective agar media.

Case Ico-delete Highlights A 37-year-old Caucasian woman swims regularly for exercise. She swims 100 laps 4-6 times per week. She starts to notice severe right ear pain. She also notes that her right ear is very itchy. She sees her family doctor and mentions her symptoms. When the doctor goes to insert the otoscope, they gently pull on her ear. This causes her quite a bit of pain. The doctor notes an inflamed external ear canal, but the tympanic membrane is normal. Question What is the most likely pathogen?

Correct answer: Pseudomonas aeruginosa Explanation This patient has signs and symptoms of otitis externa, specifically "swimmer's ear." An infection of the external auditory canal, usually due to bacteria, is called otitis externa. Symptoms include ear pain and itching. The finding of an inflamed external ear canal is consistent with otitis externa. The most likely pathogen for swimmer's ear is Pseudomonas aeruginosa (Refer to the image). Pseudomonas aeruginosa is a gram-negative aerobic rod. Other pathogens that can cause otitis externa include Peptostreptococcus, Staphylococcus aureus, Bacteroides, Proteus, and fungi. Streptococcus pneumoniae, Haemophilus influenzae, and Branhamella catarrhalis are all causes of otitis media, or inner ear infection. Corynebacterium diphtheriae does not infect the ear.

Case A 10-year-old girl presents with left ear pain. She recently returned from a trip to the beach. She denies fever or rhinorrhea. On exam, she has moderate pain with retraction of the left pinna. The left ear canal is inflamed and filled with white discharge. The tympanic membrane cannot be visualized. Question What is true regarding her likely diagnosis?

Correct answer: Pseudomonas aeruginosa is the most common pathogen. Explanation Otitis externa is an inflammation of the external ear canal. The most common pathogen is Pseudomonas aeruginosa, followed by Staphylococcus aureus and Streptococcus. It can also be caused by fungi, trauma, or dermatologic conditions, such as eczema. Otitis externa occurs when the skin in the canal breaks down and is infiltrated by bacteria, often due to excess moisture, which is why it has the name "swimmer's ear." Treatment consists of antibiotic drops, sometimes with an added topical steroid. Otitis externa—when the infection invades adjacent bone and soft tissue—is a concern in children with diabetes and immunocompromised children.

Case A 26-year-old man presents with an eye issue. He does not wear corrective lenses. The only change of lifestyle that he reports is that a few months ago, he quit his office job and began to "help out a buddy" in the construction business. On physical examination, there is a triangular fold of tissue extending from the medial conjunctiva to the cornea in both eyes. Question What is the most likely diagnosis?

Correct answer: Pterygium Explanation The clinical picture is suggestive of pterygium. The common presentation is a fleshy, triangular intrusion of the conjunctiva onto the nasal side of the cornea that is often bilateral. It is usually associated with constant exposure to sand, wind, or sunlight. Keratoconjunctivitis sicca (or dry eyes) is a condition of lacrimal gland hypofunction commonly seen in older women. Pinguecula is a yellow elevated conjunctiva nodule in the area of the palpebral fissure on the nasal side of the eye. It is common in patients over 35. Chalazion is a chronic sty. Hordeolum is an acute sty.

Case A 23-year-old man presents 2 hours after being involved in a road traffic accident in which he sustained right-sided periorbital injuries. He is seeing double; he denies headache, vomiting, and loss of consciousness. On examination, he is alert and oriented to time, space, situation, and person. His right eye deviates downwards and temporally. Question What other finding would you expect to be present in this patient?

Correct answer: Ptosis Explanation The correct response is ptosis due to denervation of the levator palpebrae superioris, which lifts the eyelid. The clinical picture is suggestive of injury to the oculomotor nerve, which is the third cranial nerve. It innervates the following muscles: Superior rectus Inferior rectus Medial rectus Inferior oblique Levator palpebrae superioris Pupilloconstrictor Corneal anesthesia with loss of corneal reflex is a result of interruption of the trigeminal nerve supply to the cornea and conjunctiva. It does not involve the oculomotor nerve. Denervation of the pupilloconstrictor causes mydriasis, not miosis. Because of the uninhibited function of the lateral rectus, denervation of the oculomotor nerve results in abduction of the eye. Causes of third cranial nerve palsy include: Intracranial and intraorbital lesions (e.g., neoplasms) Head and orbital trauma Ocular myopathies Cerebral aneurysms Transtentorial herniation

Case A 56-year-old Caucasian man presents with a 'bump' on the side of his nose. During examination the 'bump' is identified as a papule that is located on the left nostril. This papule is 3 cm in diameter, has a pearly appearance, is skin colored with smooth surfaces, and displays well defined, smooth borders. The patient states it has been present for the past 8 months, but in the last 3 months, it has started to more noticeable. The patient denies any other lesions with the same characteristics and just wants it taken care of so it's not as bothersome. He denies weight loss, night sweats, or fevers; there have been no recent changes in his appetite or sleeping issues. He is a farmer and owns over 100 acres that he plants and harvests yearly; he has done so for the 25 years. Question Considering the most likely diagnosis for this patient, what diagnostic study is most crucial to confirm this pathology?

Correct answer: Punch biopsy Explanation Basal Cell Carcinoma (BCC) is the most common form of cancer found in humans; there are around 400,000 new cases of BCC in the US annually. Described above is most likely what is classified as nodular basal cell carcinoma. The other types of BCC include ulcerating, pigmented, sclerosing, and superficial. Nodular BCC is typically described as a papule or nodule, translucent, waxy, or pearly. These may appear reddish, skin colored and may also have telangiectasia, well-defined borders, and be smooth to palpation. These lesions may develop erosions and mildly bleed. Any lesion that is suspicious to be malignant must be biopsied. Typically a suspected BCC would have a punch biopsy or shave biopsy done to help confirm diagnosis. Mohs micrographic surgery (MMS) is not correct. MMS offers superior capability of histological analysis of tumor margins while permitting maximal conservation of tissue. It is as a precise method of helping to treat skin cancer that results in the highest cure rate with maximal tissue conservation, cosmesis, and function. MMS is also indicated for basal cell cancer and is particularly useful in high-risk anatomic sites and when there is a need for maximal preservation of tissue. Excisional or full thickness biopsy is not appropriate for this type of lesion due to the pathogenesis of BCC; it would cause increased cosmetic deformity that is most likely unnecessary in this scenario. Observation only would be inappropriate and blatantly harmful practice of medicine due to the fact of all the positive components met with the ABCDE rule.

Case A 34-year-old man presents due to something being "wrong" with his left ear. He reports his hearing has been gradually declining, but most recently he noticed some discomfort and malodorous discharge draining from this ear. He denies any trauma to the ear and any symptoms in his right ear. Upon further questioning, he does admit some tinnitus and mild vertigo. He otherwise feels well. He denies nasal symptoms, headache, sore throat, and fever. His past medical history is unremarkable; he has no known medical conditions or history of surgeries; he takes no medications and has no allergies. He lives with his wife and 2 children; he works as an office manager, and he denies the use of alcohol, tobacco, and drugs. On physical exam, his vitals are normal. Examination of the left ear reveals mucopurulent drainage within the external auditory canal. The tympanic membrane is disrupted by a retraction pocket within the upper portion, with thick yellow debris and a polyp protruding from the pocket. Hearing tests are not performed. The right ear reveals mild tympanosclerosis on the tympanic membrane, but it is otherwise normal. Hearing tests are not performed. The remainder of the patient's exam is normal. Question What intervention is most appropriate in this case?

Correct answer: Refer for surgery. Explanation This patient is presenting with a cholesteatoma, which is a benign neoplasm of the tympanic membrane. It is considered a complication of chronic otitis media. The cholesteatoma is an epidermal inclusion cyst. Complications can include infection, and more significantly, erosion into bone and nerve damage. The treatment of choice is a referral to surgery. This condition could be potentially mistaken for otitis externa, for which it may be appropriate to initiate topical antibiotics. However, topical antibiotics will not help with cyst resolution, and they will not prevent the more serious complications of a cholesteatoma. If there was a secondary bacterial infection in the cholesteatoma cyst, topical antibiotics could be justified, but only in addition to surgical referral. If the growth in the patient's ear was a wart, one may consider initiation of topical imiquimod or podophyllin. However, warts can usually be distinguished by visual exam, and they would be highly unlikely to affect the tympanic membrane. These medications (and really, all medications) have no role in the treatment of cholesteatoma. Again, if the provider mistakenly diagnoses otitis externa as the cause of this patient's ear discharge, they may recommend earplugs when swimming in order to prevent recurrence. This can be a common mistake of providing treatments by phone based on the patient's report only and no physical exam. To provide no active treatment for the cholesteatoma is inappropriate. A referral for radiation is not recommended. Typically, excision is adequate treatment. A cholesteatoma is considered a benign neoplasm and does not metastasize.

Case A 41-year-old man presents for evaluation of hearing loss. He states that he is having more difficulty in his right ear than his left. He began to notice this about 6 months ago when, while talking on his cell phone, he had to routinely switch to his left ear because of difficulty understanding the words while listening with his right ear. He states that he has had ear drainage for approximately 6 months. He also states that he was in the Navy for a few years and took up scuba diving as a recreational activity. He recalls multiple ear infections during his time in the Navy. Question During the otoscope examination, you note deep retraction pockets, a white mass behind the tympanic membrane, and focal granulation at the peripheral of the tympanic membrane. What is the most definitive treatment for this patient?

Correct answer: Referral to an otolaryngologist for surgical intervention Explanation The treatment for cholesteatoma is surgical removal or marsupialization of the sac. This should be performed by a specialist. Antibiotics and/or steroids can be tried as nonsurgical measures, but the question asks for the definitive treatment. Observation and follow-up or earplugs are not recommended since the cholesteatoma may cause destruction of the middle ear ossicles and spread to the mastoid process, worsening the condition.

Case A 30-year-old man presents with recurrent vertigo. He gives a history of attacks when rising from bed in the morning and rolling over in bed. He does not have headache, earache, hearing loss, nausea, or vomiting. On examination, external auditory canals are normal. Hearing tests are within normal limits. Pulse is 72/min, and blood pressure is 120/78 mm Hg. Central nervous system examination, including higher functions and mental status, is within normal limits. Question What is most likely to be useful in treating the patient's condition?

Correct answer: Repositioning maneuvers Explanation The correct answer is repositioning maneuvers. The patient gives a classic history of benign paroxysmal positional vertigo, which most often responds to treatment with repositioning maneuvers that utilize gravity to remove the otoconia (calcium carbonate crystals) from the semicircular canals that are thought to be responsible for causing the condition1. Diuretics are used in cases of vertigo in Meniere's disease caused by increased endolymphatic pressure1. This patient does not have symptoms of Meniere's disease, which usually include hearing loss, nausea, and vomiting. Methylprednisolone is used to treat vertigo caused by vestibular neuritis which is associated with nausea, vomiting, and upper respiratory infections1. Selective serotonin reuptake inhibitors (SSRIs) are used to treat vertigo in psychosomatic disorders like major depression, anxiety, and panic disorders1. This patient does not have symptoms associated with any of these disorders. Scopolamine is an anticholinergic medication used to treat vertigo caused by motion sickness1. It causes symptomatic relief, but is not a treatment for vertigo.

Case A 68-year-old woman presents with episodic, monocular blindness lasting typically less than 5 minutes described as a curtain moving vertically over her visual field. She denies pain or other related vision symptoms. She is asymptomatic at the time that she presents to your office, and her fundoscopic exam reveals no significant abnormality. Question What is the most likely cause of the condition described?

Correct answer: Retinal artery emboli Explanation The answer is retinal artery emboli, as the diagnosis for this patient is amaurosis fugax. Amaurosis fugax is characterized by brief episodes of monocular blindness caused by retinal artery emboli, often from ipsilateral carotid disease. The majority of patients will be asymptomatic at the time that they are examined, and therefore their retinal exam may well be normal (the classically described findings of this condition, such as papilledema, cherry-red spot, and retinal pallor, are generally seen only during or immediately after an active episode). The other answers are not correct. A detached retina does not cause brief, episodic occurrences of blindness, and retinal detachment would be noted on fundoscopic exam. Retinal vein occlusion and macular degeneration do not cause the symptoms noted above, and fundoscopic exam would reveal significant abnormalities. Papilledema would typically cause bilateral episodic blindness, and fundoscopic exam would demonstrate optic disc swelling.

A 62-year-old woman presents to the emergency department with acute unilateral loss of vision for 1 hour. Fundoscopic examination demonstrates vein dilation, intraretinal hemorrhages, and cotton-wool spots with optic disc swelling. What is the most likely diagnosis?

Correct answer: Retinal vein occlusion Explanation Retinal vein occlusion results in acute vision loss with retinal vein dilation, intraretinal hemorrhages, cotton-wool spots, and optic disc swelling on fundus examination. Macular degeneration does not cause acute vision loss but progressive loss over time and does not result in the findings above. Retinal artery occlusion does not cause retinal vein dilation and causes a classic cherry-red spot at the fovea. Diabetic retinopathy does not cause the constellation of signs noted above. Retinal detachment was not found on fundoscopic examination.

Case A 2-week-old female infant is seen for her newborn well-baby exam after a normal birth and delivery. She has been nursing well, has regained her birth weight and her development appears normal for her age. Physical examination is normal with the exception that ophthalmoscopic evaluation reveals a faint white reflex in her right eye. Question What is the most likely diagnosis?

Correct answer: Retinoblastoma Explanation Retinoblastoma is the most common primary malignant intraocular tumor of childhood. It usually appears quite early in the first 5 years. Leukocoria, a white or Cat's eye reflex in the pupil is the most frequent finding. There may also be strabismus due to vision impairment. Ocular inflammation, intraocular hemorrhage, glaucoma, or heterochromia iridis may be seen. On fundoscopic exam, the tumor may appear as a small to large white mass depending on its stage. Primary treatment includes enucleation, though smaller tumors diagnosed at an earlier stage may be amenable to newer alternative treatments such as cryotherapy and photocoagulation. Though leukocoria may be seen in retrolental fibroplasia or advanced stage of retinopathy of prematurity, it is predominantly a disorder in preterm, low birth weight infants who received supplemental oxygen in the newborn period. These infants are susceptible due to the immaturity and subsequent damage of developing retinal vasculature. If the retina goes through various stages to ischemia and neovascularization, leukocoria may be seen representing retinal detachment and a subsequent membrane formation. Phakomata are retinal findings hallmarking hamartomatous disorders such as tuberous sclerosis. The distinctive ocular lesion is a yellowish multinodular cystic lesion arising from the retina or disc. Similar lesions can occur in neurofibromatosis. Retinitis pigmentosa is a progressive degeneration of the retina characterized by pigmentary changes, arteriolar attenuation, some degree of optic atrophy and progressively deteriorating visual impairment. Granularity or mottling of the retinal pigment pattern or distinctive focal pigment aggregates can be seen via fundoscopy. Retinoschisis is a congenital disorder involving splitting of the retina into an inner and outer layer. Usually, good vision is maintained. An elevation of the inner layer of the retina can be seen.

Case A 42-year-old man presents with a 4-day history of worsening headache, stuffy nose, and clear to white colored nasal discharge. He admits to facial pain, as well as a dry cough. He denies shortness of breath, abdominal pain, nausea, or vomiting. He is a non-smoker, has no significant past medical history, and is only taking acetaminophen. On exam, he has a slight fever of 99.2° F taken orally, pulse 86/min, BP 120/76 mmHg left arm sitting, and SPO2 94% on room air. Lungs are clear and abdomen normal. Nasal mucosa appears boggy, and there is tenderness with palpation over the facial bones (maxillary area). Pharynx is without exudates. Question Which of the following organisms is the most common cause of this patient's signs and symptoms?

Correct answer: Rhinovirus Explanation The condition being described in this clinical scenario is highly indicative of acute viral rhinosinusitis (acute sinusitis). Acute sinusitis clinically includes such symptoms and signs as green/yellow purulent discharge, facial pain or pressure over the affected sinus, nasal obstruction, congestion, and may also include cough, malaise, fever, or even headache. Acute sinusitis has an acute onset of symptoms, ranging from 1-4 weeks in length of duration by the time the patient presents clinically. More commonly, the origin of sinusitis is viral, and more commonly viral rhinosinusitis originates from infection of the viral organisms that cause the common cold, making the correct answer rhinovirus. Adenovirus is not the appropriate choice in this scenario. Symptoms and signs that are viral in origin will resolve as time passes and not intensify or worsen. If a case of acute Rhinosinusitis is suspected to be bacterial in origin, the most common organisms that lead to this include S. pneumoniae, H. influenzae, and Moraxella catarrhalis; viral etiologies of acute rhinosinusitis are much more commonly seen in otherwise healthy adult patients.

Case A 42-year-old man presents with a 3-day history of, as he puts it, "not being able to hear in my right ear." He is otherwise healthy and is not taking any medications. There is not a history of trauma. On physical exam, the whisper test is decreased on his right, the Weber test lateralizes to the right ear, and the Rinne test is as follows: right ear bone conduction is greater (lasts longer) than air conduction; left ear air conduction lasts longer than bone conduction. Question What is the most likely diagnosis?

Correct answer: Right ear conductive hearing loss, possibly due to cerumen impaction Explanation Hearing loss can be recognized at the bedside as either sensorineural or conductive through the Rinne and Weber tests. In this case, the Rinne test bone conduction (BC) lasts longer than air conduction (AC) in the right ear (affected ear) and the Weber test lateralizes to the right ear (affected ear). Normally AC lasts longer than BC because of the amplifying effects of the eardrum and middle ear. If BC is longer than AC, the patient is likely to have conductive deafness. Both tests in this patient indicate a conductive hearing loss in the right ear, probably produced by cerumen impaction; there is no apparent evidence of middle-ear disease. In sensorineural hearing loss, both AC and BC are equally diminished. Right ear sensorineural hearing loss due to Meniere's disease is incorrect. The patient's Rinne and Weber test results indicate conductive hearing loss confined to the right ear, not sensorineural hearing loss. There is no evidence of Meniere's disease. Left ear sensorineural hearing loss due to Meniere's disease is incorrect. This patient's hearing tests indicate a conductive hearing loss in the right ear. In sensorineural hearing loss, both AC and BC are equally diminished. Symptoms of Meniere's disease are not present. Left ear conductive hearing loss due to middle-ear disease is incorrect. The patient's hearing tests indicate a conductive hearing loss in the right ear, not the left. Right ear sensorineural hearing loss due to acoustic neuroma is incorrect. There is no evidence of sensorineural hearing loss or acoustic neuroma in this patient.

Case A 28-year-old woman presents with an itchy throat, prolonged sneezing episodes, red and watery eyes, and inflamed nasal membranes. Her temperature is normal and a throat culture is negative. She most likely has allergic rhinitis. Question Which of the following factors is a common side effect of the drug diphenhydramine?

Correct answer: Sedation Explanation Antihistamines are the drugs that have major applications in treating the symptoms of allergic rhinitis and urticaria. They may also be used in treating motion sickness and nausea. Some antihistamines are used in the treatment of insomnia because of their strong sedative properties; therefore, they also find their way into many over-the-counter sleep aids. The most common adverse effect observed with H1 receptor blockers is sedation. Other effects seen are tremors, blurred vision, lassitude, dizziness, fatigue, drying of the nasal passages, and dry mouth. Antihistamines can interact with other drugs, potentially leading to serious consequences, such as the potentiation of the effects of CNS depressants (alcohol, etc.). Monoamine oxidase (MAO) inhibitors can potentiate the anticholinergic effects of antihistamines. In spite of this, H1 receptor blockers are relatively safe. Chronic toxicity is rare; however, acute poisoning is common, especially among children, and it leads to dangerous effects, such as hallucinations, ataxia, convulsions, and if untreated, coma and cardiorespiratory collapse. Histamine is synthesized from the amino acid histidine by a decarboxylation reaction. It is found in practically all tissues; there are large amounts in the skin, lungs, and gastrointestinal tract. It is stored in high concentrations in the granules of mast cells and basophils. It is metabolized by monoamine oxidase enzymes and by methylation. Histamine is released from mast cells in response to immunoglobulin E (IgE)-mediated (immediate) allergic reactions and plays an important role in urtication, angioneurotic edema, seasonal (allergic) rhinitis (hay fever), and anaphylaxis. It may also play a role in the control of gastric acid secretions. Histamine exerts its effect by binding to 2 major types of cell surface receptors (H1 and H2). Recently, a third type (H3) has been discovered. H1 receptors are important in producing smooth muscle effects (e.g. in bronchial smooth muscle constriction leading to decreased lung capacity and symptoms of asthma, in intestinal smooth muscle contraction resulting in cramps and diarrhea, and in exocrine glands resulting in increased nasal and bronchial mucus secretion). Binding of histamine to the H1 receptor is also involved in vasodilation and increased permeability of capillaries leading to leakage of protein and fluid into tissues. Histamine binding to the H1 receptor causes bronchoconstriction by stimulating the activity of the polyphosphoinositol cycle (IP3 release) and vasodilation, probably by releasing endothelium-derived releasing factor (EDRF). The H2 receptor is involved in stimulating gastric acid secretion from parietal cells in the gastric mucosal glands. It also has a cardiac stimulatory effect. H2 receptor activity is expressed by activation of adenylate cyclase and concomitant increased cAMP levels. The H3 receptor seems to be active mainly in the CNS, where it may be involved in modulating histamine neurotransmission at some presynaptic membranes. Histamine itself has no therapeutic applications, but compounds that block its actions at the H1 receptor (H1 receptor antagonists, antihistamines) are very important clinically. There are many kinds of H1 blockers from several different chemical classes. Refer to the image. H1 blockers do not have any effect on the formation or release of histamine from storage sites. They compete with histamine for receptor sites on target cells. H1 receptor blockers do not serve as histamine agonists due to the differences between their structures. Some H1 blockers may also show some affinity for the serotonin, adrenergic, and cholinergic muscarinic receptors. H1 blockers competitively inhibit the effects of histamine. All show varying degrees of sedative and anti-motion sickness effects in the CNS. Since some may act at other receptors (noted above), they may block muscarinic, α-adrenergic, and serotonin receptor-mediated effects

Question Using the Weber and Rinne tests, a left-sided conduction hearing loss will manifest as what?

Correct answer: Sound lateralization to the left in the Weber test; left-ear sound longer through bone in the Rinne test Explanation In the Weber test, sound lateralizes to the impaired ear in conduction hearing loss, and to the good ear in unilateral sensorineural hearing loss. In the Rinne test, sound is heard longer through bone in conduction hearing loss, and it is heard longer through air in sensorineural hearing loss. Therefore, in left-sided conductive hearing loss, sound will lateralize to the left in the Weber test, and left-ear sound will be heard longer through bone in the Rinne test.

Case A 69-year-old man presents to his physician with a 3-month history of white oral lesion. Exam findings are suspicious for oropharyngeal cancer. Question What is the most common type of oropharyngeal cancer?

Correct answer: Squamous cell carcinoma Explanation Squamous cell carcinoma is the most common form of oropharyngeal cancer, accounting for 90% of cases. Adenocarcinoma, basal cell carcinoma, papillary carcinoma, and medullary carcinoma are all incorrect

Question What is the most frequent malignant neoplasm involving the larynx?

Correct answer: Squamous cell carcinoma Explanation The correct response is squamous cell carcinoma. Because the tissue of the larynx includes both mesenchymal and epithelial elements, many types of carcinomas and sarcomas may arise in this location. While adenocarcinomas, fibrosarcomas, and even metastatic tumors may be seen, by far the most common malignant neoplasm is squamous cell carcinoma. Squamous cell carcinomas comprise >90% of all neoplasms in this location. The tumors arise from the squamous epithelium, which lines the larynx, and are most likely preceded by squamous atypia and dysplasia. Lesions may arise anywhere within the larynx. Lesions of the true vocal cord are the most common in the US and are more likely to be detected early because they cause hoarseness. Squamous cell carcinomas are more common in men, with cigarette smoking being a major contributor to their development. Squamous papillomas are exophytic benign neoplasms that may arise in any age group.

Case An 8-year-old boy presents in January with a fever, pharyngeal erythema, tonsillar exudates, and enlarged cervical lymph nodes. He does not have a cough; his tympanic membrane is normal and does not show any signs of conjunctivitis. Question What is the most likely diagnosis?

Correct answer: Strep throat Explanation Group A streptococcal pharyngitis (strep throat) is caused by Streptococcus pyogenes, which are gram-positive coccoid bacteria that grow in chains. This bacterium also causes scarlet fever and impetigo. If the patient is not allergic to penicillin, this condition can be effectively treated with amoxicillin. There are no pathognomonic signs or symptoms of a strep throat. Using a scoring system that includes: age 5-15 years old, presenting between November and May, fever greater than 38.3°C, pharyngeal erythema, swelling or exudate, tender cervical lymph node enlargement greater than 1 cm, and the absence of cough, coryza, and conjunctivitis has a positive predictive value of 75%. Impetigo is also caused by the same bacterium but would involve crusty skin lesions, often on the face. Viral influenza would not present with much in the way of a fever. Otitis media is an infection of the middle ear cavity and can be detected by examination of the tympanic membrane. A child with a pertussis infection would likely have a cough.

Case A 22-year-old woman presents because she developed a fever of 101 degrees this morning. She has a 2-week history of rhinorrhea, congestion, and headache. She states that her rhinorrhea was initially clear and actually improved after 5 days, but then returned and developed into a green color. Her headache is felt in the forehead and cheeks and worsens when she bends over. Question What is the likely causative pathogen of her illness?

Correct answer: Streptococcus pneumoniae Explanation The correct answer is Streptococcus pneumoniae. Given her symptoms of rhinorrhea accompanied by sinus pressure, she is likely suffering from sinusitis. While viruses cause the majority of cases of sinusitis, the patient's clinical picture is more suggestive of a bacterial etiology for the following reasons: first, her duration of symptoms is longer than 10 days; second, her clinical condition improved then worsened again, suggesting that she may initially had a virus which allowed for a secondary bacterial infection to develop; and third, fever and purulent nasal discharge are present, both of which are more likely with bacterial sinusitis. Of the above bacterial causes, Streptococcus pneumoniae is the most common causative agent. Other streptococci species, as well as Haemophilus influenzae, are also common causes of bacterial sinusitis. Rhinovirus and adenovirus are both incorrect. While most cases of sinusitis are viral in nature, the patient's clinical picture is more suggestive of a bacterial cause. Pseudomonas aeruginosa is not a common cause of sinusitis. Staphylococcus aureus may cause sinusitis, but Streptococcus pneumoniae is more common.

Case A 20-year-old man with no significant past medical history presents with a severe sore throat for the last hour. He reports associated fever, difficulty swallowing, difficulty in opening his mouth, excessive salivation, and a self-described "raspy, harsh voice." He denies chills, sick contacts, otalgia, myalgias, malaise, rashes, wheezing, shortness of breath, and cough. He also denies smoking, drinking, drug use, or recent sexual activity. His physical exam is notable for a toxic-appearing young male patient with tonsillar hyperemia, swelling of the anterior pillar and soft palate, tender cervical lymphadenopathy, and a small left-tonsillar abscess. The primary care provider performs an oropharyngeal culture and refers the patient immediately to the emergency room for further treatment. Question What is the most likely agent isolated from the oropharyngeal culture performed by the provider?

Correct answer: Streptococcus pyogenes Explanation The correct response is Streptococcus pyogenes. This patient's presentation is significant for a peritonsillar abscess. Group A hemolytic streptococci (often as part of a mixed flora containing anaerobes) are most commonly isolated. Most common strains are Streptococcus pyogenes (group A streptococcus [GAS]), Streptococcus anginosus, Staphylococcus aureus (including methicillin-resistant S. aureus [MRSA]), and respiratory anaerobes.

Case A 9-year-old boy presents to the emergency department with acute onset of 90 minutes of fever, sore throat, and labored breathing. IV access and oxygen therapy are both initiated in preparation for diagnostic testing. The physician assistant in the emergency department is concerned that he might have epiglottitis. Question What finding on lateral neck radiograph would support this diagnosis?

Correct answer: Thumb-like projection superior to the larynx Explanation Thumb-like projection superior to the larynx is the correct answer, as this is the description that is often used for an edematous epiglottis seen with epiglottitis. The epiglottis is a flap of tissue at the entrance of the larynx that points upwards while a person is breathing and folds down while swallowing. This mechanism prevents food from entering the trachea rather than the esophagus while eating. Epiglottitis can be life threatening and is an infection of the epiglottis that can lead to obstruction of the airway. It is commonly caused by H. influenza type B, group A Streptococcus, pneumococcus, or staphylococci. The classic thumb sign is usually seen on a lateral neck radiograph, which is essentially a swollen epiglottis seen on X-ray. Narrowing of the trachea is not the correct answer, as this is not typically seen on the neck radiograph of a patient with epiglottitis. A patient with croup (an upper respiratory viral illness characterized by a barking cough, stridor, and fever) can have narrowing of the trachea on a lateral neck radiograph. Foreign body in the trachea is not the correct answer. A lateral neck radiograph would show a foreign body in the trachea if one was lodged there, and this would not be associated with epiglottitis. Soft tissue mass in the nasal cavity is not the correct answer. The epiglottitis is soft tissue, but is not located in or near the nasal cavity. A patient with a nasal polyp may have a soft tissue mass in their nasal cavity on lateral neck radiograph. Narrowing of the esophagus is not the correct answer. Narrowing of the esophagus can occur with an esophageal stricture, but is more commonly diagnosed with a barium swallow or endoscopy. Narrowing of the esophagus can be seen on a neck radiograph in a patient with esophageal cancer, as well.

Case A 62-year-old man presents with a 1-month history of mouth pain and non-healing white patches on his mouth. He denies infection and recent trauma to the area. Past medical history is significant for diabetes mellitus, hypertension, osteoarthritis, and gonorrhea, which was treated 6 months ago. Social history is positive for a 20-pack year smoking history and cocaine abuse; he quit both approximately 10 years ago. On examination, leukoplakia is present, as seen in the image. The patient is referred for biopsy. Question What is the strongest risk factor for the most likely diagnosis?

Correct answer: Tobacco use Explanation Tobacco use and alcohol use are the strongest risk factors for oropharyngeal cancer. The patient has non-healing areas of leukoplakia, which is suggestive of malignancy. Squamous cell carcinoma is the most common type of oropharyngeal cancer, and approximately 75% of patients will have a history of tobacco and/or alcohol abuse. Oropharyngeal cancer is more common with increased age and male gender, but tobacco use is a stronger risk factor. Recent history of gonorrhea is incorrect. While human papillomavirus (HPV) is strongly associated with oropharyngeal cancer, gonorrhea is not a risk factor. Cocaine use is not associated with the development of oropharyngeal cancer.

Case A 48-year-old Caucasian man presents with acute onset of blurring of vision and severe pain in the left eye that began 30 minutes ago. He notes seeing halos with his left eye; he is also experiencing nausea and vomiting; these symptoms started at the same time as the pain. The patient reports that he was relaxing on his porch when the pain started. His temperature is 36.9°C, pulse 90/min, BP 130/90 mm Hg, and respirations 20/min. Physical examination reveals a shallow anterior chamber, a hazy cornea, a fixed, moderately dilated pupil, and ciliary injection. Question What would be the next step in the management of this patient?

Correct answer: Tonometry Explanation The correct response is tonometry. The history and physical examination of this patient are suggestive of acute angle-closure glaucoma, which can be easily confirmed by measuring the intraocular pressure using a tonometer (e.g., the Schiötz tonometer). Acute angle-closure glaucoma develops in individuals with pre-existing anatomic narrowing of the anterior chamber, which is seen mainly in far-sighted people. The condition usually develops in the twilight hours, which is when the pupil is dilated in response to the low level of illumination. It may also occur with pupillary dilation for ophthalmoscopy, so topical atropine would be contraindicated. Acute angle-closure glaucoma is an ophthalmologic emergency. Treatment involves immediate lowering of the intraocular pressure via systemic acetazolamide to decrease the production of aqueous humor, which should be supplemented with topical hyperosmotic agents and topical beta-blockers. Topical pilocarpine is then used to cause miosis. Once the intraocular pressure is under control, a peripheral iridectomy can be done to prevent against future recurrences.

Case A 22-year-old man presents to his primary care physician with a painful mouth ulcer. He notes that he had similar ulcers once in the past; they resolved spontaneously without treatment. Examination reveals the following image. Question What is the recommended treatment?

Correct answer: Topical triamcinolone Explanation Topical triamcinolone is the correct response. The patient has an aphthous ulcer. The cause is unknown, but there is a possible association with human herpesvirus 6. They are often recurrent, and they usually resolve within 14 days. Treatment is nonspecific; however, topical corticosteroids, such as triamcinolone, may provide relief of pain. Oral prednisone is an incorrect response. While prednisone may be effective, it is typically reserved for patients with large or multiple aphthous ulcers or those failing topical therapy. Oral amoxicillin is an incorrect response. Aphthous ulcers are not caused by bacterial infection, so antibiotic treatment would be inappropriate. Topical nystatin is an incorrect response. Nystatin would be useful in the treatment of candidiasis, not aphthous ulcers. Oral cimetidine is an incorrect response. Cimetidine may be used as maintenance therapy in patients with recurrent ulcers, but it would not be recommended at this time.

Case A 32-year-old man presents with chronic runny nose and congestion; according to the patient, it seems to come and go. He reports getting similar symptoms each fall, which is when he goes hunting. His symptoms are just starting again this year. He describes the nasal discharge as clear and watery. He often awakes with congestion. He admits to some sneezing, "itchy nose", and an infrequent dry cough. He denies fever/chills, headache, itchy or watery eyes, and productive cough. He has tried over-the-counter allergy pills; however, they cause excessive sedation. His past medical history is unremarkable. He has no known medical conditions. He is not currently taking any medications; he has no known drug allergies, and he has not had any surgeries in the past. He lives with his girlfriend and works in customer service; he denies the use of alcohol, tobacco, and drugs. On physical exam, his HEENT exam is remarkable for violaceous nasal turbinates bilaterally, with clear nasal discharge. The rest of his exam is normal. Question What test is most appropriate for confirming this patient's current condition and symptoms?

Correct answer: Trial of intranasal corticosteroid Explanation This patient is suffering from seasonal allergic rhinitis, and he is experiencing primarily nasal symptoms. With a history of cyclic symptoms, including nasal congestion, rhinorrhea, pruritus, and clear nasal discharge, but no fevers or symptoms consistent with viral or bacterial sinusitis, the diagnosis can be made without tests. A trial of an intranasal corticosteroid would basically serve as the 'test' that would confirm a diagnosis of allergic rhinitis. Nasal corticosteroids shrink hypertrophic nasal mucosa and are considered a first-line treatment. A complete blood count (CBC) with differential may have elevated eosinophils, indicating an allergic response. However, the eosinophil level may be normal or elevated in individuals with allergic rhinitis. A CBC would not be recommended. Computed tomography (CT) of the sinuses might be useful in a patient with chronic sinusitis or headaches attributed to possible chronic sinusitis. CT is neither recommended nor confirmatory in cases of allergic rhinitis. Culture and sensitivity of nasal secretions are occasionally used in nasal/sinus infections, but are often plagued by contamination. The culture would not be useful in cases of allergic rhinitis. Sweat chloride testing is used to detect cystic fibrosis (CF), a chronic and progressive lung disease. CF does not typically produce nasal symptoms; this patient's history and physical do not indicate lung disease.

Case A 52-year-old man presents for a follow-up visit. He saw you about 2 weeks ago with the chief complaint of losing his voice. The diagnosis of acute laryngitis was made at that time, and supportive treatment was described in detail to the patient. The patient returns a little worried today because he has had no improvement in getting his voice back since his last visit. The patient is a professor at a local university and desperately needs his voice to return in order to lecture properly. Social history reveals that he has been a 1 pack per day smoker for the past 32 years. Further questioning of the patient during your review of systems reveals a 5-pound unintentional weight loss since his last visit. Question Ico-delete Highlights What would be the next course of action based on the history and physical examination?

Correct answer: Urgent referral to an otolaryngologist Explanation Acute laryngitis, as in the patient scenario above, is most common cause of hoarseness in patients of all ages. More commonly acute laryngitis symptoms will last a week or more even after the resolution of an upper respiratory infection. Typical treatment at that point in time includes avoidance of vigorous use of his/her voice (such as singing or shouting) until it returns to normal. Often the source of acute laryngitis is viral in origin, but there has been incidence of M. catarrhalis and H. influenzae isolated from the nasopharynx of these patients. For this reason, antibiotics are occasionally prescribed to lessen the severity of hoarseness and cough. Certain age groups are risk factors and should lead a provider to a more aggressive search for the diagnosis of acute laryngitis. New or longer than 2-weeks duration of hoarseness in a patient who is known to be a heavy smoker (and especially in a patient who is male) is one of those red flags. This should sway one to contemplate squamous cell carcinoma of the larynx until it can be proven otherwise. Squamous cell is the most common malignancy of the larynx and appears exclusively in patients with a history of significant tobacco use, especially men aged 50-70 years. Our patient above substantially fits into this category. Another disturbing component is his unintentional weight loss that is revealed during the review of system questioning; this is of course a constitutional symptom/sign that cannot be missed. For the reasons listed above, the correct answer is an urgent referral to an otolaryngologist for initiation of evaluation to confirm the suspected diagnosis. Further observation has an extremely low probability of bringing resolution, as is also the case with reassurance and observation. A pulmonology referral is not indicated at this time, and although the patient has the potential of having laryngopharyngeal reflux, the pertinent positives at this point more towards a type of malignancy.

Case A 6-year-old boy presents with a right earache and yellowish-white drainage. On exam, his temperature is 99.8°F, respiration is 20/min, and lungs are clear. His neck on palpation has 2+ slightly tender, right superior-anterior cervical nodes. He also has a few small, healed scars on his right neck. His mother tells you that they just got a kitten. His left ear is clear, but his right ear has a foul-smelling greenish-white discharge; after gentle curettage, it becomes bloody. The canal is swollen, erythematous, and tender to tragal pressure. He is treated with a course of amoxicillin and ofloxacin drops, and he is asked to return in 1 week for follow-up. Upon follow-up 1 week later, the boy continues to present with persistent otorrhea and mild intermittent otalgia. The exam is again normal, except for the right anterior cervical nodes, which now seem slightly more enlarged and still slightly tender. The right ear has copious amounts of light greenish, purulent, blood-streaked discharge. Repeat lavage and ear exam reveal a small dark shiny area and something that looks dark, round, and cylindrical. The patient has been fairly cooperative during the exam and ear lavages. Question What is the most appropriate next course of action?

Correct answer: Use fine alligator forceps to attempt removal of a possible foreign body. Explanation The correct response is to use fine alligator forceps to attempt removal of a possible foreign body. The presence of persistent otorrhea and a suspicious round object in the ear canal of a 6-year-old suggests a possible foreign body. In a fairly cooperative patient, it would be prudent to attempt careful extraction under direct observation. Gentle flushing with warm water or saline may sometimes be all that is necessary; this may move the object closer to the outer ear and allow easier manual extraction or better visualization. The reactive lymph nodes are in response to the local infection and inflammation. Continuation of amoxicillin and ofloxacin drops is not a good choice since the patient's symptoms are getting worse and there is an unexplained foreign body present on exam. The presence of regional lymphadenopathy in a patient known to have received cat scratches can be indicative of cat scratch disease (CSD). Infection with the gram-negative rod B. henselae is one of the most common causes of chronic lymphadenopathy in children. The most typical presentation in an immunocompetent patient is regional lymphadenitis (progressing for several weeks) with enlarged lymph nodes that may persist for months before spontaneous regression. 30-50% will have a fever. Laboratory evaluation consists of testing serum for antibodies to B. henselae by indirect immunofluorescence antibody or enzyme-linked immunosorbent assay. Most immunocompetent patients with CSD do not require treatment. Significant improvement in the course of illness has not been demonstrated with antimicrobial treatment. Atypical presentations may include Parinaud oculoglandular syndrome; this syndrome consists of unilateral conjunctivitis with ipsilateral lymphadenopathy of pre-auricular or mandibular nodes. Other atypical presentations are endocarditis, neuroretinitis, hepatitis, hepatomegaly, osteomyelitis, atypical pneumonia, fever without adenitis, and encephalopathy. Testing for CSD might be eventually considered for this patient if the lymphadenopathy persists, but treatment for the persistent otorrhea and otalgia needs to be addressed first. Prescribing a course of a different antibiotic could be attempted, but the patient has already gone through a course of amoxicillin and ofloxacin, which should provide coverage against the most commonly encountered pathogens. In addition, it does not address the unexplained foreign body observed in the examination of the ear canal. Obtaining a culture of the purulent drainage is not likely to reveal significant bacterial growth after treatment with antibiotics. It also would not address the unexplained foreign body observed in the examination of the ear canal.

Question A 15-year-old boy presents to your office for evaluation of a lesion on the side of his upper lip. The lesion appears on the outer edge of the upper lip. It appears as a small, well-circumscribed elevation with crusting. You tell the patient that the diagnosis is herpes simplex. What type of lesion is this?

Correct answer: Vesicle Explanation A vesicle is a circumscribed elevation of the skin less than 1 cm in diameter. It contains fluid. Bullae are larger than vesiclesand are produced by factors that include chemicals, friction, and heat. A nodule is a solid lesion larger than 1 cm in diameter. It consists of inflammatory cellular infiltrates or neoplasms. Pustules are circumscribed accumulation of pus in the skin. The lesion may or may not be raised. Tumors vary in size and are new skin growths composed of skin and subcutaneous tissue. They may be malignant or benign.

Case A 22-year-old woman with a history of gradual right-sided hearing loss presents seeking help. She has been experiencing some ringing in her right ear, vertigo, and a feeling of fullness in the ear. Upon physical exam, the patient appears frustrated and keeps tugging on her right ear. MRI displays a tumor that has developed in the right inner ear. Question What is the most likely explanation of the findings?

Correct answer: Vestibular schwannoma Explanation The clinical picture is suggestive of vestibular schwannoma, as she has symptoms of gradual right side hearing loss, vertigo, and the feeling of fullness in the ear. Additionally, an MRI confirmed a tumor in the right inner ear. Labryinthitis is inflammation of the inner ear or nerves with unilateral loss of auditory function. Labrynthitis is usually associated with an ear infection that is caused from bacteria or a virus; however, this patient presents with a tumor on her MRI and this helps rule out labryinthitis. Multiple sclerosis is associated with numbness, blurred vision, and weakness, and is therefore not the correct answer to this problem. Vascular compromise is blocked blood flow to an organ caused by a blocked vessel, and is not the correct answer to this problem. Vestibular neuronitis is not the correct answer to this problem, as loss of hearing is not associated with vestibular neuronitis and the patient in the stem of this problem has presented with hearing loss.

Case A 52-year-old man presents with concerns over hearing changes. He has noticed a decreased ability to hear sounds for the past few months; he tested it at home by covering each ear, and he now thinks there is a hearing loss in only 1 side (his left). Furthermore, he hears a ringing sound all the time. He is a business manager, and he denies occupational exposure to loud noises. He denies head trauma, headaches, and prior ear problems. His wife thinks that he is experiencing normal age-related hearing loss. His review of systems is negative for other neurological symptoms. The patient's past medical history is unremarkable; he has no known medical conditions. He takes no medications. He has no allergies, and he has not had any surgeries. He denies alcohol, tobacco, and drug use. On physical exam, his vitals are normal. His HEENT exam is significant only for decreased auditory acuity and Weber test lateralizing to the right. Audiometry confirms a sensorineural hearing loss on the left. An MRI is performed, and it shows a well-delineated intracranial mass. Further investigation reveals the origin of cells is from Schwann cells. Question What structure is likely involved in this patient's condition?

Correct answer: Vestibulocochlear nerve Explanation This patient is presenting with a vestibular schwannoma (or acoustic neuroma), affecting his vestibulocochlear nerve (cranial nerve VIII). This is one of the more common benign head and neck neoplasms. A common presentation is unilateral hearing loss and tinnitus. An understanding of basic head anatomy can help the clinician appreciate the affected structures. The foramen ovale is an oval-shaped opening in the sphenoid bone, through which the mandibular nerve (the 3rd division of the trigeminal nerve) passes. The internal carotid artery is one of the major blood supplies to the head. Part of its path lies near the ear structures, but an intracranial mass affecting the internal carotid artery would not produce the characteristic sensorineural hearing loss. The 3 semicircular canals are openings that are set in right angles to each other; they are found in the bony structure of the ear, which communicate with the vestibule and are partly responsible for balance. The patient's history, exam and tests do not suggest the tumor is affecting the semicircular canals. The tensor tympani are short (about 2 cm long) muscles in each ear, which pull on the malleus and tighten the tympanic membrane. Their role is to prevent hearing damage when exposed to loud sounds. They are innervated by the mandibular nerve, which is a branch of the trigeminal nerve.

Case A 6-year-old girl presents with a 2-day history of a small, tender, superficial abscess on her left upper eyelid at the lid margin. She denies discharge, fever, or trauma. There is mild pain on palpation, and she has a normal ophthalmic exam. Question What treatment should be given?

Correct answer: Warm compresses Explanation This description is of a hordeolum, which is also called a stye. The usual causative agent is Staphylococcus aureus. Treatment involves warm compresses and frequently a topical ophthalmic antibiotic. Occasionally, surgical incision and drainage are required. There is no need for a systemic antibiotic for a minor infection, and the causative agent does not appear to be allergic or viral. A stye can be confused with a chalazion, which is a granulomatous inflammation of a meibomian gland characterized by a nontender nodule. The lesion tends to be chronic, and it shows no sign of acute inflammation

Case A 25-year-old man presents with a 1-day history of redness, itching, and swelling in the right upper eyelid. The redness has increased; there has been no eye discharge or watering. On examination, a pustular swelling at the root of the lashes is found. It is surrounded by redness, and it is tender on palpation. The lesion is shown in the image. Question What is the appropriate management of this patient?

Correct answer: Warm compresses and local antibiotic cream Explanation There are 2 types of hordeolum. The first, external hordeolum, is an acutely inflamed stye is caused by infection of gland of Zeis (sebaceous glands near the eyelashes) or Moll (apocrine glands of the lid margin). It causes redness, irritation, itching, and then a white dot surrounded by erythema appears at the lash root. Warm compresses plus erythromycin cream is usually effective in most cases. The other type is internal hordeolum caused by lipogranulomatous inflammation of the meibomian gland of the tarsus. It is caused by obstruction of the gland orifice and causes a lid swelling that typically diminishes in size with voluntary squeezing of the eyelid. It may be acutely inflamed and causes diffuse lid swelling and erythema. Commonly there is remission and exacerbation. The treatment of the acute form is systemic antibiotics, hot fomentation, and topical erythromycin cream, while chronic persistent form usually requires surgical removal and curettage from the conjunctival side. Blepharitis is inflammation of the lid margin either due to allergy or infection. It is mostly associated with dry secretions, scales, redness, and there may be itching. Infective cases are treated with antibiotic drops, and allergic blepharitis is usually treated with local steroids. Conjunctivitis is inflammation of the conjunctiva and is always associated with red eye and copious eye discharge. Topical antibiotics are used to treat infective conjunctivitis, while steroids and mast cell stabilizers are used to treat allergic types. Herpes zoster ophthalmicus is an infection due to herpes zoster affecting the ophthalmic division of trigeminal nerve. It causes rash on the upper eyelid, which is extremely painful, unilateral, and may affect the cornea, causing keratitis that presents with photophobia, lacrimation, and severe pain. It is typically treated with topical antivirals and steroids. Systemic antivirals are sometimes used in this case. Antifungal cream is not useful in this patient, as external hordeolum is a bacterial infection.

Case A 25-year-old man presents to you with an acute otitis media with serous otitis in the right ear. You perform the Weber and Rinne tests. Question Ico-delete Highlights What result would you most likely expect to find?

Correct answer: Weber - sound is heard louder in right ear, Rinne - bone conduction exceeds air conduction in right ear Explanation Otitis media and serous otitis are examples of causes of conductive hearing loss. When a conductive hearing loss exists, the Weber test will result in the appearance of a louder sound in the affected ear, and the Rinne test will result in bone conduction exceeding air conduction in the affected ear. The other answers are incorrect; in a sensorineural hearing loss, the Weber test results in a louder sound in the unaffected ear and the Rinne test will result in air conduction exceeding bone conduction in the affected ear.

Case A 24-year-old man with no significant past medical history presents with a 3-month history of progressive hearing loss of his left ear. He states that he has the greatest difficulty in hearing high-pitched sounds. He denies trauma, recent travel, sick contacts, pressure changes, headache, nausea, otorrhea, otalgia, fever, chills, vertigo, swollen glands, rashes, sore throat, vision changes, or rhinitis. He states that he enjoys listening to music on his personal music device and attends concerts frequently. An otoscopic speculum exam reveals no observable abnormalities. Question What diagnostic test would be most useful in identifying the cause of this patient's hearing loss at this time?

Correct answer: Weber test Explanation This patient's presentation is most consistent with sensorineural hearing loss due to exposure to loud noise. Tuning forks are useful in differentiating conductive from sensorineural hearing losses. In this patient, the Weber test is expected to reveal a sound that lateralizes to the good or unaffected ear, as cochlear nerve damage impairs transmission to the affected ear. The following, confirmatory Rinne test will demonstrate air conduction greater than bone conduction, the normal pattern. The normal pattern prevails since the cochlear nerve is less able to transmit impulses regardless of how the vibrations reach the cochlea. Collectively, the Weber and Rinne tuning fork tests and a fistula test using pneumatic speculum must be performed. Audiometry is important to perform and includes pure-tone, speech tests, and immittance (tympanometry and acoustic reflex) tests. A cochlear biopsy is invasive and not routinely performed. Facial X-rays have no role in the workup of hearing loss. According to 2012 guidelines from the American Academy of Otolaryngology-Head and Neck Surgery Foundation, computed tomography scans are not helpful and expose the patient to ionizing radiation. CT scans are therefore not recommended in the initial evaluation of patients with presumptive sensorineural hearing loss. Routine, non-targeted laboratory testing is not recommended.

Case A 52-year-old man presents with concerns over hearing changes. He has noticed a decreased ability to hear sounds for the past few months; he tested it at home by covering each ear, and he now thinks there is a hearing loss in only 1 side (his left). Furthermore, he hears a ringing sound all the time. He is a business manager, and he denies occupational exposure to loud noises. He denies head trauma, headaches, and prior ear problems. His wife thinks this is just normal age-related hearing loss. His review of systems is negative for other neurological symptoms. The patient's past medical history is unremarkable; he has no known medical conditions. He takes no medications. He has no allergies, and he has not had any surgeries. He denies alcohol, tobacco, and drug use. On physical exam, his vitals are normal. His HEENT exam is significant only for decreased auditory acuity and Weber test lateralizing to the right. Audiometry confirms a sensorineural hearing loss on the left. An MRI is performed, and it shows a well-delineated intracranial mass. Further investigation reveals the origin of cells is from Schwann cells. Question After completing treatment for his current condition, what is the best approach for health maintenance of this patient?

Correct answer: Yearly head imaging Explanation This patient is presenting with a vestibular schwannoma (or acoustic neuroma), affecting his vestibulocochlear nerve (cranial nerve VIII). This is one of the more common benign head and neck neoplasms. A common presentation is unilateral hearing loss and tinnitus. Treatment is typically surgical removal, or possibly radiation therapy. After completing therapy, yearly head imaging is recommended to monitor for recurrence. Full body bone scans every 2 years are not recommended for vestibular schwannomas; they are not likely to metastasize to bone. Prophylactic antibiotics have no role in the original formation or the recurrence of schwannomas. Some other benign neoplasms in the ear, such as cholesteatomas, are associated with chronic ear infections, but antibiotics are still not helpful after treatment of those growths either. Serum blood tests of inflammatory markers every 6 months are not helpful for schwannoma monitoring. Schwannomas are not considered an inflammatory disease and typical serum markers (erythrocyte sedimentation rate and C-reactive protein) may be entirely normal in affected individuals. Testing only if the patient reports return of symptoms is also not appropriate, because of the usual slow-growing nature of schwannomas. Because of very slow hearing loss or balance problems (if the vestibular portion of the nerve is affected), patients with schwannomas may adapt and easily miss these symptoms for years.

Case A 40-year-old man with a past medical history of hepatitis C presents with burning and pain of his oral cavity, which has been associated with a pruritic rash of the flexor aspect of his left wrist. The physical exam is remarkable for violaceous, shiny, and polygonal papules arranged as lines and circles on his wrist. These papules range in size from 1 mm to 1 cm in diameter, and have fine, white lines on them. In the oral cavity, a reticular, white, lacy pattern is visualized, as seen in the image. Question What is the most appropriate first-line pharmacological treatment of this patient's oral lesions?

hide Correct answer: Triamcinolone Oral Paste (Kenalog in Orabase) Explanation This patient most likely has lichen planus with cutaneous and oral involvement. It is an inflammatory mucocutaneous condition that usually exhibits a distinctive morphology and is associated with hepatitis C. The classic appearance of skin lesions includes violaceous polygonal flat-topped papules and plaques, commonly occurring at the wrist. For lichen planus of the oral mucosa, topical steroids are usually tried first. Treatment with steroids is especially indicated in ulcerative forms of oral lichen planus. Topical and systemic cyclosporin has been tried with some success; however, a randomized double-blind study indicated that topical cyclosporin was a less effective and much more costly regimen than clobetasol.Newer topical calcineurin inhibitors have replaced topical cyclosporin for the treatment of lichen planus. Acitretin has been reported to be effective for the treatment of severe cutaneous disease and is not used for oral lichen planus. Nystatin and vibramycin suspensions are not indicated, as this patient is not experiencing a fungal or bacterial infectious process.

Case A 28-year-old woman with a past medical history of well-controlled asthma presents with recurrent sneezing episodes, nasal itching and congestion, as well as headache. Her physical exam is remarkable for post-nasal drip, a transverse nasal crease, and bilateral infraorbital cyanosis. An internal nasal speculum exam reveals the attached findings. Question What is correct regarding the diagnostic evaluation of this patient?

orrect answer: Sensitivity to possible etiologies is determined with skin testing Explanation Sensitivity to possible etiologies is determined with skin testing. Allergy skin tests (immediate hypersensitivity testing) are an in vivo method of determining immediate (IgE-mediated) hypersensitivity to specific allergens. Sensitivity to virtually all of the allergens that cause allergic rhinitis can be determined with skin testing. This patient's past medical history and current presentation are remarkable for allergic rhinitis. Signs and symptoms include sneezing paroxysms, nasal, ocular, or palatal itching, clear rhinorrhea, nasal congestion, pale, bluish nasal mucosa, transverse nasal crease, infraorbital cyanosis ("allergic shiners"), and serous otitis media. While patients with allergic rhinitis are more likely to have an elevated total IgE level than the normal population, this test is neither sensitive nor specific for allergic rhinitis. Up to 50% of patients with allergic rhinitis have normal levels of total IgE, while 20% of nonaffected individuals can have elevated total IgE levels. As is the case with total serum IgE, an elevated eosinophil count supports the diagnosis of allergic rhinitis, but it is neither sensitive nor specific for the diagnosis While radiographic studies are not needed to establish the diagnosis of allergic rhinitis, they can be helpful for evaluating possible structural abnormalities or to help detect complications or comorbid conditions, such as sinusitis or adenoid hypertrophy.

Case An 18-year-old man presents with blurred vision and some eye pain that began 2 days ago and has become progressively worse. Upon examination, the eye is slightly edematous with a white to yellow exudate present under the eyelid and at the corner. The rest of his clinical and physical history is unremarkable. A conjunctival scraping is obtained and gram stained. Based on the gram stain result, the conjunctival scraping was sent to the laboratory for culture and sensitivity. The patient is given instructions for topical antibiotic ointment treatment (polymixin B/trimethoprim) to be administered every 2-4 hours for 7-10 days. Pathology later shows that the conjunctival scraping culture grew out a beta hemolytic organism that was catalase positive, coagulase positive, and gram stained as gram-positive cocci. Question Ico-delete Highlights What is the most likely causative organism of the patient's conjunctivitis?

orrect answer: Staphylococcus aureus Explanation Staphylococcus aureus is a gram-positive staining cocci that is catalase positive, coagulase positive, and frequently beta hemolytic on blood agar. It is probably the 2nd most common bacterial isolate of human infections behind Escherichia coli and the most common cause of bacterial endophthalmitis. The organism has been described as an etiologic agent of many infections, including but not limited to, conjunctivitis, endocarditis, septicemia, abscesses, and urinary tract infections. The conjunctivitis caused by Staphylococcus aureusis usually characterized as non-severe, where there is little to no lid edema, scant purulent discharge, and normal cornea; however, in some cases the presentation can be severe. Chlamydia trachomatis is an obligate intracellular parasite with a unique biphasic life cycle. It does not gram stain, and laboratory procedures used for diagnosis include isolation in tissue culture, EIA detection of antigen, immunofluorescent staining, cytologic examination for intracytoplasmic inclusions, and by the demonstration of nucleic acid by direct hybridization or by amplification techniques. It can cause inclusion conjunctivitis and ocular trachoma. The inclusion conjunctivitis presents as an acute follicular conjunctivitis and is usually self-inoculated from an infected genitourinary site. The patient frequently complains of a foreign body presence in the eye. These symptoms are usually unilateral, and in the first 2 weeks, there is a mucoid discharge that becomes purulent. Pseudomonas aeruginosa is a gram-negative rod, non-lactose fermenting, oxidase-positive motile bacteria. Pathogenesis is due to its minimal nutritional requirements, relative resistance to antibiotics, and a host of other invasive and toxinogenic substances that it produces. It can cause a keratitis that is rapid in its development. The infection is usually the result of a previous injury to the eye, which causes an interruption in the epithelial surface and allows bacterial invasion of the underlying stroma. Scrapings from the floor of the ulcer exhibiting gram-negative rods are strongly indicative of Pseudomonas aeruginosa and should necessitate treatment. Haemophilus aegyptius is a gram-negative coccobacillus, non-motile, fastidious bacteria requiring the presence of special factors for its growth on agar media. These factors are hemin and nicotinamide adenine dinucleotide, which are present in chocolate agar but not on other isolation media. The organism is indigenous to humans. It is an important cause of a purulent conjunctivitis called 'pink eye' and can occur in outbreaks because of its contagious nature. The diffuse pink color of the sclera and the presence of a serous or purulent discharge are virtually diagnostic of Haemophilus aegyptius infection. Leukocytosis is absent. The infection is not acute in presentation. Bacillus cereus is a gram-positive (or gram-variable) rod that is aerobic, spore-forming, and is ubiquitous in nature. Bacillus cereus is an important cause of food poisoning. It has also been recognized as an ocular pathogen. The ocular infection is acute in presentation and requires aggressive intervention to save the eye. The presence of progressive corneal deterioration and ring abscess formation is a complication of panophthalmitis caused by Bacillus cereus. Except for infections with Pseudomonas aeruginosa, this finding is almost pathognomonic of Bacillus cereus. Because of the seriousness of the infection, early diagnosis is important. Patients presenting with ocular infections after trauma or in the setting of drug abuse should arouse suspicion. Acanthamoeba is a free-living amebae that can cause granulomatous amebic encephalitis and keratitis. It can not be cultured by routine culture methods. Detection is usually made by observing the free living motile organisms in a wet prep preparation. Acanthamoeba keratitis is a slow-developing corneal infection that occurs in healthy people and is usually associated with contact lens wearers. Symptoms include blurred vision, conjunctivitis, tearing, severe pain to the eye, and photophobia. The keratitis achieves an advanced stage in several days to several months and can exhibit patchy stromal infiltrates and dendriform epithelial involvement without frank corneal ulceration in its early stages. Candida albicans is a yeast. Yeasts appear on gram stain as large gram-positive organisms, approximately 3-5 times larger than gram-positive cocci, and are nonhemolytic on blood agar. They are aerobic and generally grow well on most non-selective agar media. Endophthalmitis due to yeast is generally a common and serious complication of intravenous drug use. Candida albicans is the most common fungal cause. It is usually of hematogenous origin where the patient has infective endocarditis or some other infective process occurring. The symptoms are blurred vision, decreased vision, white cotton appearing exudative lesions in the choroid and retina with vitreous haziness, and eye pain. A definitive diagnosis is made by obtaining vitreous fluid for gram stain and culture.


Related study sets

ExamFX Field Underwriting Procedures

View Set

Social Psychology Exam #3, Chapter #7

View Set

Programing Chapter 10 Quiz Characters, Strings, and string Class

View Set

What is the supreme law of the land?

View Set

Capstone AC & DC Circuits Review

View Set